84

Ευκλειδης Β 74

  • Upload
    demi-de

  • View
    242

  • Download
    1

Embed Size (px)

DESCRIPTION

Ευκλειδης Β 74

Citation preview

Page 1: Ευκλειδης Β 74
Page 2: Ευκλειδης Β 74

' '""" . }' �ροvιg

fΥ.1_ςιθnματικά

.ι. •··�"'

Μαθηματικά Α Γυμνασίου

Συνοδεύεται από ένθετο με τις λύσεις των ασκήσεων

του σχολικού βιβλίου

11�1

Π���οψορική Πληροφορική

Β' Γυμνασίου

Μαθηματικά Αναλυτικές λύσεις των ασκήσεων

του σχολικού βιβλίου Γ Γυμνασίου

ΕΚΔΟΣΕΙΣ ΠΑΤΑΚΗ

ατικά

Μαθηματικά Α Γυμνασίου

Συνοδεύεται από ένθετο

με τις λύσεις των ασκήσεων του σχολικού βιβλίου

Μαθηματικά Β' Γυμνασίου

Από ένα βιβλίο για κάθε τάξη του Γυμνασίου

�� Μαθηματικά • ·

Μαθηματικά Γ Γυμνασίου

------ _____7ά {;τ.rv- !fdrn lftJVI!eyz; .3'*'7t�zo!_ --·..h:ιcιf:;l',f'VΎa ��ia ?-l'f�a� σ� .VJ'�-n �ιl'f-n. __

Νέες Εκδόσεις της Ελληνικής

Μαθηματικής Εταιρείας

� �

ΕΜΗΝιΚΗ ΜΑΘΗΜΑτlΚΗ ΕΤΑΙΡΕΙΑ n,;,, ,, ,,,. , .• ,, Χ. Δ. λί'ιιηpάνtnς και S. Κ. Ch8krιιb•rtl

ΜΑΘΗΜΑΤ ΠcριοδΙkι'\ Επιστημονικο\ �

Page 3: Ευκλειδης Β 74

ΕΛΛΗΝΙΚΗ ΜΑΘΗΜΑΤΙΚΗ ETAIPEIA Τ ε ύ χ ο ς 74 • Οκ τ ώβριος • Ν ο έ μβριος • Δ ε κ έ μβρ ιος 2009· Έτ ο ς λθ' • Ε υ ρ ώ: 3,50

e-mail: [email protected] www.hms.gr

ΜΑΘΗΜΑΤΙΚΟ ΠΕΡΙΟΔΙΚΟ ΓΙΑ ΤΟ ΛΥΚΕΙΟ

ΠΕΡΙΕΧΟΜΕΝΑ ../ Ισαάκ Νεύτων

../ Ημερολόγια

../ Μαθηματικοί Διαγωνισμοί • Μαθηματικές Ολυμπιάδες

../ Homo Mathematicus

Μαθηι,ιατικά Α' Τάξης ../ Άλγεβρα: Εξισώσεις δευτέρου βαθμού • Σύνολα Συναρτή·

2

7

9

22

σεις • Η συνάρτηση f(x) = αχ+β 26 ../ Γεωμετρία: Παραλληλόγραμμα και Τραπέζια 31

Μαθηματικά Β' Τάξη«; ../ Άλγεβρα: Πολυώνυμα και πολυωνυμικές εξισωσεις 36 ../ Γεωμετρία: Εμβαδά των Ευθυγράμμων Σχημάτων 41 ../ Κατεύθυνση: Εξίσωση Ευθείας 52

Μαθηματικά Γ' Τάξης ../ Μαθηματικά Γενικής Παιδείας: Στατιστική 58 ../ Μαθηματικά Κατεύθυνσης: Γενικά θέματα ορίων • συνέχειας63

• • • • • • • • • • • • • • • • • • • • • • ../ Το Βήμα του Ευκλείδη ../ Σκέψεις πάνω σε μια ανισότητα ../ Ο Ευκλείδης προτείνει ... ../ Τα Μαθηματικά μας Διασκεδάζουν ../ Στήλη του Μαθητή

68 71 73 78 81

Γράμμα της Σύνταξης Αγαπητοί μαθητές και συνάδελφοι,

Είμαστε αισίως στο 2010. Ευχόμαστε η νέα χρονιά να

χαρίσει υγεία και δημιουργικότητα σε όλους μας καθώς

και επιτυχία στους στόχους μας.

Είναι διaπιστώμενο χρόνια τώρα ότι μετά τις γιορτές των

Χριστουγέννων, οι μαθητές βιάζονται να aτενίσουν το

τέλος της σχολικής χρονιάς και σε μας μένει το δύσκολο

καθήκον να συγκρατήσουμε aυτή την αισιοδοξία τους και

να τους πείσουμε ότι έχουμε πολύ χρόνο ακόμη ο οποίος

πρέπει να aποβεί δημιουργικός .

Για να aντιμετωπίσουμε aυτές τις δύο aλληλοσυγκρουό·

μενες aπόψεις θα προσπαθήσουμε τα δύο τελευταία

τεύχη του περιοδικού μας του Ευκλείδη θ', που aνaφέρ·

οντaι σε θέματα μερική ή ολικής επανάληψης, να τα έχο·

υ με έτοιμα μέχρι τις αρχές Μαρτίου 201 Ο. Θα παρακαλούσαμε οι προτεινόμενες συνεργασίες να

είναι και σε ηλεκτρονική μορφή για την καλύτερη aπόδο·

ση και του όρθρου, αλλά και του χρόνου που κερδίζεται

για την έγκαιρη έκδοση του περιοδικού .

Με τη καλή διάθεση για προσφορά και aποτελεσμaτικότ·

ητa στις προσπάθειες μας, ευχόμαστε ότι καλύτερο για

τον καθένα μας.

Με πολλές ευχαριστίες και προσμονή • Ο Πρόεδρος της Συντακτικής Επιτροπής Γ.Σ. Τaσσόπουλος

Ο Αντιπρόεδρος της Συντακτικής Επιτροπής Β. Ευσταθίου

Σύνθεση εξωφύλλου: Γραμματόσημο των Ελληνικών Ταχυδρομείων

Ευρώπη - 1983 ••••••••••••••••••••••••••••••••••••••••••••••••••••••••••••••••••••••••••••••

ΕΚΔΟΣΗ ΤΗΣ ΜΑΘΗΜΑ1ΙΚΗΣ ΗΑΙΡΕΙΑΣ ΠΑΝΕΠΙΣΤΗΜΙΟΥ 34 106 79 ΑΘΗΝΑ Tlj..: 210 3617784 • 3616532 Fox: 2103641025

Εκτελεστική Γραμματεία Πρόεδρος: Τασσόπουλος Γιώργος

Εκδότης: Αντιπρόεδρος: ΚαλογερόποιΜος Γρηvόριος Ευσταθίου Βαγγέλης

Διευθυντής: Τυρλής Ιωάννης

Επιμέλεια Έκδοσης: Τασσόπουλος Γιώργος

Ευσταθίου Βαγγέλης

Γραμματέας: Χριστόπουλος Παναyιώτης Μέλη: Αργυράκης Δ.

Δρούτσας Π. Λουρίδας Σ. Ταπεινός Ν.

Κωδικιiς ΕΛ.ΊΆ.: 2055 ISSN: 1105- 7998

Αθανασόπουλο<; Γεώργιος Αναστασίου Γιάννης Ανδρουλακάκης Νίκος Αντωνόπουλος Νίκος Αργυράκης Δημήτριος Βακαλόπουλος Κώστας Δρούτσας Παναγιώτης Ευσταθίου ΒαyyεΛης Ζαχαρόπουλος Κων/νος Ζώτος ΒαyyεΛης Κακκαβάς Απόστολος Καλίκας Σταμάτης ΚανεΛλος Χρήστος Καραγκούνης Δημήτρης Καρακατσάνης Βασίλης Καρκάνης Βασίλης Κατσούλης Γιώργος Κερασαρίδης Γιάννης Καρδαμίτσης Σπύρος Κηπουρός Χρήστος Κλάδη Κατερίνα

Συντακτική επιτροπή Κόντζιας Νίκος Κοτσιφάκης Γιώργος Κουτρουμπέλας Κώστας Κυριαζής Ιωάννης Κυριακόπουλος Αντώνης Κυριακόπουλος Θανάσης Κυριακοπούλου Κων/να Κυβερνήτου Χρυστ. Λαζαρίδης Χρήστος Λάτmας Λευτέρης Λουρίδας Σωτήρης Μαλαφέκας Θανάσης Μανωλάκου Στοματική Μαυρογιαννάκης Λεωνίδας Μενδρινός Γιάννης Μεταξάς Νικόλαος Μπρίνος Παναγιώτης Μυλωνάς Δημήτρης Μώκος Χρήστος Πανουσάκης Νίκος Ρέγκλης Δημήτρης

Σαϊτη Εύα

Σταϊκος Κώστας

Στάϊκος Παναγιώτης

Στρατής Γιάννης

Ταπεινός Νικόλαος

Τασσόπουλος Γιώργος

Τζιώτζιος Θανάσης

Τριάντος Γεώργιος

Τσαγκάρης Ανδρέας

Τσατούρας Ευάγγελος

Τσικαλουδάκης Γιώργος

Τσιούμας Θανάσης

Τυρλής Ιωάννης

ΦανεΛη Άννυ

Χαραλαμποπούλου Λίνα

Χαραλάμπους Θάνος

Χριστιάς Σπύρος

Χριστόπουλος Παναγιώτης

Ψύχος Βαyyέλης ••••••••••••••••••••••••••••••••••••••••••••••••••••••••••••••••••••••••••••••

• Τα διαφημιζόμενα βιβλία δε σημαίνει ότι προτείνονται από την Ε.Μ.Ε.

• Οι συνεργάτες, τα άρθρα, οι προτεινόμενες ασκήσεις, οι λύσεις ασκήσεων κτλ. πρέπει να σrεΛνονται έγκαιρα, σrα γραφεία της Ε.Μ.Ε.

με την ένδειξη "Για τον Ευκλεffiη β'". Τα χειρόγραφα δεν εmσrρέφονται.

Τιμή Τεύχους ευρώ 3,50 Ετήσια συνδρομή (12,00 + 2,00 Ταχυδρομικά= ευρώ 14,00) Ετήσια συνδρομή για Σχολεία ευρώ 12,00 Το αντίτιμο για τα τεύχη που παραyyεΛνονται σrεΛνεται με απλή επιταγή σε διαταγή Ε.Μ.Ε. Ταχ. Γραφείο Αθήνα 54 Τ.Θ. 30044 ή

πληρώνεται σrα γραφεία της Ε.Μ.Ε.

Εκτύπωση: ΙΝΠΡΠΡΕΣ Α.Ε. τηλ.: 21 Ο 8/60330 Υπειίθuνος τuποypαφείοu: Β. Σωτηριάδης

Page 4: Ευκλειδης Β 74

Ισαάκ Νεύτωv «Χτίζουμε πάρα πολλά τείχη και όχι αρκετές γέφυρες.»

Παναγιώτης Παπαχρήστου

α Χριστούγεννα του 1 642 έχουν κάποιο κοινό με την ιστορία που εκτυλίσσεται στο έργο "Νύχτα Χριστουγέννων" του Κάρολου Ντίκενς. Και στις δύο περιπτώσεις συνέβη ένα θαύμα. Ειδικότερα τη νύχτα των Χριστουγέννων του

1 642 (με το παλιό ημερολόγιο) η Λάχεση η Κλωθώ και η Άτροπος, οι τρεις μοίρες που καθορίζουν το πεπρωμένο των ανθρώπων, πρέπει να αντιμετώπισαν έναν σοβαρό προβληματισμό . Έπρεπε να αποφασίσουν αν το μωρό που γεννι)θηκε εκείνη τη νύχτα στο αγρόκτημα Γούλσθορπ (Woolstl1oφe) κοντά στην κωμόπολη Γκράνθαμ (Granthaιη) του Λινκοσάιρ της Αγγλίας θα "έβγαζε" τη νύχτα. Σύμφωνα με τη μητέρα του ήταν τόσο

Sir lsaac Newton

μικροσκοπικό, που θα μπορούσε να χωρέσει σε μια κατσαρόλα με χωρητικότητα περίπου ενός λίτρου. Οι συγγενείς πίστευαν ότι δεν θα αντίκριζε το φως του ήλιου της επόμενης μέρας. Η Άτροπος ήταν έτο ιμη να κόψει με τα ψαλίδια της το νήμα της ζωής που είχε αρχίσει να υφαίνει η Κλωθώ. Φαίνεται όμως ότι η Λάχεση που "έβλεπε" το μέλλον είχε διαφορετική γνώμη . Ίσως προέβλεψε κάτι σημαντικό για το μικρί) ανθρωπάκι. Έτσι άφησαν την Κλωθώ να υφαίνει το νήμα της ζωής του μέχρι το 1 727 .

Το όνομα του μωρού ήταν Ισαάκ Νεύτων. Γεννήθηκε την ίδια μέρα που πέθανε ο Γαλιλαίος (Gίllil�o Galilei, 1 564- 1 642). Το γεγονός είναι προφανώς τυχαίο, αλλά δεν στερείται συμβολισμού και τίποτα δc\'

μας εμποδίζει να το θεωρούμε σαν μια μοιραία συγκυρία για τη συνέχεια μιας επιστημονικι)ς επανάστασης που είχε αρχίσει με τον Νικόλαο Κοπέρνικο (Nicolaus Copen1icιιs, 1473-1 543) , συνεχίστηκε με τον Τζιορντάνο Μπρούνο (Jordano Βrιιηο , l 548-1600), θεμελιώθηκε με παρατηρήσεις και μετρήσεις από τον Κέπλερ (Johannes Kepler, 1 571- 1 630) και ερμηνεύτηκε από το τυχερό μωρό της νύχτας των Χριστουγέννων. Η επανάσταση είχε να κάνει με το ερuΗημα για τη θέση της Γης και της aνθρωπότητας στην απεραντοσύνη του σύμπαντος.

Ο Κοπέρνικος υπήρξε ο πρώτος, στα χρόνια της αναγέννησης, που Nicolaus Copernicu!i

αμφισβι)τησε την άποψη ότι η Γη αποτελεί το κέντρο του σύμπαντος και γύρω από αυτιΊν περιστρέφονταν όλοι οι άλλοι πλανήτες και ο Ήλιος (γεωκεντρικό σύστημα) . Δε επρι'>ι-:ατο όμως για μια εντελώς καινοφαν11 άποψη . Ήδη ο Αρίσταρχος ο Σάμιος (3 1 0 π .Χ. -230 π .Χ.) , 1 800 χρόνια πριν τον Κοπέρνικο, ισχυριζόταν ότι στο κέντρο του πλανητικού συστήματος βρίσκεται ο Ήλιος και όχι η

Johunnes Kepler

Γη . Την αντίληψη του γεωκεντρικού συση1ματος την είχε εισαγάγει ο

Αριστοτέλης και την είχε τεκμηριώσει ο Κλαύδιος Πτολεμαίος με τη θεωρία των επικύκλων, η οποία εξηγούσε με αρκετή ακρίβεια πολλά ουράνια φαινόμενα, όχι όμως όλα. Τη γεωκεντρική αντίληψη την υιοθέτησε και η Καθολική Εκκλησία και της προσέδωσε βαρύτητα δόγματος, δ ιότι συμφωνούσε σε πολλά σημεία με τις Γραφές. Ενδεχομένως αυτή η συμφωνία με τις Γραφές δεν ήταν ο αποφασιστικός παράγων για την δογματοποίηση της γεωκεντρικής άποψης, αλλά αυτό τουλάχιστον ισχυρίζονταν οι εκπρόσωποί της. Είναι χαρακτηριστική η απάντηση που έδωσαν κάποιοι καρδινάλιοι, όταν παρατήρησαν με τη βοήθεια του πρωτόγονου τηλεσκοπίου του Γαλιλαίου

ΕΥΚΛΕΙΔΗΣ Β' 74 τ.2/2

Page 5: Ευκλειδης Β 74

----------------------------------- Ισαάκ Νεύτων-----------------------------------τους δορυφόρους του Δία: "Αν πρόκειται να προτιμ1Ίσουμε ανάμεσα στο Δόγμα και την παρατήρηση, τότε προτιμούμε το Δόγμα.

Jordano Bruno

Η διδασκαλία του Κοπέρνικου τοποθετούσε τον Ήλιο στο κέντρο του πλανητικού μας συστήματος. Η Γη και άλλοι πλανήτες περιστρέφονται σε κυκλικές τροχιές γύρω από τον Ήλιο. Το βιβλίο του De Ι'eνο!ιιtίοnίbιιs orbiιιm

coe!e:-;tίιιιn (Α ναφορικά με τις περιστροφές των ουράνιων σωμάτων) θεωρείται συχνά ως η αφετηρία της σύγχρονης αστρονομίας, που σηματοδότησε την επιστημονική επανάσταση . Ίσως δεν στερείται σημασίας το γεγονός ότι η λέξη Ι'eνο!ιιtίοn εκφράζει στα Αγγλικά τόσο την επανάσταση όσο και την περιστροφή ενός σώματος. Ο Jordano Bnιno τοποθέτησε το θέμα σε ένα ευρύτερο πλαίσιο,

πέρα από τη γεωμετρική ερμηνεία. Θεωρούσε ότι η ανθρωπότητα δεν κατείχε κάποια προνομιακή θέση στο σύμπαν και ότι η Γη δεν ήταν ο μόνος κατοικήσιμος κόσμος. Διακήρυττε ότι υπάρχουν άπειροι κόσμοι και ότι τα άστρα που λαμπυρίζουν τη νύχτα στο στερέωμα είναι ήλιοι ανάλογοι με τον δικό μας. Πίστευε σε μια θεότητα που φρόντιζε όλους αυτούς τους άπειρους κόσμους. Αυτές οι απόψεις δεν συμφωνούσαν με τα δόγματα της Καθολικ1Ίς Εκκλησίας και, επειδή έτσι έθεταν σε αμφισβήτηση τις Γραφές, αποτελούσαν κίνδυνο για τη θέση των εκπροσώπων της Εκκλησίας ως αυθεντικών ερμηνευτών των Γραφών και ως υπερασπιστών της πίστης. Το σωτήριο έτος 1600 (Ι 7 Φεβρουαρίου) το σώμα του JoΓdano Βωηο κάηκε στην πυρά της Ιεράς Εξέτασης και η ψυχή του πέταξε, για να συναντήσει τους κόσμους που ονειρευότανε. Δεν υπέγραψε τη "δ1)λωση μετάνοιας" που του ζητούσε η εκκλησία, για να σώσει τη ζωή του και έτσι τίμησε σε καιρούς χαλεπούς τα ιδεώδη στα οποία πίστευε. Με τα δεδομένα της εποχής του και τις αντιλήψεις της θρησκείας σίγουρα υπήρξε αιρετικός. Δεν ήταν ο μόνος αιρετικός που κατάφερε να δει μακρύτερα και

Galileo Galilei

καθαρότερα. Κάποιοι ερευνητές θεωρούν ότι οι ιδέες του ήταν πανθεϊστικές και υποστηρίζουν ότι αυτή η συνιστώσα των ιδεών του ήταν εκείνη που προκάλεσε την καταδίκη του . Σε οποιαδήποτε περίπτωση πάντως υπήρξε ένας μάρτυρας των ιδεών του .

Ο Γαλιλαίος παρατήρησε με το τηλεσκόπιό του την κίνηση των δορυφόρων του Δία γύρω από αυτόν τον πλαν1Ίτη. Τ ο γεγονός ότι υπήρχαν κάποια ουράνια σώματα που δεν περιστρέφονταν γύρω από τη Γη κλόνιζε τη γεωκεντρική άποψη . Έσωσε τη ζωή του, αλλά καταδικάστηκε να ζ1Ίσει μέχρι τον θάνατό του αιχμάλωτος στο σπίτι του. Η εξωραϊστικ1Ί διάθεση της ιστορίας τον θέλει να μουρμουρίζει, βγαίνοντας από το δικασηΊριο της Ιεράς Εξέτασης, την παροιμιώδη φράση "και όμως κινείται-e puι· si ιηuονe". Αυτό δεν

Περίπου τριακόσια χρόνια μετά το ανάθεμα η Καθολtκ1Ί Εκκλησία το απέσυρε. Την I 0.1 Ι. Ι 979.

κατά τον εορτασμό των εκατό ετών από τη γέννηση του Aibert Einstein, ο πάπας Ιωάννης - Παύλος Β' εκ μέρους της Παπικής Ακαδημίας των Επιστημών αναφέρθηκε και στη συμπεριφορά της Εκκλησίας προς

ΕΥΚΛΕΙΔΗΣ Β' 74 τ.2/3

Page 6: Ευκλειδης Β 74

----------------------------------- Ισαάκ Νεύτων-----------------------------------τον Γαλιλαίο. Ζήτησε μάλιστα και την αναθεώρηση της καταδικαστικής αποφάσεως του Παπισμού σε βάρος του Γαλιλαίου . Είπε : «Η μεγαλοσύνη του Γαλιλαίου, καθώς και εκείνη

του Αλβέρτου Αϊνστάιν, είναι γνωστή εις όλους, αλλά ο πρώτος

υπέφερε πολύ από τους θεσμούς και τους ανθρώπους της (Παπικής)

Εκκλησίας. Τούτο δεν μπορούμε να το αποκρύψομε ... Θέλω, όπως

οι θεολόγοι, έτσι και οι φυσικοί επιστήμονες και οι ιστορικοί,

κινούμενοι από πνεύμα ειλικρινούς συνεργασίας, να εξετάσουν και

πάλι την υπόθεση Γαλιλαίου, να αναγνωρίσουν τα σφάλματα, όπου

αυτά υπάρχουν και να διαλύσουν τη δυσπιστία, η οποία υπάρχει ακόμη μεταξύ πολλών ανθρώπων δια την υπδθεση

εκείνη και η οποία εμποδίζει τη γόνιμη ομόνοια μεταξύ επιστήμης και πίστεως, μεταξύ Εκκλησίας και κόσμου».

Και ο Πάπας πρόσθεσε: «Οι παρεξηγήσεις γύρω από τη δίκη (τού Γαλιλαίου) έχουν προκαλέσει

οδυνηρές και πικρές αντιπαραθέσεις κατά τη διάρκεια των αιώνων»

Σε κάθε περίπτωση , τα 300 χρόνια, ακόμα και για την Αγία Καθολική Εκκλησία είναι μεγάλο χρονικό διάστημα, για να αναγνωρίσει τα σφάλματά της και την εγκυρότητα της επιστήμης.

Ο Κέπλερ βελτίωσε το μοντέλο του Κοπέρνικου και με κοπιαστικές παρατηρήσεις και μακροσκελείς υπολογισμούς απέδειξε ότι οι τροχιές των πλανητών είναι ελλείψεις στη μια εστία των οποίων βρίσκεται ο Ήλιος. Σύμφωνα με τους νόμους του Κέπλερ ήταν γνωστές οι ελλειπτικές τροχιές των πλανητών και έτσι μπορούσε να εντοπιστεί η θέση καθενός με απόλυτη ακρίβεια. Άλλο όμως θέμα είναι ο εντοπισμός της θέσης ενός πλανήτη και άλλο η αιτιολόγηση της μορφής της τροχιάς του . Εδώ ακριβώς εκδηλώνεται η μεγαλοφυ·ια του Νεύτωνα.

Το "μωρό των Χριστουγέννων" δεν άρχισε με καλούς οιωνούς τη δύσκολη ζωή του . Ο πατέρας του είχε πεθάνει τρεις μήνες πριν από τη γέννησή του . Ακόμη χειρότερα, από πολύ ενωρίς εκτός από την πατρική ορφάνια γνώρισε και μια μορφή μητρικής ορφάνιας, ενώ η μητέρα του ζούσε ακόμα. Μερικά χρόνια μετά τη γέννησή του η μητέρα του ξαναπαντρεύτηκε με έναν κληρικό από κάποιο κοντινό γειτονικό χωριό και έστειλε τον Νεύτωνα να ζήσει και να περάσει τα ευαίσθητα παιδικά του χρόνια κοντά στη γιαγιά του . Δεν είναι πολλές οι μανάδες που θα έκαναν κάτι τέτοιο και θα προσθέτανε στην

τ α εμβαδά που σαρώνει η επιβατική ακτίνα ενός πλανήτη σε ίσα χρονικά

διαστήματα είναι ίσα

αναπόφευκτη έλλειψη της πατρικής φροντίδας και προστασίας και την έλλειψη της γλυκιάς μητρικής αγκαλιάς. Κατά πάσαν πιθανότητα σε αυτά τα βασανιστικά πρώτα χρόνια της ζωής του οφείλονται ο πολύπλοκος και προβληματικός χαρακτήρας του και η τάση για απομονωτισμό . Ενδεχομένως αυτή η εσωστρέφεια ήταν η αιτία της aφοσίωσης του Νεύτωνα με τόσο πάθος στις μελέτες του .

Είναι παράξενο, αλλά μελετώντας κάποιος τη ζωή του διαπιστώνει ότι πολλοί από τους επιστημονικούς θριάμβους ξεκινούν από τραγικά γεγονότα. Η τελευταία επιδημία βουβωνικής πανώλης που έπληξε την Αγγλία το 1666 έγινε

αιτία να κλείσει το Πανεπιστήμιο του Κέμπριτζ για ένα χρόνο και ο Νεύτων να επιστρέψει στο αγρόκτημά του . Ο λα"ίκός μύθος ισχυρίζεται ότι η αφετηρία για την ανακάλυψη του Νόμου της

Παγκόσμιας Έλξης ήταν ένα μήλο που έπεσε στο κεφάλι του, όταν ξεκουραζόταν κάτω από μια μηλιά στο οικογενειακό αγρόκτημα. Μια άλλη εκδοχή αναφέρει ότι απλώς παρατήρησε κάποιο μήλο να πέφτει

ΕΥΚΛΕΙΔΗΣ Β ' 74 τ.2/4

Page 7: Ευκλειδης Β 74

----------------------------------- Ισαάκ Νεύτων-----------------------------------κάτω από μια μηλιά. Κάποιοι άλλοιθεωρούν φανταστική αυτή την αφήγηση . Είναι γεγονός πάντως ότι από τη στιγμή κατά την οποία εμφανίστηκαν οι μηλιές πάνω στη Γη , τα μήλα έπεφταν, όταν ωρίμαζαν, όπως έπεφταν οι πέτρες που εκσφενδονίζονταν, ή τα νερά των καταρρακτών. Το ερώτημα για τον Νεύτωνα ήταν: "γιατί δεν πέφτει προς τη Γη και η Σελήνη, η οποία αιωρείται μακάρια πάνω από το κεφάλι μας,

δεδομένου ότι δεν υπάρχει κάποιο ορατό μέσο στήριξής της;"

Είχε να επιλέξει ανάμεσα σε δύο εξηγήσεις. Η πρώτη ήταν ότι οι φυσικοί νόμοι που ισχύουν στην επιφάνεια της Γης δεν ισχύουν στην περιοχή του χώρου στον οποίο βρίσκεται η Σελήνη . Η άλλη επιλογή ήταν να αποδεχτεί την άποψη ότι η Σελήνη "πέφτει" πραγματικά προς τη Γη . Ο Νεύτων επέλεξε τη δεύτερη εξήγηση με βάση έναν απλό και όμορφο συλλογισμό :

Γνωρίζουμε ότι η Σελήνη περιστρέφεται γύρω από τη Γη και ότι η Γη, μαζί με τη Σελήνη, περιστρέφεται γι)ρω από

τον ήλιο. Είναι ειJκολο να αντιληφθεί κάποιος, ότ, σε αυτή τη σιJνθετη κίνηση, κάποια στιγμή η Γη θα βρεθεί σε

περιοχή που προηγουμένως βρισκόταν η Σελήνη. Αν λοιπόν στην περιοχή στην οποία βρισκόταν η Σελήνη δεν υπάρχει

ένα είδος βαρύτητας, όπως αυτή που υπάρχει στη Γη και η οποία μας κρατά "καρφωμένους" στην επιφάνειά της, τότε

και εμείς, όταν η Γη φτάσει σε τέτοια περιοχή, θα βρεθούμε κολλημένοι στα ταβάνια των δωματίων μας ή θα

απογειωθοιJμε. Κάτι τέτοιο δεν έχει παρατηρηθεί.

Το συμπέρασμα στο οποίο κατάληξε ήταν ότι και η Σελήνη κινείται σε ένα βαρυτικδ πεδίο που παράγει η παρουσία της Γης. Έτσι η Σελήνη έλκεται από τη Γη με την ίδια, από ποιοτική άποψη, δύναμη με την οποία έλκεται από τη Γη κάθε αντικείμενο που βρίσκεται κοντά ή πάνω στην επιφάνεια της Γης. Η ίδια δύναμη είναι εκείνη που αναγκάζει μια πέτρα να πέφτει προς την επιφάνεια της Γης, αν την αφήσουμε από κάποιο ύψος και η δύναμη που διατηρεί την Σελήνη σε τροχιά γύρω από τη Γη . Είναι βέβαιο όμως ότι η Σελήνη δεν "πέφτει" με την έννοια που πέφτει μια πέτρα στην επιφάνεια της Γης. Θα μπορέσουμε να ξεπεράσουμε αυτόν τον εννοιολογικό σκόπελο, αν διασαφήσουμε την έννοια της (ελεύθερης) πτώσης, όπως την αντιλαμβάνονται οι φυσικοί. Λένε, λοιπόν, ότι ένα υλικό σώμα βρίσκεται σε ελεύθερη πτώση, αν και μόνο αν η μοναδική δύναμη που του ασκείται είναι η δύναμη της βαρύτητας. ΑυηΊ η απαίτηση ικανοποιείται τόσο στην περίπτωση της πέτρας, όσο και στην περίπτωση της Σελήνης, διότι η μόνη δύναμη που δέχεται καθένα από αυτά τα αντικείμενα είναι η ελκτική βαρυτική δύναμη από τη Γη . Αποσυμπλέκοντας, λοιπόν, τη γεωμετρική αντίληψη της πτώσης (σαν μια κίνηση ενός σώματος που αναπόφευκτα καταλήγει τελικά στην επιφάνεια της Γης) από τη φυσική δυναμική της έννοια, συνειδητοποιούμε ότι τόσο η Σελήνη όσο και η πέτρα "πέφτουν" προς τη Γη . Η διαφορά είναι ότι η πέτρα φτάνει στη Γη, αλλά η Σελήνη δεν φτάνει ποτέ. Δεν είναι δύσκολο να κατανοηθεί ότι η διαφορά στη συμπεριφορά τους οφείλεται στην κατάσταση των αντικειμένων, όταν έφερε αποτελέσματα πάνω τους η επίδραση της βαρύτητας. Όταν λέμε ότι μια πέτρα πέφτει θεωρούμε υποσυνείδητα ότι την αφήνουμε να κινηθεί από κάποιο ύψος όπου την κρατούμε ακίνητη . Η αρχική ταχύτητα της πέτρας ήταν μηδενική . Σε αυτή την περίπτωση η τροχιά της είναι μια ευθεία γραμμή . Η Σελήνη όμως, οποιαδήποτε θεωρία και αν υιοθετήσουμε για τη δημιουργία της, είχε πάντοτε κάποια ταχύτητα ως προς τη Γη . Ας υποθέσουμε ότι στην περιοχή της Σελήνης φτάνει ο σούπερμαν και την αναγκάζει να ακινητοποιηθεί ως προς τη Γη . Τότε οπωσδήποτε η Σελήνη θα πέσει στην αγκαλιά της μανούλας Γης, είτε είναι γέννημά της, είτε υιοθετημένη κόρη της. Με αυτές τις σκέψεις και με κατάλληλους υπολογισμούς κατόρθωσε να προσδιορίσει τη μορφή της φόρμουλας με την οποία υπολογίζεται το μέτρο της δύναμης που ασκεί η Γη σε κάθε αντικείμενο που βρίσκεται στη γειτονιά της. Οι ιστορικοί της επιστήμης ισχυρίζονται ότι ο Νεύτωνας πέτυχε την πρώτη ενοποίηση δυνάμεων στη

ΕΥΚΛΕΙΔΗΣ Β' 74 τ.2/5

Page 8: Ευκλειδης Β 74

----------------------------------- Ισαάκ Νεύτων-----------------------------------Φυσική , δηλαδή την ενοποίηση της γήινης και της ουράνιας βαρύτητας σε μια μοναδική δύναμη , τη δύναμη της βαρυτικής έλξης των υλικών μαζών, που εκφράζεται από τον νόμο της παγκόσμιας έλξης, τον οποίο ο ίδιος διατύπωσε.

Ακόμη δεν έχουμε αναφέρει το καλύτερο . Ο Kepler απέδειξε με τη λεπτομερή παρατήρηση και μέτρηση ότι οι τροχιές των πλανητών είναι ελλείψεις. Στη μια από τις εστίες των ελλείψεων βρίσκεται ο Ήλιος. Γιατί όμως οι τροχιές των πλανητών είναι ελλείψεις; Η απάντηση στο ερώτημα αυτό ολοκλήρωσε τον επιστημονικό θρίαμβο του Νεύτωνα. Είχε ήδη προσδιορίσει τη μαθηματική μορφή του νόμου για τη βαρυτική δύναμη που ασκεί ο Ήλιος στους πλανήτες. Δεν είχε όμως τα μαθηματικά εργαλεία για τη μαθηματική αξιοποίηση του νόμου . Έκανε, λοιπόν, το αναμενόμενο : επινόησε το εργαλείο που χρειαζόταν. Το εργαλείο αυτό ήταν ο Απειροστικός Λογισμός, μια μαθηματική τεχνική , για να διαχειρίζεται aπειροστές ποσότητες (τα Fluxίons, όπως τα ονόμασε ο ίδιος) . Η τεχνική αυτή του επέτρεπε να προσδιορίζει εφαπτόμενες σε καμπύλες (Διαφορικός Λογισμός) και να υπολογίζει τα εμβαδά των χωρίων που ορίζονταν από καμπύλες γραμμές (Ολοκληρωτικός Λογισμός) . Το αποτέλεσμα ήταν ότι οι

λύσεις της εξίσωσης της παγκόσμιας έλξης, με κατάλληλους μετασχηματισμούς και τη βοήθεια του

aπειροστικού λογισμού οδηγούσαν σε εξισώσεις που απεικόνιζαν κωνικές τομές και στη συγκεκριμένη

περίπτωση ελλείψεις. Ο Νεύτων ήταν πολυπράγμων και επίμονος ερευνητής μέχρι aυτοκαταστροφικού

σημείου. Για παράδειγμα, τοποθετούσε μέσα στα μάτια του μικρά κομμάτια ξύλου, για να μελετήσει τα

φαινόμενα της όρασης. Όταν δίδασκε στο Κέμπριτζ, είχε τοποθετήσει στο δωμάτιό του κλιβά\'ους. στους

οποίους έλιωνε διάφορα μέταλλα, στην προσπάθειά του να επιτύχει μεταστοιχείωση ευτε/.6)\' μετάλλων

σε ευγενή μέταλλα, όπως σε χρυσό και άργυρο. Σε τρίχες της κεφαλής του, που εξετάστηκαν πολύ αργότερα, βρέθηκαν ίχνη από υδράργυρο, τα οποία φαίνεται ότι ήσαν τα κατάλοιπα των αλχημιστικιον

προσπαθειών του . Πολλοί αποδίδουν τις περιοδικές ψυχολογικές διαταραχές του σε δηλητηρίαση από

υδράργυρο. Ασχολήθηκε με την Οπτική και ήταν ο πρώτος που ανακάλυψε ότι το λευκό ηλιακό φως

είναι σύνθετο και αναλύεται στα απλά χρώματα της ίριδας. Εισήγαγε τη σωματιδιακή θεωρία του φωτός

και ήταν σε μόνιμη και οξύτατη αντιπαράθεση με τον Huygens, ο οποίος ήταν υποστηρικτής της κυματικής φύσης.

Ήταν βαθύτατα θρησκευόμενο άτομο και επίμονος μελετητής των Γραφών. Δεν νυμφεύθηκε και δεν

Το μνημείο του στο Westminster

άφησε aπογόνους. Δεν αναφέρονται συναισθηματικά ειδύλλια στη

διάρκεια της ζωής του, παρά μόνο μια σύντομη συναισθηματική σχέση του με μια συνομήλική του στα πρώιμα εφηβικά χρόνια του . Ήταν εριστικός, aπόμακρος και δύσκολος χαρακτήρας. Εκτός από τη διένεξή του με τον Huygens για τη φύση του φωτός είναι γνωστή και η αντιπαράθεσή του με τον Λάιμπνιτς για την πατρότητα του aπειροστικού λογισμού. Σήμερα γίνεται αποδεκτό ότι και οι δύο είχαν σημαντική συμμετοχή στη δημιουργία αυτού του κλάδου των σύγχρονων μαθηματικών.

Πέθανε στο Λονδίνο στις 20 Μαρτίου 1 727 (3 1 Μαρτίου με το νέο ημερολόγιο)

Η πατρίδα του, ο επιστημονικός κόσμος και οι λόγιοι της εποχής του τίμησαν την προσφορά του με πολλούς τρόπους.

ΕΥΚΛΕΙΔΗΣ Β' 74 τ.2/6

Page 9: Ευκλειδης Β 74

ΗΜΕΡΟΛΟΓΙΑ Θανάσης Π. Χριστόπουλος

Γιατί δεν γεννήθηκε κανr.ίς Έλληνας, από 16 μέχρι και το τέλος του Φεβρουαρίου το 1923; ο άνθρωπος από πολύ παλιά παραη1ρησε τις κινήσεις του Ήλιου και της Σελήνης καθώς και τις εποχές του έτους. Έτσι κατάφερε

να μετρι1σει το τροπικό έτος, τη χρονική δηλαδή διάρκεια από την έναρξη της άνοιξης μέχρι την αρχή της επόμενης άνοιξης. Ο Ίππαρχος ο Ρόδιος (περ.190 π.Χ. - \20 π .Χ . ) Έλληνας aστρονόμος, γεωγράφος, χαρτογράφος και μαθηματικός, υπο­λόγισε πως το τροπικό έτος είναι 365,2422 ημέρες, όταν σιΊμερα τα σύγχρονα ατομικά ρολόγια τον ε­πιβεβαιιί>νουν υπολογίζοντάς το σε 365 ,242 1 9878 ημέρες ι1 365 ημέρες 5 ώρες 48 λεπτά 46 δεύτερα και 0,96768 του δευτερολέπτου .

Ο Ίππαρχος ο Ρόδιος (περ.Ι90 π.Χ .- 120 π. Χ.)

Το έτος 709 από κτίσεως Ρώμης με αυτοκρα­τορικό διάταγμα του Ιουλίου Καίσαρα εφαρμό­σθηκε νέο (ηλιακό) ημερολόγιο δημιούργημα του έλληνα αστρονόμου Σωσιγένη (από την Αλεξάν­δρεια) . Αυτό το ημερολόγιο ονομάσθηκε Ιουλιανό ημερολόγιο. Πριν από το έτος 709 από κτίσεως Ρώμης (έτος 44 π .Χ.) χρησιμοποιούσαν το Ρωμαϊ­κό ημερολόγιο του Νουμά το οποίο ήταν σεληνια­κό και το οποίο είχε πολλά λάθη . Το Ιουλιανό η­μερολόγιο λέγεται σήμερα παλιό ημερολόγιο .

Ο Γάιος Ιούλιος Καίσαρας [Καίσαρ] ( 13 Ιουλίου I 00 π .Χ. - 15 Μαρτίου 44 π .Χ . )

Επειδή ο Σωσιγένης έκανε το Ιουλιανό ημερο­λόγιο για 365 ,25 ημέρες, γινόταν λάθος 365,25-365 ,242 1 9878 = 0,0078 ημέρες.

Το συσσωρευμένο λάθος το έτος 1582 είχε α­νέλθει σε 10 ημέρες. Ο Πάπας Γρηγόριος ανέθε­σε στον αστρονόμο Lilio (Αlοy.�ίιι8 Li/ίιt.\', Ναπολι­τάνος γιατρός από την Καλαβρία) να κάνει τη διόρθωση . Έτσι δημιουργι1θηκε το Γρηγοριανίι

ημερολίιγιο καλούμενο σι1μερα νέο. Η διόρθωση έγινε και η επομένη της 4'�c Οκτωβρίου 1582 ο­

νομάσθηκε 15-10-1582 και ορίστηκε ανά 400 χρόνια να έχουμε τρία λιγότερα δίσεκτα (τα έτη 2100, 2200, 2300 δεν είναι δίσεκτα). Ο κανόνας που καθόριζε τα δίσεκτα έτη στο Ιουλιανό ημερο­λόγιο ήταν απλός : Δίσεκτο είναι εκείνο που ο α­ριθμός του διαιρείται ακριβώς δι6 4, εκτός από τα επαιώνια που πρέπει, τα δυο πριοτα ψηφία που δη­λώνουν τον αιώνα, να διαιρούνται με 4.

Η Ελληνική Πολιτεία εφάρμοσε το Γρηγορια­νό ημερολόγιο το έτος 1 923. Η επομi:νη της 15-2-1923 ονομ6σ0ηκε 1-3-1923 γιαυτό αν ψάξετε στο Ληξιαρχείο δεν θα βρείτε Έλληνα γεννημένο από 1 6 μέχρι το τέλος Φεβρουαρίου το 1923 .

Η Ελληνική Εκκλησία εφάρμοσε στο εορτο­λόγιό της το νέο ημερολόγιο το έτος 1924. Η επο­μένη της 9-3 - 1 924 ονομάσθηκε και εορτάσθηκε ως 23-3- 1 924 και έτσι την 25η Μαρτίου 1924 συvε­ορτάσθηκε ο Ευαγγελισμός της Θεοτόκου και η επέτειος της Ελληνικής Επανάστασης.

Η ελληνική Εκκλησία όμως Ciφησε τον τρόπο υπολογισμού της ημερομηνίας του Πάσχα όπως ίσχυε με το παλιό Ιουλιανό ημερολόγιο . Τα συσ­σωρευμένα σφάλματα (και όχι μόνον αυτά) γίνο­νται αιτία να μη συνεορτάζουν το Πάσχα οι Καθο­λικοί και οι Ορθόδοξοι.

Οι εορτές του νέου έτους γίνονταν μέχρι την έ­ναρξη του Γρηγοριανού ημερολόγιου στις παγανι-

ΕΥΚΛΕΙΔΗΣ Β ' 74 τ.2/7

Page 10: Ευκλειδης Β 74

Ημερολόγια

στικές κοινωνίες, από την 25η Μαρτίου έως την ι η Ρώμης. Η λατρεία του συνδεόταν με κάθε αρχή η Απριλίου, με την εαρινή ισημερία, καθώς η εποχή ξεκίνημα, γι' αυτό και είναι ο πρώτος μήνας του αυτή χαρακτηρίζει την αναγέννηση της φύσης. Οι έτους. Ρωμαίοι συνέχισαν να εορτάζουν τον Μάρτιο το νέο Φεβρουάριος: Από το λατινικό επίθετο έτος, αλλά το 1 54 μ.Χ. με τις αλλαγές των ημερολο- februus (καθαρτικός), γιατί κατά τη διάρκεια του γίων, υιοθετήθηκε ως ημέρα εορτής του νέου έτους η μήνα αυτού - που ήταν ο τελευταίος στο αρχαίο ι η του Ιανουαρίου. Η αλλαγή αυτή της ημερομηνί- ρωμαϊκό ημερολόγιο -γίνονταν γιορτές εξαγνι­ας ήταν δύσκολο να γίνει αποδεκτή από τον κόσμο σμού. και αυτό διότι η Ι η Ιανουαρίου δεν συνέπιπτε με κά- Μάρτιος: Προέρχεται από το λατινικό martius ποια ιδιαίτερη αγροτική ή άλλη παράδοση . Ο Ιανου- (άρειος), γιατί ο μήνας αυτός ήταν αφ ιερωμένος άριος ονομάστηκε έτσι εξαιτίας του Θεού Ιανού ο στη λατρεία του θεού Άρη (Mars) . οποίος είχε δύο πρόσωπα το ένα κοίταγε το παρελ- Απρίλιος: Από το λατινικό apri l ius (ανοικτός), θόν και το άλλο το μέλλον. γιατί κατά το μήνα αυτό άνοιγαν οι ανθοί των λου-

Το 525 μ.Χ. η χριστιανική εκκλησία καταδι- λουδιών. Κάποιοι αποδίδουν το όνομα στο Αφρώ κάζει ανάμεσα σε άλλα το έθιμο της πρωτοχρονιάς (μεταγενέστερη μορφή της Αφροδίτης), επειδή οι ως ειδωλολατρικό, κάτι που ίσχυε έως τον μεσαί- Ρωμαίοι θεωρούσαν το μήνα αυτό αφιερωμένο σ ' ωνα. Ο Ιανουάριος καθιερώθηκε ω ς αρzή του αυτήν. έτους στον Δυτικό κόσμο μετά το 1500. Μάιο�: Προέρχεται από τη maja (μαία), τη

Τον 6ο αιώνα η Γέννηση του Χριστού τοποθε- μάνα της Γης, που τρέφει τον άνθρωπο και την τήθηκε χρονικά στο έτος 753 από κτίσεως Ρώμης και Κυβέλη, που ανάθρεψε τον Ερμή . αυτό το έτος ονομάσθηκε ΠΡΩΤΟ ΕΤΟΣ ΤΟΥ ΚΥ- Ιούνιος: Από το λατινικό junius [αυτός που ΡΙΟΥ, το επόμενο έτος ονομάσθηκε 2ο έτος του Κυ- σχετίζεται με την Ήρα (Juno)] , λόγω λατρείας της ρίου κ.λ.π. (άλλωστε ο αριθμός μηδέν ως σύμβολο θεάς Ήρας κατά το μήνα αυτό . είναι πολύ νεότερη ανακάλυψη, μετά την πρώτη χι- Ιούλιος: Παλαιότερα λεγόταν Quintilis (Πέ­λιετία .. Νεώτερες, βέβαια, μελέτες τοποθέτησαν τη μπτος). Το 44 π.Χ. ονομάστηκε Julius, προς τιμήν Γέννηση του Χριστού στο έτος 749 από κτίσεως Ρώ- του αυτοκράτορα Ιούλιου Καίσαρα, που γεννήθη­μης (δηλ. το 4 π. χ.) αλλά ήδη είχε επικρατήσει σαν κε στις 7 Ιουλίου 1 ΟΙ π.Χ. βάση απαρίθμη το έτος 753 από κτίσεως Ρώμης. Αύγουστος: Σημαίνει σεβαστός στα λατινικά

Η ονομασία των ι 2 μηνών

Όλοι μας γνωρίζουμε την ονομασία των 1 2 μηνών. Όμως γιατί τα ονόματα των μηνών είναι παρόμοια στις περισσότερες γλώσσες, ευρωπα"ίκές και μη ; Περιληπτικά αναφέρουμε ότι όλοι οι μή­νες έχουν ρωμαϊκά ονόματα:

Ιανουάριος: Προέρχεται από τον θεό Janus, τον προστάτη των δημοσίων διόδων και πυλών της

και αρχικά ονομαζόταν Sextus (Εκτος) . Το 27 π.Χ . ονομάστηκε Augustus προς τιμήν του αυτοκράτο­ρα Οκταβιανού Αυγούστου, που έγινε μονοκράτο­ρας. Κάποιοι αποδίδουν την ονομασία του στη λέ­ξη augur (οιωνοσκόπος) .

Σεπτέμβριος: Από το λατινικό septem (εφτά), γιατί ήταν ο έβδομος μήνας του αρχαίου ρωμα·ίκού ημερολογίου .

Οκτώβριος: Από το λατινικό octo (οχτώ), για­τί ήταν ο όγδοος μήνας. Είναι ο μόνος «αριθμητι­κός» μήνας που δεν περιέχει το γράμμα Μ. Έτσι είναι λάθος να λέμε Οκτώμβριος !

Νοέμβριο�: Από το λατινικό novem (εννέα) . Δεκέμβριος: Από το λατινικό decem (δέκα),

λόγω της θέσης που είχε στο αρχαίο ρωμαϊκό ημε­ρολόγιο .

�����

ΕΥΚΛΕΙΔΗΣ Β ' 74 τ.2/8

Page 11: Ευκλειδης Β 74

ΘΕΜΑ 1°

αθηματtκοί Δtαyωvtσμοί αθηματtκέG ΟλυμπtάδεG

Επιτροπή Διαγωνισμών της Ε.Μ.Ε.

70°ς ΠΑΝΕΛΛΗΝΙΟΣ ΜΑΘΗΤΙΚΟΣ ΔΙΑΓΩΝΙΣΜΟΣ

ΣΤΑ ΜΑΘΗΜΑΤΙΚΆ ''Ο ΘΑΛΗΣ" 21 ΝΟΕΜΒΡΙΟΥ 2009 ΕΝΔΕΙΚΤΙΚΕΣ Λ ΥΣΕΙΣ

Α' ΛΥΚΕΙΟΥ

Το τετράγωνο ενός θετικού αριθμού είναι μεγαλύτερο από το δεκαπλάσιο του αριθμού κατά 75. Να βρεθεί ο αριθμός. Λύση Α ν χ είναι ο ζητούμενος αριθμός, τότε από τα δεδομένα του προβλήματος θα ικανοποιεί την εξίσωση χ2-10χ=75<=>χ2- 10χ-75=0<=>χ=15 ή χ=-5.

Επειδή ο ζητούμενος αριθμός είναι θετικός, η μοναδική λύση του προβλήματος είναι ο αριθμός 15.

ΘΕΜΑ 2° Αν οι αριθμοί μ και v είναι θετικοί ακέραιοι και ισχύει ότι 4μ-2 + 4v+2 � γ+v+Ι,

να αποδείξετε ότι ο ακέραιος Α= 2μ + 2v είναι πολλαπλάσιο του 34.

Λύση. Από την υπόθεση έχουμε:

(22yι-2 + (22)v+2

-2·γ+v �Ο=> (γ-2)2 + (Υ+2)2

-2·2Ji+v �0=> (γ-2_2v+2)2 �Ο

από την οποία προκύπτει ότι 2μ-2 -2v+2 =Ο=> γ-v-4 = 1 => μ- v- 4 =Ο.

Επομένως έχουμε Α = 2μ + 2v = 2v+4 + 2v = 2v · ( 24 + I ) = 17 · 2v = 34 · 2v-ι,

που είναι πολλαπλάσιο του 34, αφού ο v είναι θετικός ακέραιος.

ΘΕΜΑ 3° Δίνεται τρίγωνο ΑΒΓ και έστω ΑΔ ύψος του. (α) Αν υπάρχουν σημεία Ε και Ζ των πλευρών ΑΒ και ΑΓ, αντίστοιχα, τέτοια ώστε να ισχύουν ΔΕ = ΔΖ και ΜΕ = ΜΖ , να αποδείξετε ότι το τρίγωνο ΑΒΓ είναι ισοσκελές. (β) Αν υπάρχουν σημεία Ε καιΖ στις προεκτάσεις των πλευρών ΑΒ και ΑΓ προς το μέρος του Α, αντίστοιχα, τέτοια ώστε να ισχύουν ΔΕ = ΔΖ και ΜΕ = ΜΖ , να αποδείξετε ότι το τρίγωνο ΑΒΓ είναι ισοσκελές. Λύση (α) Τα τρίγωνα ΑΔΕ και ΑΔΖ έχουν δύο πλευρές τους ίσες μία προς μία ( ΑΔ = ΑΔ, ΔΕ =ΔΖ)

και τις περιεχόμενες γωνίες των ίσων πλευρών ίσες, ΜΕ = ΜΖ . Άρα τα τρίγωνα είναι ίσα, οπότε θα έχουν και ΔΑΕ = ΔΑz , δηλαδή η ΑΔ είναι διχοτόμος της γωνίας Α του τριγώνου ΑΒΓ.

ΕΥΚΛΕΙΔΗΣ Β ' 74 τ.2/9

Page 12: Ευκλειδης Β 74

--------- Μαθηματικοί Διαγωνισμοί - Μαθη ματικές Ολυμπιάδες --------­L τη οϊ>νί:χιια συγκρίνουμε τα τρίγωνα ΑΔΒ και ΑΔΓ, τα οποία είναι ορθογι.Ονια μι; ο Λ Λ

:\:\Β ΛΛΙ · '-' 90" και έχουν την πλευρά ΑΔ κοινή και τις οξείες γωνίες ΔΑΒ και ΔΑΓ ίσες.

Λι)υ τα τρίγωνα ΑΔΒ και ΑΔΓ είναι ίσα , οπότε θα έχουν και ΑΒ = ΑΓ, δηλαδή το τρίγωνο

Λ ΙΗ ιίναι ισοσκελές

/ / / Β�-----

/ / / '. : \ .... ψ Δ

Σχήμα 5

Ε /� / :

/ / /

(β) Ομοίως όπως στο ερώτημα (α) τα τρίγωνα ΑΔΕ και ΑΔΖ είναι ίσα, οπότε θα έχουν

� � � �

ΔΑΕ=ΔΑΖ. Ι·.π::ιιiι'ι οι γωνίις ΓΆΕ και ΒΑΖ είναι ίσες ως κατά κορυφή, έπεται ότι:

I'< Λ Λ Α Λ Λ ΔΑΕ- Γ ΑΕ= ΔΑΖ- ΒΑΖ=> ΔΑΓ = ΔΑΒ,

ωυη<: ι, ω σ rην πιι)ίπτωση αυτή προκύπτει ότι η ΑΔ είναι διχοτόμος της γωνία; Α του τριγάΝου ΛΙ�Ι. Lτη συνι�χεια προχωράμε όπως στο ερώτημα (α). I \'llλ/.ιι" τικ(χ, Οα μπορούσαμε να προχωρήσουμε ως εξής: ο Λ ΛΗι> τι ι,· ισ(>τητα των τριγώνων ΑΔΕ και ΑΔΖ προκύπτει και η ισότητα ΔΖΑ= ΔΕΑ, οπίηε ::ιΊιωΛιι ιφοι,·ίJπτει ότι τα τρίγωνα ΒΔΕ και ΔΓΖ είναι ίσα, οπότε θα είναι ΔΒ = ΔΓ, η ευθεία ΑΔ ::ί\'(/.ι μ::σοi\Ίί.Οι:τη της πλευράς ΒΓ. Άρα είναι ΑΒ = ΑΓ. I( ω στι� ιiι'>Ο πψιπηl>σεις μπορούμε να χρησιμοποιήσουμε το γνωστό θα;Jρημα της Γεωμετρίας, ΙΙι/.σ::ι τοι ι οποίου, αν σε ένα τρίγωνο ένα ύψος του είναι και διχοτόμος. τότε το τρίγωνο είναι ι σι >ΟΙ<: :λ:':..;.

ΘΕΜΑ 4" i\'1 ίυ 1\ι)t>ση Λ γι:μίζει (λειτουργώντας μόνη της) μία δεξαμεν1) σε τρεις ώρες. Μία δεί>τφη ΙΙι)ι.ιση I� γψίζι:ι (λειτουργ<))ντας μόνη της) την ίδια δεξαμεν1) σε τέσσερις όψες. Μία τρίτη τί:λος ΙΙι1ιιοιι I αδι:ιάζι:ι (λειτουργ<))ντας μόνη της) την ίδια δεξαμεν1) (όταν βέβαια είναι γεμάτη) σι; {;ξι ι:ψ::..;. Ί .νας αυτόματος μηχανισμός ανοίγει με τυχαία σειρά και τις τρεις βρίJσες με τον ι:ξ1)ς

ι11ιΊrrο: ανοίγει μία βρί>ση, μετά από δύο ώρες ανοίγει μία άλλη και τέλος μετά από μία <iψα ιι\'()ίγ::ι ι<αι την άλλη βρύση. Ένας άλλος μηχανισμός μετρά το χρόνο που χρειάζεται να γεμίσει

'I ,i::-.:ιψι:,·ι·ι και ξεκινά τη λειτουργία του μόλις πι:σει νερό μέσα στη δεξαμεν1). Ποια είναι εκείνη ιι rι::φιΊ μ:: την οποία αν ανοίξει τις βρύσες ο μηχανισμός, ο αριθμός των ωρών που Οα ;11::ιιι.σωι.>ν (για να γεμίσει η δεξαμενή) να είναι ακέραιος αριθμός; Ποιος είναι σε κάθε ;r: 11ίπτι·>πη αυτι'>ς ο ακέραιος αριθμός; Λί>ση ·ι π τι•> _, . . ο ιψιΟμός των ωρών που χρειάζονται για να γεμίσει η δεξαμενή. Τότε οι δυνατοί η1ι1Ποt μ:; τους οποίους μπορεί να ανοίξει τις βρύσες ο μηχανισμός (μαζί με τις αντίστοιχες ::-.:ι<ιι:ιο::ι� ποιι ()ημιουργούνται) είναι:

ΕΥΚΛΕ ΙΔΗΣ Β ' 74 τ.2/10

Page 13: Ευκλειδης Β 74

-------- Μαθη ματικοί Διαγωνισμοί - Μαθηματικές Ολυμπιάδες -------­

(I) Λ-R-Γ

(2) Β-Α-1'

(3) Λ-Γ-Β

(4) Β-Γ-Α

(5) Γ-Β-Α

(6) Γ-Α-Β

χ χ-2 χ-3 12 -+-----=1 <;:::>5x=l2+6-6<;:::>x=-3 4 6 5 χ χ-2 χ-3 14 -+-----=1 <;:::>5χ=12+8-6<;:::>χ=-4 3 6 5 χ χ-2 χ-3 17 ----+--=1 <;:::>5χ=12+9-4<;:::>χ=-3 6 4 5 χ χ-2 χ-3 ----+--=1 <;:::>5χ=12+12-4<;:::>χ=4 4 6 3 χ

+χ-1

=1 <;:::>5x=l2+4<;:::>x=16

4 3 6 5 χ

+ χ

-1

-� = 1 <;::::> 5χ = 12 + 3 <;::::>χ= 3 3 4 6

Ί:νας φι)πος ανοίγματος είναι Β-Γ-Α με αντίστοιχη διάρκεια χ= 4 ώρες (περίπτωση (4)). 'Ενας δεί>τερος τρόπος ανοίγματος είναι Γ-Α-Β με αντίστοιχη διάρκεια χ= 3 ώρες (περίπτωση

(6)).

Στη πιφίπτωση (4) (που ανοίγει πρώτα η βρύση Β), ο χρόνος αρχίζει να μετράει με το άνοιγμα

της ΙΙι1ίισης Β. Λ ν λοιπόν υποθέσουμε ότι ο απαιτούμενος χρόνος για να γεμίσει η δεξαμενή είναι χ ώρες, τότε

η f\ι1ίιση Β θα έχει γεμίσει τα � της δεξαμενής. Στη συνέχεια ανοίγει η βρύση Γ η οποία θα 4

λι:ιτοιφγι)σα χ-- 2 cJ)ρες και θα αδειάσει τα χ-2

της δεξαμενής. Τέλος θα ανοίξει η βρύση Α η 6

οιωία Ου λι:ιτουργήσει χ-3 ώρες και θα γεμίσει τα χ-3

της δεξαμενής. Με αυτό τον τρόπο 3

Πf)UJ-.:ι·ιπτι:ι η ι.:ξίσωση ( 4). l.τη πψίπτωση (6) (που ανοίγει πρώτα η βρύση Γ), ο χρόνος αρχίζει να μετράει με το άνοιγμα

τη� 1\ιη'ισης Λ (διότι ο μηχανισμός χρονομέτρησης αρχίζει μόλις πέσει νερό στη δεξαμενή). Λ ν λοιπίιν υποΟ{;σουμε ότι ο απαιτούμενος χρόνος για να γεμίσει η δεξαμενή είναι χ ώρες, τότε

η 1\ι)ίιση Λ θα {:χει γεμίσει τα � της δεξαμενής. Στη συνέχεια ανοίγει η βρύση Β η οποία θα 3

) . I . θ

' x-l δ ξ

· -..ι:ιτοιφγησει χ- ωρες και α γεμισει τα -- της ε αμενης.

4

I ι':Jως η βρίιση Γ θα λειτουργήσει χ ώρες, και θα αδειάσει τα � της δεξαμενής. Με αυτό τον

φιιιτο προ κ ίιπτει η εξίσωση (6). Λ νι'ιλογα εξηγούνται και οι υπόλοιπες περιπτώσεις.

Β' ΛΥΚΕΙΟΥ Θ Ε ΜΑ Ι"

.'\ , . ο. jΊ ι:ί ναι θετικοί πραγματικοί αριθμοί, να αποδείξετε ότι: 4.J;;jj s (.!. + .!.J. α+ β . α+β α β 2

Λίιση

Τχσιψε r::D α+β ν uμ < ---

2 '

ΠρCι.yματι, αρκεί να δείξουμε ότι: Ο s ( .J;; -.fjj)2, που ισχύει.

ΕΥΚΛΕΙΔΗΣ Β ' 74 τ.2/1 1

( 1)

Page 14: Ευκλειδης Β 74

--------- Μαθηματικοί Διαγωνισμοί - Μαθηματικές Ολυμπιάδες ---------

Επίσης έχουμε: 4 1 1 -- <- + ­α + β-α β' (2)

Πράγματι γι ' αυτό αρκεί να δείξουμε ότι: 4 1 1 , 4 α + β , 4 β ( β)2 , 0 ( β)2 , -- �- + -, η -- � -- , η α � α + , η � α- , που ισχυει. α + β α β α + β αβ

Με πολλαπλασιασμό κατά μέλη των δύο ανισοτήτων ( 1 ) και (2) λαμβάνουμε τη ζητούμενη ανισότητα

Θ Ε ΜΑ 2°.

4Γαβ � (_!_ + _!_J. α + β . α + β α β 2

Δίνεται οξυγώνιο τρίγωνο Α8Γ , εγγεγραμμένο σε κύκλο C(O,R) . Αν Α1, 81, Γ1 είναι τα μέσα των πλευρών του 8Γ, ΑΓ, Α8 αντίστοιχα και Α2, 82, Γ2 είναι τα μέσα των ΟΑ, Ο8, 0Γ αντίστοιχα, να αποδείξετε ότι το εξάγωνο Α281Γ2Α182Γ1 έχει τις πλευρές του ίσες και ότι οι διαγώνιές του Α1Α2 , 8182 και Γ1Γ2 περνάνε από το ίδιο σημείο.

ΛίJση Εφόσον Ο είναι το κέντρο του περιγεγραμμένου στο τρίγωνο κύκλου, θα ισχύει: ΟΑ = 08 = ΟΓ = R .

I I

I ι

I I

/ I

I

/

" " " "'

Α "' "'

-------- -

,' , ' I --�· _ _,_,- " ,. : ι' ......... , ' '

' ' ------�--, ""_. I ,,''Κ\ .,. ............ , , \ "; ...... I ,, ' , • \ • / -._..., , ······ ο ' ���·············· \ ,' \ : \ ' ,,

Σχήμα 6

..... ..... ..... ' ' ' ' ' ' ' ' ' \ ' '

\

I

\ \ \ I

I Γ I

Το ευθύγραμμο τμήμα Α2Β1 συνδέει τα μέσα των πλευρών του τριγώνου ΟΑΓ, άρα:

Α28ι = ο; = � (1). Το ευθύγραμμο τμήμα Α182 συνδέει τα μέσα των πλευρών του τριγώνου ΟΒΓ , άρα:

Αι82 = ο; = � (2).

Με όμοιο τρόπο αποδεικνύουμε ότι όλες οι πλευρές του πολυγώνου είναι ίσες με R

. 2

Χρησιμοποιώντας τις σχέσεις (1) και (2) συμπεραίνουμε ότι το τετράπλευρο Α181Α282 είναι

ΕΥΚΛΕΙΔΗΣ Β' 74 τ.2/12

Page 15: Ευκλειδης Β 74

-------- Μαθηματικοί Διαγωνισμοί - Μαθηματικές Ολυμπιάδες -------­παραλληλόγραμμο, οπότε οι διαγώνιές του θα διχοτομούνται στο σημείο Κ . Με όμοιο τρόπο συμπεραίνουμε ότι το τετράπλευρο Α1Γ2Α2Γ1 είναι παραλληλόγραμμο, οπότε και σε αυτή τη περίπτωση οι διαγώνιες θα διχοτομούνται στο σημείο Κ .

Θ Ε ΜΑ 3"

Αν για τους πραγματικούς αριθμούς x, y με χ 2 2009 και y 2 -2009 ισχύει ότι :

�χ - 2009 + � y + 2009 = χ + Υ + 1 , 2

να βρεθεί η τιμή της παράστασης

Λύση Οι άρρητες παραστάσεις ορίζονται γιατί δίνεται ότι : χ 2 2009 και y 2 -2009 . Αν θέσουμε �χ - 2009 = α και � y + 2009 = b , τότε λαμβάνουμε χ = α2 + 2009 και

y = b2 - 2009 , από τις οποίες προκύπτει η εξίσωση χ + y = α2 + b2 • Τότε από τη δεδομένη ισότητα έχουμε :

α2 + b2 , , 2 2 α+ b = + 1 => α- +b- - 2α - 2b + 2 = 0 => ( α - 1) + ( b - 1 ) = Ο => α - 1 = b - 1 = Ο => α =b = 1 ,

2 οπότε θα είναι χ = 20 1 0, y = -2008 και Α = 20 1 0 .

Θ Ε :\1 Α 4"

Να λυθεί το σύστη μα: { (χ + y)3 = z - 2x - y (y + z)3 = χ - 2y - z (z + x)3 = y - 2z - x

στο σύνολο των πραγματικών αριθμών. Λύση

(Σ)

Θέτουμε χ + _ι · = α , y + z = β και z + χ = r , οπότε το δοσμένο σύστημα γίνεται: !α3 + 2α = β !α( α2 + 2) = β β3 + 2 β = r � β( β2 + 2 J = r r3 + 2r = α r( r2 + 2 J = α

Από το τελευταίο σύστημα συμπεραίνουμε ότι έχει τη προφανή λύση : α = β = r = Ο .

Θα αποδείξουμε ότι το σύστημα δεν έχει άλλη λύση . Αν αβr ::f:. Ο τότε πολλαπλασιάζοντας τις σχέσεις έχουμε:

αβr (α 2 + 2) (β2 + 2J (r2 + 2) = αβr => (α2 + 2)(β2 + 2)(γ2 + 2) = 1 . Η τελευταία ισότητα δεν είναι δυνατό να ισχύει , οπότε καταλήγουμε σε άτοπο. Αν υποθέσουμε ότι α = Ο τότε θα ισχύει: β = r = Ο . Α ν υποθέσουμε ότι β = Ο τότε θα ισχύει: α = r = Ο . Α ν υποθέσουμε ότι r = 0 τότε θα ισχύει: α = β = 0 . Αποδείξαμε λοιπόν ότι το σύστημα δεν έχει άλλη λύση εκτός από την α = β = r = Ο . Άρα το αρχικό σύστημα γίνεται: {χ + y = O

y + z = O <=> x = y = z = O . z + x = O

ΕΥΚΛΕΙΔΗΣ Β ' 74 τ.2/13

Page 16: Ευκλειδης Β 74

-------- Μαθηματικοί Διαγωνισμοί - Μαθηματικές Ολυμπιάδες --------

Γ ' ΛΥΚ Ε Ι Ο Υ Θ Ε ΜΑ 1° Να αποδείξετε ότι δεν υπάρχουν θετικοί ακέραιοι x, y που να επαληθεύουν την εξίσωση

2χ2 + 3χ (χ - 2) + 1 1χ - 1 Ο y = 20 1 5 . Λύση Η δεδομένη εξίσωση είναι ισοδύναμη με την χ (χ + 1 ) - 2 y = 403 . ( I ) Επειδή για όλους τους θετικούς ακέραιους χ, y οι αριθμοί χ (χ + I ) και 2 y είναι άρτιοι θι:τι κο ί ακέραιοι και η διαφορά τους χ (χ + Ι ) - 2 y θα είναι άρτιος θετικός ακέραιος, οπότε δεν r. ίνα ι δυνατόν να ισούται με 403 .

Θ Ε ΜΑ 2° Για τη συνάρτηση f : IR. � IR. ισχύει ότι:

f (x - / (y)) - f (y - / (x) ) = 2/ ( .f (x) - / (y)) , για κάθε x, y ε iR . Να aποδείξτε ότι .f(x - .f(x)) = Ο ,για κάθε χ ε IR. . Λύση Θέτουμε στη δοσμένη συναρτησιακή σχέση όπου y το χ και παίρνουμε:

f ( χ - f ( χ ) ) - f ( χ - f ( χ ) ) = 2f ( f ( χ ) - t' ( χ ) ) , οπότε θα είναι .f(O) = Ο . Θέτουμε στη δοσμένη συναρτησιακή σχέση όπου χ το Ο και παίρνουμε:

.f(O - .f(y )) - .f(y - .f(O )) = 2 .f(.f(O) - .f(y )) και χρησιμοποιώντας την ισότητα .f (Ο) = Ο , καταλήγουμε:

.!(- .f(y )) - .f(y) = 2.f(- .f(y )) <=> .!(- .f(y )) = -.f(y ) . Θέτουμε (στη τελευταία ισότητα) όπου y το χ και έχουμε τη σχέση:

.f ( -.f (χ) ) = -.f (χ) . Θέτουμε στη δοσμένη συναρτησιακή σχέση όπου y το Ο και παίρνουμε:

.f(x - .f(O )) - .f(O - .f(x )) = 2.f(.f(x ) - .f(O )) και χρησιμοποιώντας την ισότητα .f (Ο) = Ο , καταλήγουμε:

Ι (χ) - .f ( -.f (χ) ) = 2/ (Ι (χ )) . Από τις σχέσεις ( 1) και ( 2) έχουμε: .f(.f(x )) = .f(x ) , για κάθε χ ε IR. .

( 1 )

( 2 )

Θέτουμε τέλος στη δοσμένη συναρτησιακή σχέση όπου y το .f( χ) και χρησιμοποιιίJντας τ η προηγούμενη ισότητα έχουμε .f(x - .f(x )) = Ο , για κάθε χ ε IR. .

Θ Ε ΜΑ 3° Δίνονται τρεις θετικοί ακέραιοι αριθμοί της μορφής α 000 · · · 000 α , όπου α είναι Οι:τι κί)ς '-γ--------1

2ν-ψιιφία μονοψήφιος ακέραιος και μεταξύ του πρώτου και του τελευταίου ψηφί.ου του α ι1 ι Ο μ οίι α ΟΟ · · · ΟΟ α , μεσολαβούν 2v το πλήθος μηδενικά. Να αποδείξετε ότι: "ή ένας από αυτοίJς Οα διαιρείται με το 33 ιΊ το άθροισμα κάποιων από αυτούς θα διαιρείται με το 33 " .

ΛίJση Πρώτα θα αποδείξουμε ότι κάθε αριθμός της μορφής α ΟΟΟ · · · ΟΟΟ α διαιρείται μι: το / / . '--γ----/

2 ιι-ψηφία

Page 17: Ευκλειδης Β 74

--------- Μαθηματικοί Διαγωνισμοί - Μαθηματικές Ολυμπιάδες --------­Πράγματι, κάθε αριθμός της παραπάνω μορφής γράφεται;

α ΟΟ · · · ΟΟ α = α · 1 011 + 0 · 1 01

+ · · · + 0 · 1 0 2v + α . J ο Λ·+ Ι = = α + α · 1 0 2 ιι+ Ι = α(! + Ι 02v+ ι ) =

= α( l + I 0 ) ( / 0 2v - 1 0 2 ιι -- / + · · · + l) = l lα · Ι( .

'Ε στω τιορα α1 , α2

, α3 τρεις οποιοιδ1Ίποτε θετικοί ακέραιοι αριθμοί . της μ(ψφιΊς α ΟΟΟ · · · IJ IHJ ιι . ' --·-···· - - - - , _ -

"' ι · ιμηι - ι ./ ι ι Θα αποδείξουμε ότι: "ή ένας από αυτούς θα διαιρείται με το 3 ll το άΟρο ισμα κ<'ιπο ι ων ι ι π 1 ι

αυτσί)ς Ο α διαιρείται με το 3 " . ( I ) Λ ν Κ<iποιος από τους αριθμούς α 1 , α 2 , α 1 διαιρείται με το 3 , τότε Πf1Οψαν<ίις Ο υ ι πχι'ι ι : ι ' 1 πρόταση . 'Εστω ότι το 3 δεν διαιρεί κανένα από τους αριθμούς α 1 , α

2, α3 .

Τ<'ηε υπάρχουν οι παρακάτω δυνατές περιπτώσεις:

I ) Λ ν όλοι οι αριΟμοί είναι της μορφ1Ίς 3/( + I , τότε προφανώς α 1 + α 2 + α 1 = = 3ηι 2) Αν όλοι οι αριθμοί είναι της μορφ1Ίς 3/( + 2 , τότε προφανώς α 1 + α2 + α3 = 3n Σε όλες τις άλλες περιπτό)σεις ένας τουλάχιστον αριθμός θα είναι της μορφι1ς 3Α· + I Κ'(Ι Ι :':νι ι<

τουλάχιστον της μορφ1Ίς 3k + 2 , οπότε το άθροισμα αυτών των δύο αριθμών Οα είναι πι)οψυ\•ι : ι-;

πολλαπλάσιο του τρία .

Επειδ ιΊ καθένας από τους αριθμούς α 1 , α2 , α3 της μορφής α ΟΟ · · · ΟΟ α δ ιαιρε ίτα ι μ ι : το ι ι , ι':πετα ι ότι και το 6θροισμα οσωνδήποτε από αυτούς θα διαιρείται με το I / . ΛαμΒ<'Νοντας υπόψιν τις προηγούμενες προτάσεις, καταλήγουμε στο ζητούμενο .

Δίνι.:ται τρίγωνο ΑΒ Γ , εγγεγραμμένο σε κύκλο C(O, R) και έστω Α , , Β , , Ι Ί τα μ ι :συ τ ι • ι \ '

πλευρών του Β Γ, ΑΓ, ΑΒ αντίστοιχα. Θεωρούμε τους κύκλους C, ( A , , �-) , ('_, ( Β , , -i- ) "· ι ι ι

('\ ( Γ , , � ) . Αποδείξτε ότι ο ι κύκλοι C, , C2 , C3 περνάνε από το ίδιο σημείο ( έστω Ν ) και ι'η ι τ ι ι

δείιτερα κοινά ση μεία τους είναι τα μι�σα Α2 , 8 2 , Γ2 των ΟΑ, ΟΒ, ΟΓ αντίστοιχα . Στη π ι ιvι':z: : ι ι ι

να aποδείξτε ότι ο ι Α 1 Α2 , Β , Β 2 , Γ 1 Γ2 και Ο Ν περνάνε από το ίδιο σημείο .

Το τρίγωνο Α , Β , Γ, είναι όμοιο με το τρίγωνο ΑΒΓ . Ο λόγος ομοιότητας των δίιο φ ι γι ί ι \ ' ι • ι ν

1 είναι λ = - , οπότε ο περιγεγραμμένος κύκλος του τριγώνου Α , 8 1 Γ, 2

θ ' ' ({ α εχει ακτινα -

2

Οι κίικλοι τώρα που έχουν κέντρα τις κορυφές του τριγώνου Α β , Γ, και ακτίνα .!� Οα π : : ι )νι'ι \ · : : 2

από το περίκεντρο Ν του τριγώνου Α , Β , Γ, . (Το σημείο Ν είναι το κέντρο του κι'ικλο ι ι το ι ι

Ειι l c Γ του τριγώνου ΑΒΓ ) Λ ν Λ 2 , 82 , Γ2 είναι τα μέσα των ΟΑ, 08, ΟΓ αντίστοιχα, τότε :

R Α , Β2 = Α , Γ2 = Β 1 Α 2 = Β , Γ2 = Γ1 Α2 = Γ, Β2 = 2

(Τα παραπάνω τμήματα Α , 82 , Α , Γ2 , Β , Α 2 , Β , Γ2 , Γ1 Α 2 0 Γ, 82 είναι διάμεσοι προς την ι ιποτι : ίνιΗ ι σ ι ι

ΕΥΚΛΕΙΔΗΣ Β ' 74 τ.2/ 1 5

Page 18: Ευκλειδης Β 74

--------- Μαθηματικοί Διαγωνισμοί - Μαθηματικές Ολυμπιάδες --------­των ορθογωνίων τριγώνων ΟΑ1 Β , ΟΑ1 Γ , ΟΒ1 Α , ΟΒ1 Γ , ΟΓ1 Α και ΟΓ1 Β . )

Άρα τα δεύτερα κοινά σημεία των κύκλων C 1 (A 1 ,-�) , C2 (B 1 , R ) και C3 (Γ1 , �) είναι τα σημεία 2 2 2

Τα τετράπλευρα Γ1 ΝΒ1Α2 και ΟΒ2Α 1 Γ2 είναι ρόμβοι με πλευρές μήκους � και οι πλευρές του

ενός τετραπλεύρου, είναι παράλληλες με τις πλευρές του άλλου ( Α2 Β 1 = 11 Β2Α 1 ,

ΓΙ Α2 = 11 Α Ι Γ2 , . . . . ) .

Α

·----·----

· · · · · . . . . .. .. .. .-·.·:: · · ·

82 .�:=���,-; · · · · ·

. · · · · · · · · · · · . . . . . .

Σχήμα 7

Από τα παραπάνω προκύπτει ότι:

Το τετράπλευρο Α20Α 1 Ν είναι παραλληλόγραμμο οπότε οι διαγώνιές του θα διχοτομούνται.

Δηλαδή η Α 1Α2 περνά από το μέσο Κ του ΟΝ που είναι μέσο και του Α 1 Α2 •

Το τετράπλευρο Γ1 Α2Γ2Α1 είναι παραλληλόγραμμο οπότε οι διαγώνιές του θα διχοτομούνται.

Δηλαδή η Γ1 Γ2 περνά από το μέσο Κ του Α 1 Α2 που είναι μέσο και του Γ1 Γ2 •

Τέλος το τετράπλευρο Β1Γ1Β2Γ2 είναι παραλληλόγραμμο οπότε οι διαγώνιές του θα

διχοτομούνται. Δηλαδή η Β1Β2 και περνά από το μέσο Κ του Γ1 Γ2 που είναι μέσο και του

Β Ι Β2 .

ΕΥΚΛΕΙΔΗΣ Β' 74 τ.2/16

Page 19: Ευκλειδης Β 74

-------- Μαθηματικοί Διαγωνισμοί - Μαθηματικές Ολυμπιάδες --------

1 2η ΜΕΣΟΓΕΙΑΚΉ ΜΑΘΗΜΑΤΙΚΉ ΟΛΥΜΠΙΑΔΑ στη μν1Ί μη Peter Ο' Hal l oran

2 5 Απριλίου 2009 ΕΝΔΕ Ι ΚΤΙ ΚΕΣ Λ ΥΣΕΙΣ

Π ρόβλημα ι . Να προσδιορίσετε όλους τους ακέραιους n � 1 , για τους οποίους υπάρχουν n πραγματικοί

αριθμοί Χι , χ2 , . . . , Χ11 στο κλειστό διάστημα [ -4, 2] , έτσι ώστε να ικανοποιούνται οι επόμενες

τρεις συνθήκες : • Το άθροισμα των αριθμών αυτών είναι τουλάχιστον n . • Το άθροισμα των τετραγώνων των αριθμών αυτών είναι το πολύ 4n . • Το άθροισμα των τέταρτων δυνάμεων των αριθμών αυτών είναι τουλάχιστον 34n .

Λύση Επειδή τα δεδομένα του προβλήματος αφορούν n πραγματικούς αριθμούς Χι , χ2 , • • • , χ" στο

κλειστό διάστημα [ -4, 2] , θεωρούμε το πολυώνυμο Ρ ( χ) = (χ + 4 ) (χ - 2 ) (χ - 1 )2 , το οποίο στο

διάστημα [-4, 2 ] ικανοποιεί τη σχέση Ρ(χ) = (χ + 4) (χ - 2) (χ - 1 )2 � 0 . ( 1)

Με πρόσθεση κατά μέλη των ανισοτήτων που προκύπτουν από την ( 1 ) , για χ = Χι , χ2 , . . . , Χ11 , και λαμβάνοντας υπόψη τις συνθήκες του προβλήματος, λαμβάνουμε:

η 11 11 Ο � Ρ ( χι ) + . . . + Ρ ( χ" ) = Σχ: - 1 1Σχi2 + 1 8Σχi - 8n � 34n - 1 1 · 4n + 1 8n - 8n = Ο. (2) i=ι i= ι i= ι

Επομένως , αφού Ρ( χί ) � Ο, για κάθε ί = 1, 2, . . . , n , από τη (2) προκύπτει:

P (xi ) = O, για κάθε i = 1 , 2, . . . , n , οπότε θα είναι χί ε { -4, 1 , 2} , για κάθε ί = 1 , 2, . . . , n . Ας υποθέσουμε ότι από τους ακέραιους

χι , χ2 , • • • , χ" , α είναι ίσοι με -4, b είναι ίσοι με 1 και c είναι ίσοι με 2, τότε θα έχουμε

α + b + c = n και από τα δεδομένα του προβλήματος προκύπτουν οι ανισότητες { 1 6

-α4::: :c2� : (: :b

b :cc ) } δηλαδή { ; : �: } ·

256α + b + 1 6c � 34 ( α + b + c) 222α � 33b + 1 8c Με πολλαπλασιασμό των δύο μελών της πρώτης με 1 8 , της δεύτερης με 33 και πρόσθεση

κατά μέλη των ανισοτήτων που προκύπτουν, λαμβάνουμε την ανισότητα 33b + 1 8c � 1 32α + 90α = 222α ,

η οποία σε συνδυασμό με την ανισότητα 222α � 3 3b + 1 8c δίνει: 3 3b + 1 8c = 222α ,

η οποία μπορεί να ισχύει, αν, και μόνον αν, b = 4α και c = 5α , δηλαδή όταν είναι α + b + c = Ι Οα ή Ι Οα = n .

Επομένως, οι αριθμοί που ζητάμε υπάρχουν, αν, και μόνον αν, το πλήθος τους είναι πολλαπλάσιο του 1 Ο. Για n = 1 Om, όπου m θετικός ακέραιος, μία δυνατή λύση προκύπτει, αν

πάρουμε m φορές τον αριθμό -4, 4m φορές τον αριθμό 1 και 5m φορές τον αριθμό 2 .

Π ρόβλημα 2 . Έστω τρίγωνο ABC με Α "Φ 90° και Α "Φ 1 3 5° . Τα σημεία D και Ε είναι εξωτερικά του

ΕΥΚΛΕΙΔΗΣ Β' 74 τ.2/17

Page 20: Ευκλειδης Β 74

--------- Μαθη ματικοί Διαγωνισμοί - Μαθηματικές Ολυμπιι'ιδες --------­τριγώνου ABC και τέτοια ώστε τα τρίγωνα DAB και JΞA C' να είναι ορΟογe.i>νια και ισοσκελ1Ί με ορθές γωνίες στις κορυφές D και Ε , αντίστοιχα. Έστω επίσης F = ΒΕ n CD και Μ , Ν τα

μέσα των ευθυγράμμων τμημάτων BC και DE , αντίστοιχα. Να απο<>είξετε ότι, αν οποιαδήποτε τρία από τα τέσσερα σημεία Α, F, Μ, Ν είναι συγγραμμικά, τ(>τε και τα τ i:σσ::ρα σημεία είναι συγγραμμικά.

Λύση Θα χρησιμοποιήσουμε δύο αρκετά γνωστά βοηθητικά λήμματα της επιπεδομετρίας :

Λή μμα 1 . Με τις υποθέσεις του προβλήματος, ανεξάρτητα από τη Οέση των σημείων

A, F, Μ, Ν το τρίγωνο DME είναι ορθογώνιο και ισοσκελές με MD = ΜΕ και DME = 90° ,

δηλαδή η ευθεία ΜΝ είναι μεσοκάθετη του ευθύγραμμου τμήματος DE .

Απόδr.ιξη Θεωρσί>με το συμμετρικό G του σημείου Ε ως προς το μέσο Μ της BC' ( hΕς σχ1Ί μα I ) και

αποδεικνύουμε ότι το τρίγωνο EDG είναι ορθογώνιο και ισοσκελές με [)Ε = f)G και EDG = 90n .

Α

D

Λή μμα 2. Με τις υποθέσεις του προβλήματος, ανεξάρτητα απ6 τη Ο:':ση των σημ :� ίων Α , F, Μ, Ν , η ευθεία A F είναι κάθετη προς την ευθεία f)E .

Απόδειξη Ιl pοκύππ:t απ6 τη γνωστή θεωρία για το σύστημα Vcctc n . Στη συνέχεια θα διακρίνουμε περιπτώσεις θεωρώντας κάΟε φορά ό τι τρ ία απί> τα τ{;σσερα

σημι:: iα Α, F, Μ, Ν είναι συνευθειακά και σε κάθε περίπτωση Οα αποδείξουμε 6τι προκύπτει η

ισότητα Α Β = A C , από την οποία έπεται ότι και τα τέσσερα ση μεία Α, F, Μ, Ν είναι

συνευθειακά. Έχουμε:

ΕΥΚΛΕΙΔΗΣ Β ' 74 τ.2/1 8

Page 21: Ευκλειδης Β 74

--------- Μαθηματικοί Διαγωνισμοί - Μαθηματικές Ολυμπιάδες ---------

Σχήμα 2 (α) Τα σημεία M, N, F ;:iνω σΙJγγραμμιιιά. Σύμφωνα με τα προηγούμενα, η ευθεία ΜΝ που περιέχει και το σημείο F είναι

μεσοκάθετη του ευθύγραμμου τμήματος DE , αλλά και ευθεία F Α είναι κάθετη προς την ευθεία DE . Επομένως και τα τέσσερα σημεία Α, F, Μ, Ν είναι συνευθειακά.

Μ ία δεύτερη απόδειξη για την περίπτωση αυτή είναι η εξής: Οι ευθείες ΒΕ, CD και ΜΝ ορίζουν πάνω στις ευθείες BC και DE τμήματα ανάλογα,

οπότε, σύμφωνα με το δεύτερο αντίστροφο του θεωρήματος δέσμης ευθειών θα είναι DE 1 1 BC . Έτσι έχουμε τις ισότητες

FB FM FC - = -- = -FE FN FD ' Α Α

από τις οποίες προκύπτει ότι τα τρίγωνα FBD και FCD είναι όμοια, οπότε έχουν FDB = FEC . Αυτό δίνει ότι το τραπέζιο BCED είναι εγγράψιμο και συνεπώς ισοσκελές, δηλαδή είναι BD = CE , οπότε από τα ορθογώνια ισοσκελή τρίγωνα ABD και Α CE προκύπτει η ισότητα ΑΒ =' A C , από την οποία τελικά προκύπτει το ζητούμενο. Πράγματι, εύκολα προκύπτει η

tσότητα των τριγώνων BCD και BCE ( BC κοινή πλευρά, BD = CE και CBD = 45° + Β = 45° + C = BCE ). Άρα τα σημεία D, Ε ισαπέχουν από την ευθεία BC , οπότε

θα είναι DE 1 1 BC και το τραπέζιο BCED είναι ισοσκελές. Επειδή και το τρίγωνο ADE είναι ισοσκελές, έπεται ότι η ΑΜ είναι μεσοκάθετη της πλευράς BC , αλλά και της DE , οπότε θα περνάει από τα σημεία Ν και F.

(β) Τα σημεία Α, Μ, F είναι συγγραμμικά. Όμοια, σύμφωνα με τα λήμματα 1 και 2 , η ευθεία AFM είναι κάθετη προς την ευθεία DE ,

οπότε η ευθεία αυτή είναι ο φορέας του ύψους του ορθογώνιου και ισοσκελούς τριγώνου DΛ1Ε . Άρα αυτή περνάει από το μέσο Ν της DE.

(γ) Τα σημεία Α, Μ, Ν είναι συνευθειακά. Όμοια, σύμφωνα με τα λήμματα 1 και 2 , η ευθεία ΑΝΜ είναι κάθετη προς την ευθεία DE ,

αλλά και η FA είναι κάθετη προς την DE , οπότε πάλι τα τέσσερα σημεία A, F, M, N είναι συνευθειακά.

ΕΥΚΛΕΙΔΗΣ Β ' 74 τ.2/19

Page 22: Ευκλειδης Β 74

--------- Μαθηματικοί Διαγωνισμοί - Μαθηματικές Ολυμπιάδες ---------

(δ) Τα σημεία A, F, N είναι συvευθειακά. Όμοια, σύμφωνα με τα λήμματα 1 και 2, η ευθεία ANF είναι κάθετη προς την ευθεία DE ,

αλλά και η ευθεία ΝΜ είναι κάθετη προς την DE , οπότε πάλι τα τέσσερα σημεία Α, F, Μ, Ν είναι συνευθειακά.

Π ρόβλημα 3. Να εξετάσετε, αν οι αριθμοί 1 ,2 , . . . , 1 00 μπορούν να τοποθετηθούν στα κελιά C ( ί, J ) ενός

l Ο χ l Ο πίνακα, όπου Ι :::;; ί , j :::;; Ι Ο , έτσι ώστε να ικανοποιούνται οι επόμενες τρεις συνθήκες : • Σε κάθε γραμμή το άθροισμα των στοιχείων της είναι ίσο με S . • Σε κάθε στήλη το άθροισμα των στοιχείων της είναι S . • Για κάθε k = Ι , 2, . . . , Ι Ο τα στοιχεία των δέκα κελιών C ( ί, j ) με ί - j = k ( mod 1 Ο) έχουν

άθροισμα S .

Λύση Το πρόβλημα αφορά ένα μαγικό τετράγωνο με κοινό άθροισμα των δέκα γραμμών και

στηλών, αλλά και μία επιπλέον συνθήκη σταθερού αθροίσματος κατά μήκος των wrap­around διαγωνίων.

Ας υποθέσουμε ότι η ζητούμενη τοποθέτηση των αριθμών 1 ,2 , . . . , 1 00 είναι δυνατή . Επειδή το άθροισμα όλων των αριθμών είναι 50 · 1 Ο Ι = 5050 έπεται ότι το άθροισμα των στοιχείων κάθε γραμμής ή στήλης θα είναι 505 , δηλαδή περιττός αριθμός. Στη συνέχεια χωρίζουμε τα κελιά C ( ί, J ) σε τέσσερα υποσύνολα Α, Β, C και D, ως εξής:

Το σύνολο Α περιέχει τα κελιά με τα ί , j και τα δύο περιττά. Το σύνολο Β περιέχει τα κελιά με περιττό ί και άρτιο j . . Τ ο σύνολο C περιέχει τ α κελιά με άρτιο ί και περιττό j . Το σύνολο D περιέχει τα κελιά με άρτιο ί και άρτιο j . Συμβολίζουμε το άθροισμα όλων των στοιχείων των συνόλων Α, Β, C και D με SA ,

S11 , Sc, και S0 , αντίστοιχα. Τότε θα έχουμε: • Επειδή τα σύνολα Α και Β περιέχουν όλα τα κελιά C ( ί, j ) των περιττών γραμμών ,

θα είναι: ( I )

• Επειδή τα σύνολα Β και D περιέχουν όλα τα κελιά C ( ί , J ) των άρτιων στηλών, θα είναι:

(2) • Επειδή τα σύνολα Α και D περιέχουν όλα τα κελιά C ( ί, J ) με άρτιο ί - j , θα είναι

SA + Sυ = SS . (3 )

Με πρόσθεση κατά μέλη των ισοτήτων ( Ι ) , (2) και (3) προκύπτει η ισότητα:

(4)

Η ισότητα (4) όμως οδηγεί σε άτοπο, αφού το πρώτο μέλος της είναι άρτιος ακέραιος, ενώ το δεύτερο μέλος της είναι περιττός ακέραιος, ως γινόμενο περιττών.

ΕΥΚΛΕΙΔΗΣ Β ' 74 τ.2/20

Page 23: Ευκλειδης Β 74

-------- Μαθηματικοί Διαγωνισμοί - Μαθηματικές Ολυμπιάδες --------

Πρόβλημα 4. Έστω x, y, z θετικοί πραγματικοί αριθμοί. Να αποδείξετε ότι:

Λύση

xy yz zx χ y z ------'':--? -�? + 2 2 + 2 2 � -- + + --xy + x- + y yz + y + z zx + z + χ 2x + z 2y + x 2z + y

Αρκεί να δείξουμε ότι :

Χρησιμοποιώντας τον μετασχηματισμό

οπότε αbc = I , αρκεί να δείξουμε ότι:

χ Υ z - = α - = b - = c Υ

, z , χ ,

Ι Ι + Ι < -Ι- + _Ι_ + _Ι_ Ι + α + l_ +

I + b + l_ I + c + l - 2 + α 2 + b 2 + c .

α b c

Μετά από πράξεις και χρήση της σχέσης αbc = Ι , αρκεί να δείξουμε ότι:

3 ( α + b + c) + 3(αb + bc + cα) + ( αb + bc + cα)( α + b + c) < ( αb + bc + cαγ + (αb + bc + cα)( α + b + c) + (α + b + cγ -

< Ι2 + 4 (α + b + c) + ( αb + bc + cα) - 9 + 2(αb + bc + cα) + 4(α + b + c) ·

Αν θέσουμε α + b + c = S, αb + bc + cα = Ρ , αρκεί να δείξουμε ότι:

3S+3P+SP < I2+4S+P δ λ δ ' ' Ρ3 4S3 3P2S PS2 6P2 > 2 7S 2 7Ρ I5PS ( * ) Ρ2 +PS+S2 - 9+2P+4S ' η α η αρκει + + + + - + +

Εύκολα αποδεικνύουμε ότι S2 ?:. 3Ρ και επίσης S ?:. 3, Ρ ?:. 3 , από την ανισότητα αριθμητικού - γεωμετρικού μέσου και τη σχέση αbc = Ι . Επομένως, έχουμε:

Ρ3 = Ρ2 · Ρ ?:. 32 · Ρ = 9Ρ, 6Ρ2 = 6Ρ · Ρ ?:. 6 · 3Ρ = Ι 8Ρ,

οπότε με πρόσθεση κατά μέλη των παραπάνω ανισοτήτων προκύπτει η ανισότητα (*) .

Η ισότητα ισχύει, αν, και μόνον αν, S 2 = 3 Ρ .

Αλλά: S2 = 3P = 9 G α = b = c = I G X = y = z . ΕΥΚΛΕΙΔΗΣ Β ' 74 τ.2/2 1

Page 24: Ευκλειδης Β 74

ΗΟΜΟ MATHEMATICUS Η Homo Mathematicus είναι μια στήλη στο περιοδικό μας, με σκοπό την ανταλλαγή απόψεων και την ανάπτυξη προβληματισμού πάνω στα εξής θέματα: 1 ) Τι είναι τα Μαθηματικά, 2) Πρέπει ή όχι να διδάσκονται, 3) Ποιοι είναι οι κλάδοι των Μαθηματικών και ποιο το αντικείμενο του καθενός, 4) Ποιες είναι οι εφαρμογές τους, 5) Ποιες επιστήμες ll κλάδο ι επιστημών απαιτούν καλή γνώση των Μαθηματικών για ν α μπορέσει κάποιος ν α τους σπουδάσει.

Για του; συνι:ργ(�τι:ς τη; ση]λη;: παράκληση ! τα κείμενα της στήλης αυτής, ως προς το περιεχόμενό τους και ως προς το επίπεδό τους, θα πρέπει να είναι συμβιβαστά με τα ενδιαφέροντα και το επίπεδο κατανόησης από μέρους των παιδιών.

I . "τι είιιαι τα ΜαΟηματικά;

«ΕΡΩΤΗΜΑ 4. Πως υπεισέρχονται τα Μαθηματικά στις ά}λες επιστήμες;

ΑΠΆΝΤΗΣΗ. Ο F i n ste i n έχει πει: <<Σκοπός της επιστήμης είναι να συντονίσει τις εμπειρίες, ώστε να αποτελέσουν ένα λογικό σύστημα». Υπάρχει, όμως, λογικό σύστημα, που να μην είναι και μαθηματικό σύστημα; Φανερά, όχι. Π .χ. η «κοινή» λογική , με δύο τιμές «ναι» και «όχι» αποτελεί Άλγεβρα του Boo l e . Το ηλεκτρικό ψυγείο λειτουργεί σύμφωνα με μια «λογική» με τρεις τιμές (ιστορικά, η τελευταία αυτή λογική βρίσκεται στον Αριστοτέλη , που λέει ότι δεν μπορούμε να aποφανθούμε με ένα «ναι» ή «όχι» για τα γεγονότα του μέλλοντος · ενώ, η δίτιμη, που, συνήθως, αποκαλείται «αριστοτέλεια», προϋπήρχε στους aρχαίους Έλληνες. Με τα θέματα αυτά είχε ασχοληθεί και ο μεγάλος Ρουμάνος Μαθηματικός Μο'ίζίλ) . Εξ άλλου, μόλις θελήσουμε να οργανώσουμε λογικά εμπειρίες πιο σύνθετες, αναγκαία θα χρησιμοποιήσουμε Μαθηματικά (κάποια Μαθηματικά) . Έτσι π.χ. η μετάβαση από τη διαπίστωση «ως σήμερα, ο Ήλιος βγαίνει κάθε μέρα από την Ανατολή» στην υπόθεση «Ο Ήλιος θα βγει κι αύριο από την Ανατολή», μπορεί να μη χρειάστηκε Μαθηματικά · όμως, η μετάβαση από την υπόθεση αυτή στη γενικότερη «το πλανητικό σύνολο αποτελεί ηλιοκεντρικό σύστημα», χρειάστηκε Μαθηματικά, ακόμα κι αν στο αρχικό της «λαϊκό» φτάσιμο ( ορφικοί ύμνοι, . . . ) δεν χρειάστηκε χαρτί και μολύβι . . .

1 1 . 'Ά υτδ το ξf:ρατι:; ,

Τι είναι οι δρακόντειες καμπύλες;

1 1 1 . , οι συ vι:ργάτεc: τιιc: στιίλιιc: JΙΙ)άφου ν-ερωτοι)ν ,

επιμέλεια : Κερασαρίδης Γι{ιννης

ΕΡΩΤΗΜΑ 5 . Ποιος είναι ο συγκεκριμένος

τρόπος, με τον οποίο περιγράφουμε μαθηματικά

φυσικές καταστάσεις; ΑΠΆΝΤΗΣΗ. Ένα κατανοητό απ' όλους

παράδειγμα είναι η μετάβαση απ' την «εικόνα» του χώρου, που μας δίνουν οι αισθήσεις (όραση και αφή) στην Ευκλείδεια Γεωμετρία, σαν ιδεατιί

μαθηματική περιγραφή του . Θεμελιώδες επίτευγμα των αρχαίων Ελλήνων υπήρξε, εδώ, η αφαίρεση των διαστάσεων · δηλ. πως, από τα τρισδιάστατα υλικά αντικείμενα φτάσανε στα τρισδιάστατα νοερά, ύστερα στα δισδιάστατα και τα μονοδιάστατα νοερά, και, το δυσκολότερο, στο χωρίς διαστάσεις σημείο. Λέει, σχετικά, ο Ήρων ο Αλεξανδρεύς, ότι, στην αρχή , ο άνθρωπος διαχώρισε το γεωμετρικό από το φυσικό στερεό · και ύστερα, «δι ' αφαιρέσεως, κατήντησεν επί το σημείον \» (Μέσα σ' αυτές τις δύο γραμμές βρίσκεται η μια από δυο θεμελιωδέστερc:ς προϋποθέσεις για τη δημιουργία της επιστιί,ιι η.:; · η άλλη, ήταν η έννοια του αφηρημένου αριθμού · δηλ. , πως από τα «πέντε μήλα» πιiγαμε στον

αριθμό «πέντε» · μετάβαση , που ασφαλιος δεν έγινε δια μιας «στο σύνολο των φυσικών αριθμών» . . . )»

[πηγή : Σπύρου Ζερβού «τΙ ΜΠΟΡΟΥ� �Α ΜΑΣ ΠΟΥ Ν Ε ΤΑ ΜΑΘΗΜΑτΙΚΆ ΚΑΙ ΟΙ ΦΥΣ Ι ΚΕΣ Ε ΠΙΣΤΗΜΕΣ;», τρας διαλέξεις « I . Για τα θεμέλια της γνώσης στις Φυσικις Επιστήμες και στα Μαθηματικά». εκδ. ΚΑ ΡΑΒ Ι ΑΣ, Αθήνα 1 99 1 ]

Γιάννη ς Κερασαρίδης

[η απάντηση στο τέλος της στήλης]

Πρlότο Οιiμα: «Ποια τμιίματα των ΜαΟηματιιαvιι είναι χριίσιμα,· (του (i. H. HaN(Ι')

Ο φίλος της στήλης Δ . Κ. Ληδός (Ιωάννινα), μας έστειλε για δημοσίευση το παρακάτω κείμενο με την επισήμανση : «είναι κείμενο του μεγάλου μαθηματικού G Η. Hardy». Εμείς το ψάξαμε και διαπιστώσαμε πως ο φίλος μας, δανείστηκε το κείμενό του από την «ΑΠΟΛΟΓΙΑ ΕΝΟΣ ΜΑΘΗΜΑτΙΚΟΥ» του G.H. Hardy (εκδ. "Πανεπιστημιακές Εκδόσεις Κρήτης", μετάφραση-σχόλια: Δ. Καραγιαννάκης-Μ. Λάμπρου). Ζητάμε συγγνώμη από τον Δ. Κ. Ληδό για την καθυστέρηση της δημοσίευσης, επικαλούμενοι «φόρτο» ύλης. Σ ' αυτό το απόσπασμα, ο G.H. Hardy απαντά στο ερώτημα: Ποιο υλικό από τα Μαθηματικά είναι χρήσιμο;

ΕΥΚΛΕΙΔΗΣ Β ' 74 τ.2/22

Page 25: Ευκλειδης Β 74

------------- ΗΟ Μ Ο M A'ΓHEMAτiCUS

«Πρώτα απ ' όλα ο κίφιος όγκος των σχολικών \ιl α Ο η ματικιίιν, η Αρ ιθμητ ική , η στοιχειώδης Λλγεβρα , η στο ιχειώδης Ευκλείδεια Γεωμετρία, ο στο ιχαώδης Δ ιαφορ ικός και Ολοκληρωτικός Λογ ισμι'ις . J J ι1 ι:πι: ι να εξαιρέσουμε ορισμένα μέρη απ ' αυτά που δ ιδάσκονται σι «Ειδ ικούς», όπως η Γ/ ρο βολικ Ι) Γεωμετρία. Από τα Εφαρμοσμένα, χρι ιάζονται τα στοιχεία Μηχανικ1)ς ( ενιο ο 1 /λι:;κτ ρ ισμός , ίιπως διδάσ κεται στα σχολεία, πρi:πα να ταξ ινομηθε ί στη Φυσική ) .

Κατι'ιπ ιν, ί:να αρκετά μιγάλο μέρος των Μ αΟη ματ ικιiιν σι πανεπ ιστη μ ιακό επίπεδο είναι ι;π ί σης Χf> Ι1 σ ιμ ο , ικ ι; ίνο το μέρος τους που πραγματ ικιJ. αποτελι ί αν(χπτυξη των σχολικών

Μ αΟηματ ικ<:ιν με π ιο ολο κλη ρωμ{;νες τεχνικές, καΟ<ίJς και κ(ι.ποιο μέρος των εγγίJτερων προς τη Φυσι κ ΙΊ Ο ι : μάτω ν , 6πως ο 1- Ιλcκτρ ισμός και η Υδρο μηχανωΊ . Ι Ι ι1ι":πα να θυμόμασπ επίσης ότι ένα απ60ψα γνι!Jση.ιJV αποη.:λε ί πάντοτε προσόν, και ίηι ο πλi:ον πρακτ ικός από τους μαθη ματικούς μποριί να βριΟι: ί σε σομαι.}]Ί αδυναμία αν οι γνι!Jσεις του ιίναι οι ενη:λώς ελάχιστες που του χρι: ι α-,οντ α ι . Γι αυτό το λίJγο πρέπει να πιχισΟι':σουμ:; στα χρ 1Ίσιμ α κάτι λίγο από κάθε κλ<Ίδο . Λλλά το γι:ν ικό μας συμπέρασμα πρtπει να ι : ίναι πως χp1Ίσ ιμα είναι εκείνα τα ΜαΟηματικά που του ζη ηΊ ένας ανιiηερος μηχαν ικός Ιl ένας μ ι':φιος φυσ ικός. Και αυτ6 ε ίναι το ίδ ιο, σε γι;ν ικι':ς Υι1α μ μ ι;ς, μι; το να λi:με ότι αυτά τα Μ α 0 η μ ατ ι κ6 6εν έχουν ιδ ια ίτφο αισθητικό χ< ·φ ισμα . Αιrι) την Ευκλε ίδι: ια Γεωμετρία, για ιrαρά6ι: ιγ μ υ , χr) ]Ίσ ι μ ο ψ 1) μα τη ς ε ίναι το πληκτικό

ι)ιν χι1 : :ωζι'ψ ιι.στε το αξ ίωμα της παραλληλίας,

τ η Οασρ ία των αναλογ ιcί.JV 1) την κατασκευή του κανον ι κυίι π:�νταγι!Jνου.

ΊΞνα μάλλον αξιοπψίι;ργο συμπέρασμα ι':ρχι:ται στην επ ιφάνε ια ότι τα καθαρά Μ αΟη ματικ<'ι. ε ίναι συνολικά εμφανώς πιο χρ ι)σ ι μα απ ίηι τα εφαρμοσμ{;να. Ένας u.κραιφνΙΊ ς μαΟ η ματι κι'ις φα ίν::τα ι να υπcρέχει τόσο από την πρακτι κ ΙΊ όσο και από την

α ισΟ η τ ικ ι) πλcυρά. Γιατ ί αυτό που είναι χρήσιμο πάνω απ ' όλα c ίναι η τεχιJtκή, και η μαθηματική

:t ;: ι)τφο Ο/μα: Μια σπουδαία είδηση

τεχνική διδάσκεται κυρίως μέσα από τα καθαρά Μαθηματικά.

Ελπίζω να μη χρειάζεται ν' αναφέρω πως δεν προσπαθώ να δυσφημίσω τη ΜαΟηματικ1Ί Φυσική , ένα θαυμάσιο αντικcίμενο μcλέτης μι.; τρομακτικού ενδιαφέροντος προβλήματα, <'>που άνθρωποι με την πλέον εκλεπτυσμένη φαντασία την άφησαν να καλπάσει ανεξέλεγκτα. Αλλά δcν είναι η θέση ενός συνηθισμένου ανθρώπου των Εφαρμοσμένων Μαθηματικών κατά κάποιο τρόπο λίγο aξιολύπητη ; Αν θέλει να είναι χρ1Ίσιμος πρέπει να εργασθεί με τρόπο μονότονο, και δεν μπορεί να αφ1)σει ελεύθερη τη φαντασ ία του ακόμη και όταν επιθυμεί να αιθεροβατ ι) σcι . Τα σύμπαντα της «φαντασίας» είναι πολύ π ιο όμορφα από αυτό το ανόητα κατασκευασμένο «πραγματικό» σύμπαν. Τα περ ισσότερα απc'> τα προϊόντα της φαντασίας κάποιου στα εφαρμοσμένα Μαθηματικά πρέπει να απορριφθούν αμέσως μόλις δημιουργηθούν, για την ωμή αλλά ικανή αιτία ότι δεν αντιστοιχούν στα δεδομένα.

Το γενικό συμπέρασμα σίγουρα ε ίναι αρκετά φανερό. Αν, όπως συμφωνήσαμε προσωρινά, χρήσιμη γνώση είναι αυηΊ που κατά πάσα πιθανότητα, τώρα ή στο συγκρ ιτι κά κοντινό

μέλλον, συμβάλλει στις ανθριiιπ ινες υλικi;ς ανέσεις και άρα η καθαρή διανοητικ1Ί ικανοποίηση δεν μετράει, τότε ο μεγάλος όγκος των ανώτερων Μαθηματικών είναι άχρηστος. Η μοντέρνα Γεωμετρία και Άλγεβρα, η Θεωρία των Αριθμών, η Θεωρία των Συνόλων και των Συναρη)σεων, η Σχετικότητα, η Κβαντομηχανική , κανένα απ ' αυτά δεν ανταποκρίνεται στο κρ ιη)ριο αυτό καλύτερα από τα υπόλοιπα, και δεν υπάρχε ι πραγματικός μαθηματικός που, από αυτή την

άποψη , μπορεί να δικαιολογήσει τη ζωή του. Αν το κριη)ριο είναι αυτό, τότε ο Abel , ο Rieιηann και ο Poincan: σπατάλησαν τη ζω1Ί τους · η συμβολή τους στις ανθρώπινες αν{;σε ις ι'μαν αμελητέα, και ο κόσμος θα ήταν εξ ίσου cυτυχής και χωρίς αυτούς.»

Δ ικαιlί.JΟηιο:, μαά απδ 55 χplί ι•ια, ο JΙΓ'!άl.ος μαΟηματιιιΊίς A llan Τιιι·ίιη,: Μ ι: χθεσ ιν Ιi ανακοίνωσι) του ( 11 /9/2θ09) στην συστήματος «Enigma», το οποίο χρησιμοποιούσαν

επ ίση μη ιστοσι:λίδα της Ντάουνινγκ Στριτ, ο για τις επικοινωνίες τους οι ναζιστικές δυνάμεις. πρωθυπουργός Γκόρντον Μπράουν ζήτησε Χάρη στην επιτυχία του, οι Σύμμαχοι κατάφεραν συγγνιiψη για «τον απάνθρωπο τρόπο» που να υποκλέπτουν πληροφορίες για τις θέσεις του φιρΟη κc το βρcτανικό κράτος στον φημισμένο γερμανικού ναυτικού, aποτρέποντας έτσι τη μ αΟ η ματ ι κό και 1)ρωα του Β ' Παγκόσμιου διαιώνιση των πολεμικών επιχειρήσεων και Πολι':μου, Αλαν Το ύρ ινγκ . Ο Τούρινγκ έγινι.; σώζοντας με αυτόν τον τρόπο αναρίθμητες ειφύ τερα γνωστός για την καΟοριστtκJΊ συμβολ1Ί ανθρώπινες ζωές. του στο «σπάσιμω» του κρυπτογραφικού Εντούτοις, πέντε χρόνια μετά το τέλος του

Ε Υ ΚΛ Ε ΙΔΗΣ Β ' 74 τ.2/23

Page 26: Ευκλειδης Β 74

------------- ΗΟΜΟ MATHEMAτiCUS πολέμου, ο Τούρινγκ καταδικάστηκε για κάποιο παράπτωμά του . Η ποινή που του επιβλήθηκε ήταν "θεραπεία" με πειραματικά φάρμακα. Αν δεν την αποδεχόταν, θα έπρεπε να φυλακιστεί. Του έγιναν δεκάδες ενέσεις . . . , απολύθηκε από τη δουλειά του στη διεύθυνση επικοινωνιών του κράτους και τελικά αυτοκτόνησε το 1 954, σε ηλικία 4 1 ετών. «0 Τούρινγκ έπεσε θύμα της νομοθεσίας της εποχής και το ρολόι δεν μπορεί να γυρίσει πίσω στο χρόνο», σημείωσε μεταξύ άλλων στη χθεσινή του ανακοίνωση ο Βρετανός πρωθυπουργός και συμπλήρωσε: «Χωρίς τον Τούρινγκ, η εξέλιξη του Β · Παγκόσμιου Πολέμου ενδεχομένως να ήταν διαφορετική . Χάρη σε αυτόν, οι θηριωδίες του Ολοκαυτώματος αποτελούν μέρος του παρελθόντος της Ευρώπης και όχι του παρόντος. Εκ μέρους της βρετανικής κυβέρνησης, σου λέω με περηφάνια: συγγνώμη , δεν έπρεπε να σου φερθούμε έτσι» . Σημειώνεται ότι η ανακοίνωση έγινε ύστερα από αίτημα που υπέβαλαν 30 .000 πολίτες στην ιστοσελίδα του πρωθυπουργικού γραφείου, με την ευκαιρία της συμπλήρωσης 55 χρόνων από το θάνατο του Τούρινγκ. Οι πολίτες ζήτησαν επίσης από τη βασίλισσα να του απονείμει τον τίτλο του ιππότη μετά θάνατον. Εξάλλου, πέραν της

προσφοράς του στην πολεμική προσπάθεια, ο Τούρινγκ θεωρείται από τους πλέον διακεκριμένους επιστήμονες της γενιάς του, ενώ, το 1 999, το περιοδικό Time τον είχε συμπεριλάβει στις 1 00 σημαντικότερες προσωπικότητες του 20ού αιώνα.

Το πριί>το κομπιοί>τερ Πρωτοπόρος της πληροφορικής, ο Τούρινγκ κατασκεύασε τον πρώτο «σύγχρονο» υπολογιστή στο Πανεπιστήμιο του Μάντσεστερ, αμέσως μετά τον πόλεμο . Επίσης, έβαλε τα φιλοσοφικά θεμέλια στη μελέτη της τεχνητής νοημοσύνης, με την εργασία του «περί υπολογίσιμων αριθμών», το 1 936 . Τέλος, αξίζει να υπογραμμιστεί ότι επίκαιρο ακόμη και σήμερα θεωρείται το «test Turing», με το οποίο θεωρητικά καθορίζεται αν ένας υπολογιστής διαθέτει νοημοσύνη . Στην κλασική του μορφή , το test προβλέπει την ανταλλαγή γραπτών μηνυμάτων μεταξύ δύο ανθρώπων και ενός υπολογιστή από απομακρυσμένες τοποθεσίες. Εάν ο ένας άνθρωπος δεν αντιληφθεί ποιος από τους δύο «συνομιλητές» του είναι υπολογιστής, τότε θεωρείται ότι το μηχάνημα έχει νοημοσύνη . [Πηγή : εφημερίδα «ΚΑΘΗΜΕΡ Ι Ν Η » http ://news.kath i ιnerin i .gr/4dcgi/ _ w _articles_ world_ l 00070_ 1

2/09/2009-329062] Συμπί.ηρωματιια} βιογραφιια} του A llan Τιιι·ίιιχ

Η απροθυμία του Turing να εργαστεί σκληρά έστησε τρία από τα τέσσερα στάδια της στις κλασικές του σπουδές, όσο στα Μαθηματικά, ηλεκτρομηχανολογικής δυαδικής πολλαπλότητας. τον οδήγησε να αποτύχει να κερδίσει μια Τον Ιούνη του 1 938 έλαβε το Ph.D. από το υποτροφία στο Trinity Colledge, του Cambridge, Princeton . Μ' αυτή τη διδακτορική διατριβή και πήγε στο κολλέγιο της δεύτερης επιλογής του, εισήγαγε την έννοια της σχετικής υπολογιστικής, το King's College, του Cambridge. Έκανε ένα όπου οι "Μηχανές Turing" είναι ενισχυμένες με προπτυχιακό εκεί, από το 1 93 1 ως το 1 934, με τους ονομαζόμενους "χρησμούς" . διακεκριμένο βαθμό, και το 1 93 5 εκλέχτηκε μέλος Επέστρεψε στο Caιnbridge και παρακολούθησε του King's College με αφορμή την πραγματεία τα μαθήματα του Ludwig Wittgenstein πάνω στα του στο "centra1 limit theorem". θεμέλια των Μαθηματικών. Οι δυο τους άνοιξαν

Στη βαρυσήμαντη εργασία του «On διάλογο και διαφώνησαν, με τον Turing να Computable Numbers, with an App1 ication to the υπερασπίζεται τον φορμαλισμό και τον Wittgenstein Entscheidungs Prob1em» (υποβλήθηκε στις να υποστηρίζει ότι τα Μαθηματικά δεν 28/5/ 1 936), ο Turing αναδιατύπωσε τα ανακάλυψαν τις απόλυτες αλήθειες αλλά τις συμπεράσματα του Kurt Gδdel (του 1 93 1 ) πάνω εφεύραν. Από το Σεπτέμβρη του 1 938 ο Turing στα όρια της απόδειξης και του υπολογισμού, άρχισε να εργάζεται (με μερική απασχόληση) στο αντικαθιστώντας την Καθολική Αριθμητική του GCCS (Goνemment Code and Cypher School), δηλ. Gδdel με αυτό που σήμερα ονομάζεται "Turing στον βρετανικό οργανισμό για το "σπάσιμο" των Machine". Απόδειξε ότι κάθε τέτοια "Μηχανή" θα κωδίκων. είναι ικανή να εκτελεί ένα κατανοητό Κατά τη διάρκεια του Δευτέρου Παγκοσμίου υπολογισμό εάν αυτός ήταν δυνατόν να Πολέμου ήταν ένας ουσιώδης εταίρος στις παρασταθεί με έναν αλγόριθμο. Οι "Turing προσπάθειες που γινόντουσαν στο B1etchley Park Machines" είναι μέχρι σήμερα το κεντρικό (όπου η έδρα του GCCS) για σπάσιμο των αντικείμενο σπουδής στην "Theory Computation". κρυπτογραφικών κωδίκων των Γερμανών. Με

Από το Σεπτέμβρη του 1 936 ως τον Ιούλη βάση τις εργασίες στην Κρυπτανάλυση (που του 1 938 πέρασε τον περισσότερο καιρό του στο πραγματοποιήθηκαν προπολεμικά στην Πολωνία Institute for Adνanced Study, στο πανεπιστήμιο από τους: Marian Rejefski, Jerzy Rδzyski , Henryk του Princeton, μελετώντας υπό τον Alonzo Zygalski), ο Turing συντέλεσε αποφασιστικά στο Church . . . Μελέτησε Κρυπτολογία και επίσης "σπάσιμο" της μηχανής "Enigma" και του "Lorenz

ΕΥΚΛΕΙΔΗΣ Β' 74 τ.2/24

Page 27: Ευκλειδης Β 74

------------- ΗΟΜΟ MATHEMAτiCUS SZ 40/42 ' ' . Για ένα διάστημα ήταν επικεφαλής του ''Hut 8" (του τμήματος που ήταν αρμόδιο για το "σπάσιμο" των γερμανικών ναυτικών κωδίκων).

Στο GCCS εργάσθηκε πάνω στο πρόβλημα της γερμανικής μηχανής ' 'Enigma" και συνεργάστηκε με τον Dil ly Knox, που ήταν παλιός ειδικός στο θέμα. Στις 4/9/ 1 939, μια μέρα μετά την κήρυξη του πολέμου της Βρετανίας εναντίον της ΙΙα. 'Ά υηί το ξιiρατε; " /η απά1ιτησηj «Πάρτε μια μακριά χάρτινη ταινία, διπλώστε την στη μέση μία φορά και διπλώστε την και πάλι στη μέση . Στηρίξτε τη διπλωμένη ταινία όρθια στη μια πλευρά της και ξεδιπλώστε την σχηματίζοντας ορθές γωνίες σε κάθε τσάκισμά της ( σχ. 1 ). Αν την κοιτάξετε από πάνω, θα δείτε κάτι όπως τα διαγράμματα στα σχήματα 2α και 2β . . . . . . . . .

Έπειτα από τρία διπλώματα μπορούμε να πάρουμε τελείως διαφορετικά διαγράμματα (σχήματα 2γ και 2δ), ανάλογα με τον τρόπο που διπλώνουμε την ταινία. Στο σχήμα 3 βλέπουμε ένα από τα διαγράμματα που προκύπτουν αν διπλώσουμε την ταινία πέντε φορές. Είναι πρακτικά αδύνατο να διπλώσουμε την ταινία περισσότερες από εφτά φορές - μετά το όγδοο δίπλωμα θα υπάρχουν ήδη 28=256 στρώματα! Μπορούμε να σχεδιάζουμε διαγράμματα που θα αντιστοιχούν σε πολλαπλά διπλώματα της ταινίας, χωρίς την ταινία . . . . . . . . .

Αν τσακίσουμε την ταινία περισσότερες από τρεις φορές, τότε ξετυλίγοντάς την, μερικές από τις

σχ. Ι σχ. 2

Γερμανίας, ο Turing βρέθηκε στο Bletchley Park, στο σταθμό του GCCS. Το Νοέμβρη του 1 942 πήγε στις ΗΠΑ όπου εργάστηκε ως Κρυπταναλύστας στο Πολεμικό Ναυτικό για το Naval Enigma. Το Μάρτη του 1 943 επέστρεψε στο Bletchley Park.

[πηγή : Αγγλική Βικιπαίδεια, λήμμα «AI Ian Turing»]

γωνίες της θα "εφάπτονται" αναγκαστικά μεταξύ τους (σχήματα 2δ και 3) . Εξαιτίας των πολυπληθών επαφών αυτού του είδους, τα μεγάλα διαγράμματα έχουν περιοχές που μοιάζουν με πλέγμα παρά με μια επιμήκη ορθογώνια στρεφόμενη διαδρομή . Για να γίνει ορατή αυτή η διαδρομή , μπορούμε να στρογγυλέψουμε τις γωνίες της (όπως στην εσωτερικη καμπύλη του σχήματος 3) . . .

Μια τέτοια ακριβώς εικόνα ενέπνευσε στον Αμερικανό φυσικό John Ε. Heighway την ονομασία "δρακόντειες καμπύλες" . Οποιοσδήποτε έχει "δει" ένα δράκο μπορεί να επιβεβαιώσει ότι αυτά τα πλάσματα έχουν αυτή ακριβώς τη μορφή . Τα αποτελούμενα από ευθύγραμμα τμήματα διαγράμματα που παράγουν τα δρακόντειες καμπύλες ονομάζονται "δρακόντεια σχέδια"»

Στο σχήμα 4 μπορούμε να δούμε ένα από τα διαγράμματα που μπορεί να προκύψουν αν διπλώσουμε την ταινία 1 2 φορές. Αποτελείται από 2 1 2=4 .096 τμήματα. Ανάλογα έχουμε την καμπύλη στο σχήμα 5

σχ. 3

� 4 � 5 [πηγή : απόσπασμα από άρθρο των Nikolay Vasi lyeν και Victor Gutenmacher «ΔΡΑΚΟΝΤΕΙΕΣ ΚΑΜΠΥΛΕΣ», περιοδικό QUANTUM, τόμ. 2, τεύχ. 6, εκδ. ΚΑΤΟΠΠΡΟ, Αθήνα I 995]

ΕΥΚΛΕΙΔΗΣ Β' 74 τ.2/25

Page 28: Ευκλειδης Β 74

���,, Μαθ ηματ ι κά 1 1

i ι

γ ι α την Α ' τάξ η τ ο υ Λυκε ίου 1 1 Εξισώσεις δευτέρου βαθμού - Σύνολα Συναρτήσεις - Η συνάρτηση f(x) = αχ+β

Χ Ι") Ι-i Σ Ι ί\1 [Σ Ε Π Ι Σ Ι-Ι Μ Α :\ Σ Ε Ι Σ Ι . Α ν Δ = β2 - 4αγ είναι η διακρίνουσα της εξίσωσης αχ2 + βχ + γ = Ο όπου α, β, γ πραγματικοί αριθμοί και α i- Ο τότε προκύπτουν τα παρακάτω συμπεράσματα: • Α ν Δ>Ο η εξίσωση έχει 2 ρίζες άνισες τις

-β + JΔ -β - JΔ χ ι= χz= -'-----2α 2α

• Αν Δ=Ο η εξίσωση έχει μια διπλή ρίζα την

Χ0 = -β (ισοδύναμα δύο ρίζες ίσες τις χ ι=χz= -β ) 2α 2α

• Αν Δ<Ο η εξίσωση δεν έχει πραγματικές ρίζες. 2. Αν οι συντελεστές α, γ της εξίσωσης αχ2+βχ+γ=Ο είναι ετερόσημοι, η εξίσωση έχει πάντοτε δύο ρίζες άνισες. 3. Η εξίσωση x2-Sx+P=O με S2 2 4Ρ μας δίνει την δυνατότητα: • Να βρίσκουμε 2 αριθμούς αν είναι γνωστό το

άθροισμά τους S και το γινόμενό τους Ρ . π.χ. Οι αριθμοί που έχουν άθροισμα S = 3/2 και γινόμενο Ρ= I /2 είναι οι ρίζες της εξίσωσης 2 3 I

Ο . λ , χ -- χ + - = προκειται οιπον για τους 2 2

αριθμούς Ι και Υ2 • Να λύνουμε εξισώσεις χωρίς την διακρίνουσα

αφού βρούμε δυο αριθμούς που έχουν γινόμενο Ρ και άθροισμα S . π .χ. Οι ρίζες της x2- (J3 + 2)χ + 2J3 =Ο είναι

προφανώς οι .J3 και 2 . • Α ν φυσικά γνωρίζουμε τις ρίζες της

δευτεροβάθμιας εξίσωσης, π.χ. Χ ι = 2, Xz = 3 τότε η εξίσωση βρίσκεται άμεσα. Πρόκειται για την (χ - 3)(χ - 2) = Ο, ισοδύναμα

Κυριακοπούλου Κωνσταντίνα

χ2 - 5χ + 6 = ο 4 . Για δύο ευθείες ε 1 : y=α 1 χ+β 1 και

ε 2 : y=α 2 χ+β 2 ισχύουν οι ισοδυναμίες:

ε Ι //ε 2 <;:::;> {αΙ = α2 } και ε Ι :: εz <;:::;> {αΙ : α2 } β Ι :;t: β2 β Ι - β2 5 . Για την εύρεση του πεδίου ορισμού μιας συνάρτησης f χρησιμοποιούμε πάντα τον αρχικό τύπο της f και όχι αυτόν που προκύπτει μετά από πιθανές aπλοποιήσεις . 6. Για να βρούμε τα σημεία τομής μιας ευθείας y=αχ+β ,α :;t: Ο με τους άξονες εργαζόμαστε ως εξής : • Θέτουμε χ = Ο και έτσι βρίσκουμε το σημείο

τομής με τον y 'y που είναι τοΒ(Ο,β) .

Ουσιαστικά λύνουμε το σύστημα {Υ = αχ + β} χ = Ο

• Θέτουμε y=O και έτσι βρίσκουμε το σημείο

τομής με τον χ 'χ που είναι το A(-f ,o). α

Ουσιαστικά λύνουμε το σύστημα {Υ = αχ + β} y = O

ί\σκηση 1 11 Δίνεται η εξίσωση 2χ2- 2χ+(2 - 6αβ) = Ο (I) όπου

α, β πραγματικοί αριθμοί.

α) Αν ο αριθμός ρ = α + β είναι μια ρίζα της

εξίσωσης, να βρείτε τις τιμές των α, β.

β) Για τις τιμές των α, β που βρήκατε να λύσετε

την εξίσωση.

ΛίJση α) Αφού ο αριθμός ρ = α + β είναι ρίζα της εξίσωσης, τότε την επαληθεύει, επομένως έχουμε: 2( α + β)2 - 2( α + β) + (2 - 6αβ) = Ο :::::>

ΕΥΚΛΕΙΔΗΣ Β ' 74 τ.2/26

Page 29: Ευκλειδης Β 74

Μαθηματικά για την Α ' Λυκείου

2( α2 + 2αβ + β2 ) - 2α - 2β + 2 - 6αβ = Ο � 2α2 + 4αβ + 2β2 - 2α - 2β + 2 - 6αβ = Ο � 2α2 - 2αβ + 2β2 - 2α - 2β + 2 = Ο � α2 + α2 - 2αβ + β2 + β2 - 2α - 2β + 2 = Ο � ( α2 - 2α + 1 ) + ( α2 + 2αβ + β2) + (β2 - 2β + 1 ) = Ο � (α - 1 )2 + (α - β)2 + (β - 1 )2 = ο� α = β = 1 β)Λύνουμε την εξίσωση , για α = β = 1 Έχουμε: (Ι) <=> 2χ2 - 2χ + 2 - 6 = Ο <=> <=> 2χ2 - 2χ - 4 = ο <=> χ2 - χ - 2 = ο όμως Δ = β2 - 4αγ = Ι +8 = 9 > Ο

1 + 3 1 - 3 Άρα χ 1 = -- = 2 και Xz = -- = - 1 2 2 Πράγματι Χ ι = α + β ί\σΥ{ηση 2 11

α)Να βρεθούν οι πραγματικοί αριθμοί β και γ

ώστε οι ρίζες της εξίσωσης i + (β-1)χ+γ+2 =0

να είναι οι β και γ+ 2.

βχ2 + γχ + 2 ' β θ ' β)Έστω f(x) = , οπου , γ οι αρι μοι χ· - 1

που βρήκατε στο προηγούμενο ερώτημα για

τους οποίους επιπλέον ισχύει β ·γ = -3. Να λυθεί

η εξίσωση : f(-2)(x - 1 )2 - 1 7 = 4 Ιχ - 1 1 - 9 (1) . \ α) Για να είναι οι αριθμοί β και γ+2 είναι ρίζες της εξίσωσης χ � + (β - Ι )χ + γ + 2 = 0 , πρέπει και αρκεί : β+γ+2= -β+ 1 (Ι) και β(γ+2) = γ+2 (ΙΙ) (Παρατηρήστε ότι αν S = Χ ι + Xz , Ρ = Χ ι Χ2 , τότε S2 - 4Ρ = (Χ ι + Χ2)2 - 4Χ ι Χz = (χ ι - Xz)2 ;::: 0 οπότε η χ2 - Sx + Ρ = Ο έχει σίγουρα ρίζες στο R.) Έχουμε: (Π) <=> β (γ+2) - (γ+2) = Ο <=> (γ+2) (β--1 ) = 0 <:::> γ+2=0 ή β--1=0 <:::> γ=-2 ή β= 1

Ι Αν γ = -2 τότε: (Ι)<:::>β-2+ 2=-β+ 1 <=> 2β= 1 <=> β= 2 Αν β = 1 , τότε: (Ι) <=> Ι +γ+2= - 1 + Ι <=> γ= -3

1 β)Αν (β, γ)= ( 2 , -2) τότε β ·γ = - Ι ,άρα δεν επαληθεύεται η β ·γ = -3 . Αν (β, γ)=( Ι , -3) τότε ισχύει β ·γ = - 3 ' χ 2 - 3χ + 2 δ ' ' αρα f(x) = 2 με πε ιο ορισμου το χ - 1 (-οο ,- Ι ) υ (- Ι , 1 ) υ ( Ι , + οο ) . Επίσης f -2 = (-2)2 - 3·(-2) + 2 = 4 + 6 + 2 = .!3. = 4 ( ) ( -2)2 - Ι 4 - 1 3 Άρα: ( 1 ) <=> 4•(χ - 1)2 - 1 7 = 4•Ι χ - 1 Ι -9 <=>

<=> 4·1 χ - 1 1 2 -4·1 χ - 1 1 -8 = ο <=> Ι χ - 1 12 - Ι χ - 1 1 -2 = 0 <=> ( I χ - 1 1 -2)·(1 χ - 1 1 +1) = 0 <=> Άρα Ιχ- 1 1 = 2 <=> χ-1 = 2 ή χ-1 = -2 <:::> χ=3 ή χ = -1 , αφού Ιχ- 1 1+ 1 >0 για κάθε x εiR

ί\ σ κη ση 3' 1 Δίνονται τα σύνολα {α} και { l ,β} για τα οποία

ισχύει {α} ς { 1 , β} .

α)Να δείξετε ότι η εξίσωση χ2 - α2χ + αχ - αβχ + βχ - 4χ +2 = Ο (Ι) έχει δύο

ρίζες πραγματικές και άνισες. β)Α ν χ1 , χ2 οι παραπάνω ρίζες , να υπολογίσετε

' ' 1 4 1 4 η τιμη της παραστασης 2 χ 1 + 2 χ2 • γ)Να εξηγήσετε χωρίς να υπολογίσετε τους αριθμούς χ 1 , χ2 ότι χ 1 > Ο, χ2 > Ο και να

κατασκευάσετε εξίσωση δευτέρου βαθμού με

ρίζες τους αριθμούς F;,Fz . Λίιση : α) Για α = Ι καθώς και για α = β έχουμε: (Ι) <:::> χ 2 - 4χ + 2 = Ο , η οποία έχει δύο ρίζες πραγματικές και άνισες αφού Δ= ( - 4)2 - 4·2 · 1 = Ι 6 - 8 = 8 > 0 όμως: β) Σύμφωνα με τους τύπους του Vieta θα ισχύουν οι σχέσεις : χ 1 + χ 2 = 4 και Χ ι Χz =2 . Επίσης χ 1 2 + χ/ =( χ 1 + χ 2 ) 2 - 2χ 1 •χ 2 = 42 - 2• 2=

, ι 4 1 4 1 ( 4 4 ) = Ι 6-4= Ι 2 . Οποτε : - χ 1 + - χ , =- χ 1 + χ , = 2 2 - 2 -=- χ - + χ - - 2χ · ·χ · = ι [( , ., )2

., ., J 2 Ι 2 I 2 =�[ ( χ 1 2 + χ/ )2 - 2 ( χ 1 •χ 2 )2 J =�·( 1 22 - 2•2 2 )

1 ι = 2·( 1 44 - 8) = 2·ι 3 2 = 66 γ) Για τους αριθμούς χ 1 , χ 2 ισχύει Χ ι Χz =2 >Ο άρα είναι ομόσημοι. Επιπλέον χ 1 + χ 2 = 4 >Ο άρα Χ ι > Ο, Xz > Ο Ακόμη (Fι +F )2 =Fι2 + 2Fι·F +F2 = = χ 1 + 2Jx 1 •X 2 + x2 = 4+2 J2 .Αρα Fι + F; = J 4 + J2 .Επιπλέον έχουμε Fι·Fz = .}χ 1 •χ 2 = J2 ,οπότε η ζητούμενη εξίσωση είναι η χ 2 - .}4 + 2J2x + J2 = Ο

ΕΥΚΛΕΙΔΗΣ Β ' 74 τ.2/27

Page 30: Ευκλειδης Β 74

Μαθηματικά για την Α ' Λυκείου

Να βρεθεί το πεδίο ορισμού των συναρτήσεων: 9 r-- 5χ

α) f(x) = β) f(χ) = ν-χ + ---Ι χ + 3 Ι +χ + 3 Ι Ι χ Ι -5 1 -7

) f( 7x + Jx - 2

γ χ) = . χ2 + 3χ + 15

α)Για να ορίζεται η συνάρτηση αυτή πρέπει αρκεί: Ιχ+3 1 -ι:. - (χ+3) (1) . Όμως (I) <=> χ+3>0 <:::> χ>-3 . Άρα το πεδίο ορισμού της f είναι το (-3 ,+ οο ) . β) Ένας αριθμός χ Ε IR. ανήκει στο πεδίο ορισμού της συνάρτησης f αν και μόνο αν : ��� ����*ο} (Σ) Αλλά (Σ) <c> { ι -χ -

,5�-ΟΜ}

χ <Ο χ < Ο <=> { - <=> χ+5 -ι:. 7 <=> χ-ι:.2 <=> { - } {χ � Ο {χ�Ο

Ι χ +5 Ι-ι:. 7 χ -ι:. -12 χ +5 -ι:.-7 χ-ι:.-12 Άρα, το πεδίο ορισμού Α της f είναι :

Α=(-οο ,- 1 2) υ (-1 2 ,0] γ) Ομοίως πρέπει και αρκεί: χ2 + 3χ + 15 -ι:. Ο . που ισχύει για κάθε x EIR αφού Δ=9-60 =- 5 1 < Ο. Άρα το πεδίο ορισμού της συνάρτησης f είναι το IR.

Μία συνάρτηση f IR--+IR έχει σύνολο τιμών το

( 1 , 10 1 . Να βρεθούν οι τιμές του λ Ε IR. ώστε η

εξίσωση f(x)-lλ+2 j=1 :

α) Να έχει τουλάχιστον μία λύση

β) Να είναι αδύνατη .

Εφόσον η f έχει σύνολο τιμών το ( 1 , 1 Ο] θα ισχύει: 1 <f (χ) � 1 0 . για κάθε χ ε IR. Έχουμε: f(x)-lλ+2 1= 1 <=> f (x)=lλ+2 1+ 1 α) Έχει τουλάχιστον μία λύση όταν και μόνο όταν: { I λ + 2 1 + 1 > 1 l < jλ+2 1+ 1 � 1 0 (I) Αλλά (I) <=> <=> l λ + 2 1 + 1 � 1 0 { I λ + 2 1> ο {λ -ι:. -2 {λ -ι:. -2

Ι λ + 2 1� 9 <=> -9 � λ + 2 � 9 <=> - 1 1 � λ � 7 <:::> λ Ε [- 1 1 , -2) υ (-2,7] β)Η εξίσωση είναι αδύνατη ,όταν και μόνο :

λ ε (-οο ,- 1 1 ) υ {-2} υ (7, + οο ) ι) 1 j

σε ποιο τεταρτημόριο βρίσκονται τα σημεία

Α(κ - 7, λ2 - 3)

β) Αν γνωρίζετε ότι το Β(5, -1 )είναι συμμετρικό

του Α ως προς την αρχή των αξόνων ,να βρεθεί

το κ.

γ) Να βρεθεί σημείο Γ του αρνητικού ημιάξονα

Οy 'ώστε το τρίγωνο ΑΒΓ να είναι ορθογώνιο με

υποτείνουσα την ΑΒ.

. \ α) Παρατηρούμε ότι Ικ-5 1 = 5-κ =:> κ-5 < Ο =:> κ < 5 =:> κ-7 < Ο =:> χΑ < Ο. Επίσης Ι-3λ+6 1 � Ο =:>-3λ+6=0 =:> λ=2 =:> λ 2 -3= l >O=:>yA>O . Άρα τα σημεία Α(κ-7 , λ 2 -3) βρίσκονται στο 2° τεταρτημόριο. β)τα Α, Β είναι συμμετρικά ως προς την αρχή των αξόνων =:> κ-7 = -5 =:> κ = 2 . γ)Είναι: Α( -5 , 1 ) , Β(5 , - 1 ) και έστω Γ(Ο,μ) με μ<Ο το ζητούμενο σημείο . Τότε : (ΑΒ)2 = (-5 - 5)2 + ( 1 + 1 )2 = 1 00+4 = 1 04 (ΑΓ)2 = 52 + (μ - 1 )2 =25 + μ2 - 2μ + 1 = 26-2μ+μ2 . (ΒΓ)2 = 52 + (μ + 1 )2 = 25 + μ2 + 2μ + 1 =26+2μ+μ2 Το τρίγωνο ΑΒΓ είναι ορθογώνιο με υποτείνουσα την ΑΒ, <=> ΑΒ 2 =ΒΓ 2 +ΑΓ 2 <:::> 1 04 = 26+2μ+μ 2 +26-2μ+μ 2 <:::> 1 04 = 52+2μ 2 <=> <:::> 52 = 2μ 2 <:::>26 = μ 2 <=> μ= - J26 , αφού μ < Ο. λ σ κη ση 7'' {7χ + 7, ανχ 2 3 Δίνεται συνάρτηση f(x) =

5αχ - 2, ανχ � 3

α)Να βρεθεί ο α.

β)Να βρεθούν οι τιμές του μ Ε IR. ώστε οι ευθείες 5

ε , : y = - -•f(1)• 1 μ + 1 j •x + 7 και 8

ε 2 : y = [(μ+ 1)2 - 6 ]χ + 2010 να είναι παράλληλες.

α)Επειδή η f είναι συνάρτηση πρέπει οι τιμές της για χ=3 και στους δύο τύπους να είναι ίσες. οπότε : 7 · 3 +7 = 5α·3 -2 =:> 2 1 +7 = 1 5α-2 =:> {7χ + 7 , ανχ � 3 1 5α = 30 =:> α = 2 . Άρα: f(x) = 1 0χ - 2, ανχ � 3 β)Είναι f(l) = 1 0-2 = 8 οπότε, ε 1 : y =-.S.I μ+ l l•x + 7 {-5 lμ + l l = (μ + l) 2 - 6} επομένως ε , I I ε2 <=>

α) Αν για τους πραγματικούς αριθμούς κ, λ 7 * 20 1 0 ισχύουν Ικ-5 1 = 5-κ και Ι-3λ+6 1 � Ο, να βρείτε <=>-5 lμ+ l l = (μ+ 1 ) 2 -6 <=> l μ+ l l 2 +5 lμ+ l l-6 =Ο

ΕΥΚΛΕΙΔΗΣ Β' 74 τ.2/28

Page 31: Ευκλειδης Β 74

Μαθηματικά για την Α ' Λυκείου

<=> { l μ + 1 1 = ω } <=> {1μ+� =ω}<=> lμ+ 1 1= 1 <:::> ω2 + 5ω- 6 = Ο ωΕ{4--6} (μ + l = 1 ή μ + 1 = -1 ) <=> μ = ο ή μ = -2 . Λ σ κη σv1 !'; ' � {3, ανχ Ε Q Δίνεται η συνάρτηση f(x) =

Ο, ανχ E JR - Q

α)Να βρεθεί η εξίσωση της ευθείας (ε ι )που

διέρχεται από το σημείο Α ( f(.J3) , f(0,7) ) και

είναι παράλληλη στην ευθεία (ε 2 ) :4y-2x = 6.

β)Να δείξετε ότι τα σημεία A (r {.J3) ,r (o, 7)) Β ( r( r{ Fs}) , 5) και Γ ( -ι,1) είναι συνευθειακά.

γ)Αν Κ είναι τα σημείο τομής της (ε ι ) με τον

χ ' χ, Λ το συμμετρικό του Α ως προς τον χ ' χ

και Ο η αρχή των αξόνων να βρεθεί η

περίμετρος του τριγώνου ΚΛΟ, το εμβαδόν του

και το ύψος που αντιστοιχεί στην υποτείνουσα.

Ο J3 είναι άρρητος άρα f( J3) = Ο και ο Ο, 7 = 2 9 είναι ρητός άρα f(O, 7 ) = 3 , οπότε Α (0,3) . Λύνουμε την εξίσωση της (ε 2 ) ως προς y προκειμένου να βρούμε τον συντελεστή διεύθυνσης της και έχουμε : (ε 2 ) :y=..!.. χ + � οπότε 2 2 ' λ ' δ ' θ 1 εχει συντε εστη ιευ υνσης α 2 = -2 {y = ..!.. χ + κ} Άρα (ε ι ) // (ε 2 ) <=> (ε ι ) : 2

3 Κ "#- -2

Εξ άλλου : (ε ι ) <=> 3= ..!_ ·Ο+κ <=> κ = 3 , δεκτή τιμή 2 αφού 3 7:- � . Άρα (ε ι ) : y = ..!.. χ+3 . 2 2 β) Είναι f(f( J5 ) = f(O) = 3 ,άρα Α(Ο,3) , Β(3 ,5) και Γ( -1 , 2 ) . Βρίσκουμε την εξίσωση της ευθείας 3 ΑΒ. Αυτή θα είναι της μορφής y = αχ+β και επειδή τα Α, Β ανήκουν σε αυτήν οι συντεταγμένες τους θα την επαληθεύουν. Άρα προκύπτει το εξής σύστημα:

<::::> <::::> <=> β = 3 και {3 = α•Ο + β {β = 3 {β = 3 5 = α•3 + β 5 = 3•α + 3 3·α = 2

α = 2/3 . Δηλαδή η εξίσωση της ευθείας ΑΒ είναι η 2 3 π ' ' ' y= -• χ+ . αρατηρουμε οτι οι συντεταγμενες του 3

Γ επαληθεύουν την εξίσωση αυτή , αφού για χ=-- 1 παίρνουμε y=�• (- 1 )+3=2 ,οπότε τα Α, Β, Γ 3 3 είναι συνευθειακά . γ) Για να βρούμε το σημείο τομής Κ της (ε ι ) με τον χ 'χ θέτουμε όπου y = Ο και έχουμε:

Ο = ..!_ χ+3 <=> χ = -6 . Άρα Κ(-6, 0). 2 Το συμμετρικό του Α (0, 3 )ως προς τον χ 'χ είναι το Λ(Ο, -3 ) .Είναι ΚΟ = 1-6 1 = 6, ΛΟ = 1 -3 1 = 3 και ΚΛ= �(0 + 6)2 + (-3 + 0)2 = -J36+ 9 = -J4s =3 J5 Άρα η περίμετρος του ΚΛΟ είναι ίση με 6+ 3+ 3 J5 =9+ 3 J5 και το εμβαδόν του ίσο με Ε= ..!_ΚΟ·ΟΛ = ..!_·3·6 = 9. Για το ύψος ΟΡ που 2 2 αντιστοιχεί στην υποτείνουσα ΚΛ έχουμε : ..!_·ΟΡ·ΚΛ=Ε => ..!.. . op .3)5 =9 => OP=_i_ =>ΟΡ= 6J5 2 2 J5 5

Δίνεται συνάρτηση f : JR -7 JR για την οποία

ισχύει : f(4x-3) = 2x+l0- f(5) για κάθε χ Ε JR . α)Να βρεθεί το f(5). β)Να βρεθεί ο τύπος της f.

γ)Να αποδείξετε ότι για κάθε α , β Ε JR ισχύει

f(α2 + 1) + f(β2) > 2f(αβ)

δ)Αν για τρεις πραγματικούς αριθμούς κ, λ, μ

ισχύει κ + λ = μ-1 , να αποδείξετε ότι :

f(κ3)+f(λ3 + 3λ(λ+1)+1 )+f(- μ3) = f(-3κ(λ+l)·μ)+9

α)Η σχέση f(4x-3) = 2χ+ 1 0- f(5) ισχύει για κάθε χ Ε JR . Θέτοντας όπου χ το 2 παίρνουμε : f(5)=4+ 1 0 -f(5 ) => 2f(5) = 1 4 => f(5) = 7 β)Αφού f(5 ) = 7 από την σχέση ; f(4x-3) = 2χ+ 1 0 - f(5) παίρνουμε: f(4x-3) = 2χ+ 1 0 - 7 => f(4x -3) = 2χ +3 . Θέτουμε y = 4χ-3 δηλαδή χ = Υ + 3 και με 4 αντικατάσταση προκύπτει: f ( Υ) = 2· Υ + 3 + 3 = Υ + 3 + 3 = Υ + 9 . 4 2 2

ΕΥΚΛΕΙΔΗΣ Β ' 74 τ.2!29

Page 32: Ευκλειδης Β 74

Μαθη ματικά για την Α ' Λυκείου

Άρα f (χ) = χ + 9 . 2 γ)Αρκεί να δείξουμε ότι f(α2 + Ι) + f(β2 ) > 2f(α•β) ,

, α2 + Ι + 9 β2 + 9 2 αβ + 9 , 2 β2 αρκει + -- > -- , αρκει α + 2 2 2 + Ι 9 - 2αβ - Ι 8 > Ο, αρκεί ( α-β)2+ 1 >0, που ισχύει. δ)Έχουμε : f(κ3 ) + f(� + 3λ•(λ + Ι) + Ι ) + f(-μ3 ) =

κ3 + 9 (λ + 1 )3 + 9 (-μ )3 + 9 = -- + + = 2 2 2 27 + κ3 + (λ + 1)3 - μ3 Ό λ Ι -----'---'------'---- . μως κ+ = μ- � 2

κ+λ+ Ι -μ = Ο ,� κ3 + (λ + 1)3 - μ3 = -3κμ(λ + Ι ) . Άρα f(κ3 ) + f(� + 3λ•(λ + Ι) + l) + f(-μ3 ) = = 27 - 3κμ(λ + l ) = -3κμ(λ + 1) + 9 + 9 = 2 2 = f(-3κμ(λ + 1)) + 9 ;�, ςy κψιη 10 ' 1 Δίνεται η εξίσωση χ2 - (2λ - 3)χ + (1 - 3λ) = Ο ,

με λ Ε 1R. . α)Να βρεθούν οι τιμές του λ Ε 1R. ώστε

η παραπάνω εξίσωση να έχει δύο πραγματικές

ρίζες αρνητικές και άνισες.

β)Να βρεθούν οι τιμές του λ Ε 1R. ώστε η

εξίσωση να έχει δύο ρίζες πραγματικές και άνισες για τις οποίες ισχύει ρ 1 = - 2ρ 2 •

α) Δ= (3-2λ)2 -4(Ι-3λ) = 9-12λ+4Κ -4+ Ι2λ=4Κ +5 Για να έχει η εξίσωση δύο πραγματικές ρίζες αρνητικές και άνισες πρέπει και αρκεί: {Δ > Ο, � > Ο, -� < Ο} (Σ)

Αλλά : (Σ) <=> Ι-3λ>Ο <=> � <=> λ < 1 { λ Ε R } ιλ<�} 2λ-3 <0 λ<-3

β) Για κάθε λ Ε IR πρέπει και αρκεί να υπάρχε λ Ε 1R. ώστε να είναι συμβιβαστό ως προς ρ ι . ρ2 τ

σύστημα: {�::=�-3t� ( 1 ) ,(2)<=> { ρ 1 = 2λ - 3 } ρ? = 3 - 2λ �Pz =l-3λ (3) -

Για να έχει το σύστημα των τριών εξισώσεων λύση ως προς ρ ι , ρ2 πρέπει και αρκεί οι τιμές των ρ ι , ρ2 που βρήκαμε να επαληθεύουν την (3) , δηλαδή : - (2λ - 3)2 = Ι - 3λ (4)

(4) <::> 4λ2 - 1 5λ + l Ο = Ο <::> λ = I 5 ± J65 8 ί\σκη ση 1 1 ' Ι

Δίνονται τα σύνολα :

Α={χ E 1R. I (x 2 - 1 )(χ 4 -13χ 2 +36)=0}

1 8χ4 - 3 ' Β={ χ Ε Ζ I Ε Ζ } , Γ={ χ Ε Ζ I οι αριθμοι χ

4χ\ 4χ -1 είναι διαδοχικοί άρτιοι}

α)Να γράψετε με αναγραφή των στοιχείων τους,

τα παραπάνω σύνολα.

β)Να βρεθούν τα σύνολα Α υ Β , Α n Β ,Α n Γ.

γ)Να εξεταστεί αν Β ς Α.

,\!.'Ιση : Έχουμε : (χ 2 -I )(χ 4 - 1 3χ 2 + 36)=0 <=> χ 2 - Ι = Ο ή χ 4 - 1 3χ 2 + 36 = Ο <::> χ = Ι ή χ = -Ι ή χ 4 - Ι 3χ 2 +36 = 0

? { χ 2 = ω } Αλλά: χ 4 - 1 3χ - + 36 = Ο <=> ω2 - Ι 3ω + 36 = Ο { χ2 = ω } 2 2 <=> <=> χ = 9 ή χ = 4 <::::> ω Ε {4, 9} <=> χ Ε { -3 , -2, 2 , 3 } τελικά Α = {-3 ,-2,- I , I ,2,3 } Είναι : Ι8χ4 -3 = Ι 8χ 3 - � Επειδή χ Ε Ζ , 1 8χ 3 ΕΖ χ χ θ ' Ι 8χ4 - 3 '71 3 '71 δ ' α εχουμε Ε ιu <=> - Ε ιu <=> χ ιαιρετης χ χ του 3 <=> χ = -3 ή χ = 3 ή χ = 1 ή χ = - I . Άρα Β= {-3 -1 ι 3 } ' ' ' Για κάθε χ Ε z είναι (2χ- \ ) 2 > ο � 4χ 2 -4χ+ Ι >Ο � 4χ 2 > 4χ - 1 . Οι αριθμοί 4χ 2 ,4χ -1 είναι διαδοχικοί άρτιοι<::>4χ 2 -( 4χ- 1 )=2 <=> 4χ 2 -4x-l =Ο Έχουμε Δ = \ 6+ Ι 6 = 32 > Ο οπότε

4 + .ffi 4 + 4.J2 I + .J2 Ι - .J2 Χ ι = = = -- και χ2 = --8 8 2 2 Καμία όμως από τις δύο αυτές τιμές δεν είναι ακέραια ,άρα Γ= 0 β) Η ένωση δύο συνόλων αποτελείται από τα κοινά και τα μη κοινά στοιχεία των συνόλων αυτών, άρα: Α υ Β = {-3 , -2, - Ι , I , 2, 3 } Η τομή δύο συνόλων αποτελείται από τα κοινά στοιχεία των συνόλων αυτών ,άρα:

Α n Β = {-3 -I Ι 3 } Α n Γ = 0 ' ' ' ' γ)Κάθε στοιχείο του Β ανήκει στο σύνολο Α άρα Β ς Α.

ΕΥΚΛΕΙΔΗΣ Β ' 74 τ.2/30

Page 33: Ευκλειδης Β 74

Μαθη ματικά για την Α ' Λυκείου

ΠΑΡΑι\ΛΗΛΟ ΓΡΑΜΜΑ κα ι ΤΡΑΠ ΕΖΙΑ

Θανάσης Χριστόπουλος - Παναγιώτης Χριστόπουλος

Μέχρι τώρα τα αξιώματα, τα θεωρήματα και τα πορίσματα, μας δίνουν τη δυνατότητα να

αντιμετωπίσουμε γεωμετρικά προβλήματα με πολλούς τρόπους, να δώσουμε δηλαδή πολλές διαφορετικές

λύσεις. Στα προβλήματα που ακολουθούν θα στηριχθούμε στις προτάσεις του 4"" και 5°" κεφαλαίου του

σχολικού βιβλίου για την καλύτερη και πληρέστερη κατανόησή τους.

Γ .. kχημα τηω. δίνουμε τις ικανές προτάσεις που μας οδηγούν από το ένα γεωμετρικό σχήμα στο άλλο με

τις σχετικές ιδιότητες.

-�\ \ '' ΤΡΑΠΕΖΙΟ I ' 1 ΤΕΤΡΆΠΛΕΥΡΟ αν έχει δυο π�υρές παράλληλες \ τοτε

_.-------��----------------------------, ,------- � Αν

Απέναντι πλευρές παράλληλες (ορισμός)

ή Απέναντι πλευρές ίσες ή Απέναντι γωνίες ίσες

ή Διαγώνιοι διχοτομούνται ή Δυο πλευρές παράλληλες και ίσες

τότε l Ln ΑΡΑΛΛΗΛΟΓΡΑΜΜΟ 7 Αν l Αν l Αν l

0 1 : Μια γωνία ορθή

ή 02 : Διαγώνιοι ίσες

τότε

ΟΡΘΟΓΩΝΙΟ

Κα� μια από τις Ρ

τότε

Μια Ο πρόταση και Pl : Δυο διαδοχικές πλευρές ίσες

μια Ρ πρόταση ή Ρ2 : Διαγώνιοι τέμνονται κάθετα

ή Ρ3 : Διαγώνιος διχοτομεί μια γωνία του

τότε \ τότε

ΤΕΤΡΆΓΩΝΟ και

μια από τις Ο

τότε

ΕΥΚΛΕΙΔΗΣ Β ' 74 τ.2/31

Page 34: Ευκλειδης Β 74

Μαθηματικά για την Α ' Λυκείου

Ερωη1σεις σωστοίJ- λάΟους

Να χαρακτηρίσετε τις παρακάτω προτάσεις ως σωστές (Σ) ή λάθος (Λ). Ι . Οι διαγώνιες ενός ρόμβου είναι ίσες. 2 . Το τετράγωνο είναι ρόμβος. 3 . Η διάμεσος τραπεζίου ισούται με την ημιδιαφορά των βάσεών του. 4 . Το άθροισμα των εξωτερικών γωνιών ενός Ι 6-γωνου είναι 4 ορθές. 5 . Αν σε ένα τετράπλευρο οι τρείς γωνίες του έχουν άθροισμα 270° τότε αυτό είναι ορθογώνιο . 6 . Οι διαγώνιες του ορθογωνίου είναι κάθετες. 7 . Οι διαγώνιες ενός ισοσκελούς τραπεζίου είναι ίσες. 8 . Το τετράπλευρο που έχει ίσες διαγώνιους είναι ορθογώνιο . 9 . Αν η διάμεσος ενός τραπεζίου διέρχεται από το σημείο τομής των διαγωνίων του τότε αυτό είναι

παραλληλόγραμμο . Ι Ο . Το μέσον της υποτείνουσας κάθε ορθογωνίου τριγώνου ισαπέχει από τις κορυφές του. Ι 1 . Τα μέσα ενός ρόμβου ορίζουν κορυφές ορθογωνίου.

Δίνεται παραλληλόγραμμο ΑΒΓΔ

προεκτείνουμε τη ΒΓ κατά ΓΕ=ΒΓ, από το Ε

φέρουμε την ΕΖ κάθετη στην ΑΒ. Δείξτε ότι

ΑΓ=ΔΖ.

Στο τρίγωνο ΕΖΒ, Γ μέσον της ΕΒ και ΓΘ//ΒΖ άρα το Θ είναι μέσον της ΕΖ αλλά Δe-1-ΕΖ οπότε ΔΘ μεσοκάθετος της ΕΖ και ΔΕ=ΔΖ( Ι ) . Επίσης είναι ΑΔΕΓ παραλληλόγραμμο ( ΑΔ : ΓΕ) επομένως ΔΕ=ΑΓ(2) και τελικά από τις σχέσεις ( Ι ) , (2) είναι ΔΖ=ΑΓ

Είναι ΑΖ//ΔΓ άρα ΑΖΓΔ τραπέζιο, επίσης ΖΓ διάμεσος στο ορθογώνιο τρίγωνο ΖΒΕ οπότε ΖΓ = ΒΕ = ΒΓ αλλά ΒΓ=ΑΔ επομένως ΖΓ=ΑΔ 2 αυτό σημαίνει ότι το τραπέζιο ΑΖΓ Δ είναι ισοσκελές και θα έχει διαγώνιους ΑΓ=ΔΖ

Α Ζ Β ,�-- , _, I .: ' , ; \ / ' ,

"' \ I '"'-. \

, " ' \ I , "' ' , ' / , " ' \

' / ' "' Ι'o---- ------···-----:::f---f Δ ... ... Θ Γ ... ... ... ... ... ... ... ... ... ... Ε

Σχ. 1

λσκη ση 2 '1 Δίνεται τραπέζιο ΑΒΓΔ (ΑΒ//ΓΔ). Αν Η το Λ Λ

σημείο τομής των διχοτόμων των γωνιών Α , Δ

και Θ το σημείο τομής των διχοτόμων των

γωνιών Β,Γ. Να αποδείξετε ότι

ΗΘ = (ΑΒ + ΓΔ) - (ΑΔ + ΒΓ)

. 2

Απόδειξη Είναι Α + Δ = 1 80° άρα

ΑΗΔ = 1 80° - (Α + Δ) = 1 80° - Α +Δ = 1 80" - 90° = 90° 2 2 2 Άρα το τρίγωνο ΑΗΔ είναι ορθογώνιο οπότε αν Ε το μέσον της ΑΔ τότε ΗΕ = ΑΔ = ΑΕ = ΕΔ από 2 Λ Λ ΕΗ=ΕΔ έχουμε ΕΗΔ = ΕΔΗ αλλά

Λ Δ Λ Λ Λ ΕΔΗ = - = ΑΔΓ άρα ΕΗΔ = ΗΔΓ οπότε ΗΕ//ΔΓ 2 Ομοίως αν Ζ είναι το μέσον της ΒΓ τότε

ΘΖ//ΔΓ και επειδή ΕΖ//ΔΓ θα είναι Ε, Η, Θ, Ζ συνευθειακά οπότε ΗΘ=ΕΖ-ΕΗ-ΘΖ=

Δ

ΑΒ + ΓΔ _ ΑΔ _ ΒΓ _ (ΑΒ + ΓΔ) - (ΑΔ + ΒΓ) 2 2 2 2 Α rrτ------�-

Σχ. 2 Γ

ΕΥΚΛΕΙΔΗΣ Β ' 74 τ.2/32

Page 35: Ευκλειδης Β 74

Μαθη ματικά για την Α ' Λυκείου

211 περίπτωση

Α ν Η είναι δεξιά του Θ τότε θα προκύψει ΕΖ=ΕΗ+ΘΖ-ΗΘ ή ΗΘ=ΕΗ+ΘΖ-ΕΖ ή

ΗΘ = (ΑΔ + ΒΓ) - (ΑΒ + ΓΔ) (γιατί;) . 2

Άσκη ση 3 '� Δίνεται τρίγωνο ΑΒΓ, Ι το έκκεντρο και ΑΖ

διχοτόμος της εξωτερικής γωνίας της Α και ΑΔ,

ΑΕ οι κάθετοι από το Α προς τις διχοτόμους

των εξωτερικών γωνιών Β και Γ αντίστοιχα. Να Λ Λ Λ αποδείξετε ότι ΒΙΓ = ΒΑΖ = ΔΑΕ .

Απόδε ιξη • ΒΙΓ = 1 800 - ( Β + Γ ] = 1 800 - 1 800 - Α = 2 2 2

Λ Λ = 1 80° - 90° + Α = 90° + Α

2 2 Λ Λ �ι Λ Β + Γ Λ 1 80° _ Α ΒΑΖ = Α + -- = Α +-- = Α +---2 2 2 Λ Λ Λ

900 Α 900 Α = Α + - - = + -2 2 � � Λ Λ ΔΑΕ = ΔΑΒ + Α + Γ ΑΕ = [ 90" - 82" ) + Α + [ 90" - f; ) =

Λ Λ = 1 80° + Α 1 800 + Α = 1 80° + Α - 90" - Α = 90" + Α

2 2 2 Λ Λ Λ Α άρα ΒΙΓ = ΒΑΖ = ΔΑΕ = 90u + - (Δες σχ.3 ) 2

z

Β Γ Σχ. 3

οποία τέμνει την ΑΓ στο Ε. Προεκτείνουμε την

ΑΚ κατά ΚΖ=ΑΚ, αν Μ, Η είναι τα μέσα των

ΒΓ, ΕΖ αντίστοιχα δείξτε ότι

(i) Το ΑΒΖΕ είναι ρόμβος

(ii) ΜΗ = 'γ -%1 , γ = ΑΒ, β = ΑΓ

(ί) Το ΑΚ είναι ύψος και διχοτόμος του τριγώνου ΑΒΕ οπότε αυτό είναι ισοσκελές και η ΑΚ θα είναι και διάμεσος δηλ. ΒΚ=ΚΕ οπότε το ΑΕΖΒ είναι ρόμβος αφού οι διαγώνιοί του διχοτομούνται και είναι κάθετοι. Α

z Σχ. 4

Γ

(ί ί) Αφού ΑΒΕ ισοσκελές ΑΕ=ΑΒ άρα ΕΓ = ΑΓ­ΑΕ = ΑΓ -ΑΒ = β -γ επίσης ΕΓ!/ΒΖ αφού ΑΕ//ΒΖ οπότε ΕΓΖΒ τραπέζιο και το ΜΗ είναι το ευθύγραμμο τμήμα που ενώνει τα μέσα των διαγωνίων του επομένως ΜΗ = ΒΖ - ΕΓ = ΑΒ - ΕΓ = γ - (β - γ) = γ - �

2 2 2 2 (Υποθέσαμε ΒΖ>ΕΓ<:=>γ>β-γ<=>2γ>β) αν στο

τραπέζιο είχαμε

ΜΗ = ΕΓ - ΒΖ = � - γ . 2 2

ίΒΖ < ΕΓJ γ < β - γ 2γ < β

τότε

Άf>κηση 5η Σε τραπέζιο ΑΒΓ Δ είναι Α = Δ = 90° ,

Γ Δ=2ΑΒ και Β = 3 Γ αν Ε μέσον της Γ Δ δείξτε

Ά σ κη ση 4η ότι: Δίνεται τρίγωνο ΑΒΓ και ΑΔ διχοτόμος της (ί) ΑΒΕΔ τετράγωνο

Α. Φέρουμε κάθετη στην διχοτόμο από το Β, η (ίί) ΒΔΓ ορθογώνιο και ισοσκελές

ΕΥΚΛΕΙΔΗΣ Β ' 74 τ.2/33

Page 36: Ευκλειδης Β 74

Μαθηματικά για την Α ' Λυκείου

Απόδειξη (i) Είναι B + f = l 80° αν f = ω τότε Β + f = 4ω

άρα 4ω= 1 80° οπότε 2ω=90°. Επίσης ΑΒΕΔ /I ορθογώνιο αφού ΑΒ = ΕΔ και Α = 90° ,

ΕΒΓ = Β - ΑΒΕ = 3ω - 90° = 3ω - 2ω = ω = Γ Δ επομένως Β Ε Γ ορθογώνιο και ισοσκελές άρα

ΒΕ=ΕΓ=ΔΕ άρα το ΑΒΕΔ είναι τετράγωνο.

( i i ) Το ΑΒΕΔ είναι τετράγωνο άρα ΑΕ .l ΒΔ αλλά ΑΕ//ΒΓ επομένως ΒΔ .l ΒΓ , επίσης ΒΕ μεσοκάθετος του ΔΓ. Άρα το ΒΔΓ είναι ισοσκελές τρίγωνο και έχει τη ΔΒΓ = 90° .

Α � _____ α ________ �Β

/ /

/ /

/ /

/ /

/

Δ

Α σ κη ση 611

/ /

/ /

/

/ /

/

Ε Σχ. 5

ω

Γ

Σε ορθογώνιο ΑΒΓ Δ είναι ΒΑΓ=30° αν η

κάθετος στην ΑΓ στο μέσον της Ο τέμνει την

ΑΒ και Γ Δ στα Ε και Ζ αντίστοιχα δείξτε ότι: 2

ΕΖ = - ΑΒ . 3

Α πόδ :: ιξη

Τα τρίγωνα ΑΟΕ και ΓΟΖ είναι ορθογώνια και έχουν γωνία 30° άρα ΟΕ = ΑΕ και ΟΖ = ΓΖ

2 2 Επίσης τα παραπάνω τρίγωνα είναι ίσα

(ΟΑ=ΟΓ) άρα ΓΖ=ΑΕ, θα είναι λοιπόν ΟΕ + ΟΖ = ΑΕ + ΓΖ � IEZ = AEI ( 1 ) 2 2

Σχ. 6

Επειδή ΑΒΓΔ ορθογώνιο άρα ΔΒ=ΑΓ και ΔΒ διέρχεται από το μέσον Ο της ΑΓ και επίσης ΑΟ=ΟΒ (μισά ίσων διαγώνιων) ΑΒΟ = ΟΑΒ = 30° , στο τρίγωνο ΟΒΓ είναι ΟΒΓ = orB = 60° άρα και η ΒΟΓ = 60° επομένως ΕΟΒ = ΕΟΓ - ΒΟΓ = 90° - 60° = 30° άρα ΕΟΒ τρίγωνο ισοσκελές ΕΒ=ΕΟ αλλά Εο ΑΕ , ΕΒ ΑΕ , = τ οποτε = τ η

ΑΕ 3ΑΕ 2 ΑΕ + ΕΒ = ΑΕ + - � ΑΒ = -- � ΑΕ = - ΑΒ 2 2 3 και τελικά ΕΖ = � ΑΒ . 3

ί\σκηση 7' � Δίνεται τρίγωνο ΑΒΓ και ΒΔ, ΓΕ διάμεσοί

του, αν Ζ, Η, Θ μέσα των ΒΔ, ΓΕ, ΒΓ να δείξετε

ότι η περίμετρος του ΑΒΓ είναι τετραπλάσια

της περιμέτρου του ΖΗΘ.

Λ.π6δειξη Τα Ε, Δ μέσα των ΑΒ, ΑΓ άρα ΕΔ//ΒΓ και

ΕΔ = ΒΓ επομένως το ΕΒΓ Δ είναι τραπέζιο και 2 το ΖΗ ενώνει τα μέσα των διαγωνίων του οπότε

ΒΓ - ΒΓ ΖΗ = ΒΓ - ΕΔ = 2 ΒΓ ή ΒΓ=4ΖΗ ( 1 )

του

Β

2 2 4

Α

Σχ. 7

Γ

Στο τρίγωνο ΒΔΓ τα Ζ, Θ είναι μέσα πλευρών ΑΓ

άρα ΖΘ = ΔΓ = 2 2 2 οπότε ΖΘ = ΑΓ

4 ή ΑΓ=4ΖΘ (2)

ΕΥΚΛΕΙΔΗΣ Β' 74 τ.2/34

Page 37: Ευκλειδης Β 74

Μαθη ματικά για την Α ' Λυκείου

Ομοίως στο τρίγωνο ΕΒΓ τα Η, Θ μέσα ΑΒ

πλευρών του άρα ΘΗ = ΕΒ = 2 = ΑΒ ή 2 2 4

ΑΒ=4ΘΗ οπότε από τις ( 1 ), (2), (3 ) έχουμε ΑΒ+ΒΓ+ΑΓ=4(ΘΗ+ΖΗ+ΖΘ)

λιc;κυ1σ j � § ' ! Από την κορυφή Α ενός τριγώνου ΑΒΓ

φέρουμε ΑΚ κάθετη στη διχοτόμο της γωνίας Β

και ΑΛ κάθετη στη διχοτόμο της εξωτερικής

γωνίας Β. Επίσης από το Α φέρουμε παράλληλη

προς τη ΒΓ που τέμνει τη διχοτόμο της γωνίας

Β στο Ε. Ν α δείξετε ότι το ΑΛΚΕ είναι

παραλληλόγραμμο.

Έχουμε ΑΛ .l ΒΛ και ΒΕ .l ΒΛ (αφού ΒΕ, ΒΛ διχοτόμοι εφεξής και παραπληρωματικών γωνιών) άρα ΑΛ // ΒΕ � IΑΛ Ι/ ΚΕ I ( 1 ) το ΑΛΒΚ είναι ορθογώνιο (3 ορθές) άρα ΔΒ=ΔΚ (μισά των ίσων διαγωνίων) οπότε ΔΒΚ = ΔΚΒ αλλά ΒΚ διχοτόμος άρα ΔΒΚ = κΒr επομένως ΔΚΒ = ΚΒΓ συνεπώς ΔΚ//ΒΓ και αφού ΑΕ//ΒΓ άρα IΛΚ // ΑΕ Ι (2)

Γ Σχ. 8

Από ( 1 ), (2) έχουμε ΑΛΚΕ παραλληλόγραμμο .

Λσκηση 9 ' 1 Δίνεται παραλληλόγραμμο ΑΒΓ Δ του

οποίου προεκτείνουμε την πλευρά ΔΓ κατά

ΓΕ=Γ Δ αν Ζ το μέσον του ΒΕ και Η το σημείο

τομής της ΔΖ με την ΑΓ, να δείξετε ότι:

ΑΗ=3ΗΓ

Είναι ΑΒΕΓ παραλληλόγραμμο άρα ΑΓ//ΒΕ και επομένως ΓΗ//ΖΕ επίσης είναι Γ μέσον της ΔΕ

άρα στο τρίγωνο ΔΖΕ αφού Γ μέσον ΔΕ και ΓΗ//ΖΕ θα είναι Η μέσον της ΔΖ και

ΒΕ ΗΓ = ΖΕ = 2 = ΒΕ , ΒΕ=ΑΓ άρα ΗΓ = ΑΓ

4 ή 2 2 4 ΑΓ=4ΗΓ ισοδύναμα ΑΓ-ΗΓ=4ΗΓ-ΗΓ<=:>ΑΗ=3ΗΓ.

(', !�� � ' � Δ Γ Ε Σχ. 9

ί''ΗΗΟ1 ση ll 01 1 Δίνεται ορθογώνιο τρίγωνο ΑΒΓ,

Α = 90° και Γ = 30° . Φέρουμε τη μεσοκάθετο

της ΑΓ που τέμνει τη ΒΓ στο Δ και τη διχοτόμο

της εξωτερικής γωνίας του Β στο Ε. Να δειχτεί

ότι ΑΒΕΔ είναι ρόμβος.

Απιiδηξη Έχουμε Γ = 30° και ΔΑ=ΔΓ άρα

ΔΑΓ = Γ = 30° οπότε ΒΑΔ = 90° - 30° = 60° επίσης Β = 90° - Γ = 60° επομένως το ΒΑΔ τρίγωνο είναι ισόπλευρο άρα ΑΒ=ΑΔ ( 1 )

'• . -----��---- � Γ Σχ. lΟ

Είναι ΑΒ .l ΑΓ, ΔΕ .l ΑΓ άρα ΑΒ//ΔΕ (2) ΔΒΕ =

Βεξ =

1 20ο = 60ο άρα 2 2 ΑΔΒ = ΔΒΕ οπότε ΑΔ//ΒΕ (3) από (2), (3 ) έχουμε ΑΒΕΔ παραλληλόγραμμο και αφού ισχύει η ( 1 ) είναι ΑΒΕΔ ρόμβος.

ΕΥΚΛΕΙΔΗΣ Β' 74 τ.2/35

Page 38: Ευκλειδης Β 74

���- Γ�--- -- Μαθ ηματ ι κά

γ ι α τη Β ' τάξ η του Λυκ ε ίου

Πολuιuνuμα κα ι πολυwνυμ ι κ έ ς ε ξ ι σώσε ι ς

Χρήστος Λαζαρίδης Η επίλυση πολυωνυμικών εξισώσεων, είναι ένα από τα παλαιότερα προβλήματα της Άλγεβρας. Το

πρόβλημα συνίσταται στην εύρεση τύπων, οι οποίοι να υπολογίζουν τις ρίζες της εξίσωσης ως συνάρτηση , των συντελεστών της.

'Ε , , , δ , -α ± �β2 - 4αγ , , ' ζ νας τετοιος τυπος ειναι ο ιασημος χ = , ο οποιος ως γνωστο εφαρμο εται για την 2α επίλυση της πολυωνυμικής εξίσωσης δευτέρου βαθμού, αχ2 + βχ + γ = Ο.

Η προσπάθεια για την εύρεση τύπων επίλυσης της πολυωνυμικής εξίσωσης τρίτου και τετάρτου βαθμού διήρκεσε αιώνες ! Το 1 452 δημοσιεύθηκε ο τύπος, που δίνει την λύση για την πολυωνυμική τρίτου βαθμού και μία μέθοδος μετατροπής πολυωνυμικής τετάρτου βαθμού σε τρίτου βαθμού.

Η επίλυση του προβλήματος για πολυωνυμικές βαθμού μεγαλύτερου του τετάρτου απασχόλησε μεγάλους Μαθηματικούς για 3 αιώνες. Το πρόβλημα όμως παρουσίαζε μεγάλες δυσκολίες. Εκφράστηκε μάλιστα η άποψη ότι επρόκειτο για ένα αδύνατο πρόβλημα;

Την τελική λύση έδωσε ο Eνariste Galois ( 1 8 Ι Ι- Ι 832), ένας ιδιοφυής, απείθαρχος και άτυχος νεαρός μαθηματικός. Ο Galois διατύπωσε κριτήρια, ώστε να γνωρίζουμε αν οι λύσεις μίας εξίσωσης μπορούσαν ή όχι να αναζητηθούν μέσω ριζών. Την προηγουμένη του θανάτου του, σε σε ένα κακογραμμένο χειρόγραφο 3 Ι σελίδων, έγραψε τις σκέψεις του. Το χειρόγραφο παρουσιάστηκε αρκετά χρόνια αργότερα ( 1 843) και του χάρισε την αθανασία.

Λ σ rc<ηση i Έστω το πολυώνυμο

Ρ(χ) = αχ3 + βχ2 + γχ + δ, α :;e 0 και το

πολυώνυμο Q(x) τέτοια ώστε:

Ρ( χ) = Q(x)(x2 - 1) .

α) Να αποδείξετε ότι: γ = -α και δ = -β. β) Να βρείτε το πολυώνυμο Q(x).

γ) Ν α υπολογίσετε το Ρ (-�) . , \ {Jση α) �χ)={; -ψΧχ)=>�χ)=( χ-1)( x+I)Qx) . Συμπεραίνουμε ότι το Ρ(χ) έχει παράγοντες το

χ-Ι και το χ+ Ι , άρα: { P(l) = O => { α + β + γ + δ = Ο => Ρ(- Ι) = Ο -α + β - γ + δ = Ο

=> {2β + 2δ = Ο => {γ = -α 2α + 2γ = 0 δ = -β

� �χ) = αχ3 + βχ2 - αχ - β =

= αχ(χ2 - Ι) + β(χ2 - l) = (x2 - Ι)(αχ + β) ( Ι ) . Το Q(x) ισούται με το πηλίκο της διαίρεσης Ρ(χ) : (χ2 - Ι ) , άρα Q(x) = αχ + β .

γ) Από τη σχέση ( Ι ) , {�) = (�: - 1}-β + β) = Ο

Ά σκηση 2 Το πολυώνυμο Ρ( χ) διαιρούμενο με το

χ2 + 2009 , δίνει πηλίκο π( χ) και υπόλοιπο υ(χ), έτσι ώστε: π( χ) = υ(χ).

α) Να εξετάσετε αν είναι δυνατόν το Ρ(χ) να είναι τετάρτου βαθμού.

β) Να αποδείξετε ότι: Ρ(Ο) = 2010υ(Ο). γ) Αν υ( χ) = χ + 1 , να λύσετε την εξίσωση

Ρ(χ) = Ο.

ΛίJση Ρ( χ) = (χ2 + 2009)π(χ) + υ( χ) =

= (χ2 + 2009)υ(χ) + υ( χ) = (χ2 + 20 10)υ(χ) ( Ι ) . α) Το υ(χ) θα είναι το μηδενικό πολυώνυμο ή θα είναι το πολύ πρώτου βαθμού.

ΕΥΚΛΕΙΔΗΣ Β ' 74 τ.2/36

Page 39: Ευκλειδης Β 74

------------- Μαθηματικά για την Β ' Λυκείου ------------Από τη σχέση ( 1 ) ο βαθμός του Ρ( χ) ισούται με το α) Το Ρ( χ) διαιρούμενο με το χ = χ-0, δίνει βαθμό του ( χ2 + 20 1 0 )υ( χ) , το οποίο είναι το πηλίκο -χ και υπόλοιπο Ρ(Ο), άρα,

Ρ(χ) = χ(-χ)+Ρ(Ο) => Ρ( χ) = -χ2 + Ρ(Ο) ( 1 ) . πολύ τρίτου βαθμού ή είναι το μηδενικό πολυώνυμο. Συμπεραίνουμε ότι το Ρ(χ) δεν είναι δυνατόν να είναι τετάρτου βαθμού. β) Η ( 1 ), για χ = Ο, γίνεται: Ρ(Ο) = 20 1 0υ(Ο). γ) Η εξίσωση ορίζεται στο R. Έχουμε :

( \ ) 2 Ρ( χ) = Ο<::>(χ + 20 1 0)(χ + 1) = Ο <:::> χ = - 1 .

Άσκψτη 3 α) Να βρείτε τη μορφή του πολυώνυμου

Ρ(χ) δευτέρου βαθμού, τέτοιο ώστε: P(x+l) - Ρ(χ) = 2χ+2, για κάθε χ Ε JR

β) Να υπολογίσετε το άθροισμα, • *

2+4+6+ . . . +2ν, ως συναρτηση του ν Ε Ν . γ) Να βρείτε το άθροισμα 1+2+3+ . . . +2009.

α) Έστω, Ρ( χ) = αχ2 + βχ + γ, α * Ο . Ρ( χ+ 1 ) - Ρ( χ) =

α( χ + 1)2 +β( χ + 1) +γ-αχ2 -βχ - γ = 2αχ + (α+β) . Έχουμε: 2αχ+(α+β)=2χ+2, για κάθε

x E JR <=>{α�;:

2 <=>{;:; .

άρα Ρ(χ) = χ2 + χ + γ, γ ε R . β) Η σχέση Ρ(χ+ 1 ) - Ρ(χ) = 2χ+2, αν χ = Ο,χ = 1 , χ = 2, . . . ,χ = ν-1 ,αντίστοιχα γίνεται:

P( l )-P(O) = 2, P(2)-P( l ) = 4, Ρ(3)-Ρ(2) = 6, . . . , Ρ(ν)-Ρ(ν- 1 ) = 2ν.

Με πρόσθεση κατά μέλη, προκύπτει: Ρ(ν)-Ρ(Ο) = 2+4+6+ . . . +2ν => => (ν2 + ν + γ) - (02 + 0 + γ) = 2 + 4 + 6 + . . + 2ν => ν2 + ν = 2 + 4 + 6 + . . . + 2ν => => 2 + 4 + 6 + . . . + 2ν = ν( ν + 1) γ) Από β) 2 + 4 + 6 + . . . + 2ν = ν(ν + 1) => 2(1 + 2 + 3 + . . . + ν) = ν( ν + 1) =>

ν( ν + Ι) Ι + 2 + 3 + . . . + ν = . 2

Η τελευταία σχέση για ν = 2009, . 2009 .20 Ι Ο δινει: Ι +2+3+ . . . +2009 = = 2.0 1 9 .045 .

2 ί\ σκηση :j Το πολυώνυμο Ρ(χ) διαιρούμενο με το χ,

δίνει πηλίκο -χ και διαιρούμενο με το χ2 -2 αφήνει υπόλοιπο -1 .

α) Να αποδείξετε ότι: Ρ(χ) = - χ2 +1 . β) Ν α λύσετε την εξίσωση :

��Ρ(ημχ) + ι = J2 (Ι).

Αν π(χ) το πηλίκο της διαίρεσης Ρ(χ) : (χ2 -2), τότε, Ρ(χ) = (χ2 - 2) π(χ) + (- Ι) , για

κάθε χ Ε JR (2). Οι ( Ι ),(2) αν θέσουμε όπου χ το J2 , αντίστοιχα γίνονται: Ρ( J2) = -2 + Ρ( Ο), Ρ( J2) = - 1 , επομένως -2 + Ρ( Ο) = - Ι => Ρ( Ο) = Ι . Από ( 1 ) , προκύπτει: Ρ( χ) = -χ2 + 1 . β) Ρ(ημχ) = -ημ2χ + 1 = συν2χ . Η εξίσωση (1), ορίζεται στο R και:

( Ι ) <:::> �� συν2χ + 1 = J2 <:::> Jlσυνχ l + Ι = J2 <=> <:::> !συνχ l + 1 = 2 <:::> !συνχ l = 1

<:::> συν2χ = Ι <:::> Ο = 1 - συν2χ <:::> ημ2χ = Ο <:::> <:::> ημχ = 0 <:::> ( Χ = kπ, k Ε Ζ)

α) Έστω το πολυώνυμο Ρ(χ) = χ3 +αχ+β,β =F Ο .

ί) Αν το πολυώνυμο έχει διπλή ρίζα το λ Ε R , να αποδείξετε ότι: α = -3λ2 , β = 2λ3 και να εξετάσετε αν ισχύει το αντίστροφο.

ίί) Να βρείτε την άλλη ρίζα του Ρ(χ) και να εξετάσετε αν είναι δυνατόν το λ να είναι τριπλή ρίζα.

β) Να λύσετε την ανίσωση

χ3 - 3.20092 χ + 2.20093 � ο (1).

α) ι Το Ρ( χ) θα έχει παράγοντα το χ-λ. Ι ο α β λ

λ λ2 αλ+λ3 Ι λ α+ λ2 αλ+λ3+β Το υπόλοιπο της διαίρεσης Ρ( χ): (χ-λ), είναι

αλ+λ3+β. Πρέπει: αλ+λ3+β = Ο ( 1 ) Το πηλίκο της διαίρεσης Ρ ( χ) : (χ-λ), είναι

χ2 +λχ+(α+λ2) . Το πηλίκο πρέπει να έχει παράγοντα το χ-λ,

άρα λ2 +λ.λ+α+λ2 = Ο=> α = -3λ2 . Από (1) : -3λ3 + λ3 +β = Ο=> β = 2 λ3 . Αν α = -3λ2 και β = 2λ3 , τότε,

Ρ( χ) = χ3 - 3λ2χ2 + 2λ3 . Εφαρμόζοντας το σχήμα Homer, διαπιστώνουμε ότι το λ είναι διπλή ρίζα, επομένως το αντίστροφο ισχύει.

ii) Ρ( χ) = ( χ - λ )2 (χ + 2λ) . Η άλλη ρίζα είναι χ = -2λ. Έστω ότι λ είναι τριπλή ρίζα, τότε: -2λ=λ=> λ = Ο=> β = 2.03 => β = Ο, άτοπο. Άρα το λ δεν είναι δυνατόν να είναι τριπλή ρίζα. β) Αν θεωρήσουμε λ = 2009, τότε το πολυώνυμο

ΕΥΚΛΕΙΔΗΣ Β ' 74 τ.2/37

Page 40: Ευκλειδης Β 74

------------ Μαθηματικά για την Β ' Λυκείου-----------­χ3 - 3 .20092 χ + 2 .20093 είναι το Ρ( χ) με α = -3λ2 και β = 2 λ3 . Από το α) το πολυώνυμο θα έχει ρίζες λ = 2009 (διπλή) και το -2λ = -40 1 8 . Η aνίσωση (Ι) ορίζεται στο R και:

(χ - 2009)2 (χ + 40 1 8) 2: ο� χ 2: -40 1 8 ο

α) Το πολυώνυμο Ρ(χ) διαιρούμενο με τα x­l ,x-2 δίνει αντίστοιχα πηλίκα π1 (χ), π2(χ). Να αποδείξετε ότι: π1 (2)= π2( 1 ) = Ρ(2)- P(l).

β) Το πολυώνυμο Ρ(χ) διαιρούμενο με το χ-1 , δίνει πηλίκο χ2+2χ+3 ενώ διαιρούμενο με το χ-2 αφήνει υπόλοιπο 1 5.Να υπολογίσετε το πολυώνυμο Ρ(χ).

γ) Να λύσετε την εξίσωση JP(x) = χ - 1 ,

όπου Ρ(χ) το πολυώνυμο, που βρήκατε στο β) ερώτημα.

α) Ρ( χ) = (χ - 1)π1 (χ) + P(l) , για κάθε χ Ε JR

=> Ρ(2) = Π ι (2)+Ρ( l ) => Π ι (2) = Ρ(2) -Ρ( 1 ) . Ρ( χ) = (χ - 2)π2 (χ) + Ρ(2) , για κάθε χ Ε JR

=> P( l ) = -π2( 1 )+Ρ(2) => π2( l )=P(2)-P( l ) . Από τις σχέσεις προκύπτει, πι (2)= π2( 1 ) = Ρ(2)- P( l ) . β) Από το α) , αν θεωρήσουμε, π1 (χ) = χ2 + 2χ + 3 και Ρ(2) = 1 5 , έχουμε :

π ι (2) = Ρ(2)- Ρ( 1 ) => 1 1 = 1 5-P( l ) => P( l ) = 4. Ρ( χ) = (χ - 1)π1 (χ) + P(l) = (χ - 1)(χ2 + 2χ + 3) + 4 = χ3 + χ2 + χ + 1 . γ) Ρ(χ) = χ3 + χ2 + χ + 1 = (x + l)(x2 + 1) . Η εξίσωση ορίζεται μόνο όταν χ2:-1 και είναι ισοδύναμη με την: �(χ2 + 1)(χ + 1) = χ - 1 (Ι) .

( ! ) <c> ι + 1)(:2 �\�� (χ - !)2 <c> { χ3 : :χι= Ο { χ 2: 1 { χ 2: 1 � 2

� αδύνατη . χ(χ + 3) = ο χ = ο

Το πολυώνυμο Ρ(χ) διαιρούμενο με το χ3+ 1 , αφήνει υπόλοιπο -χ2-2χ-1 . Θεωρούμε το πολυώνυμο Q(x)=P(x)+(x+1)2•

α) Να υπολογίσετε το υπόλοιπο της διαίρεσης του Ρ(χ) δια του x+l .

β) Να αποδείξετε ότι η διαίρεση Q(x) : (x2-x+l) είναι τέλεια.

γ) Αν το Q(x), έχει παράγοντα το χ-1 , τότε το υπόλοιπο της διαίρεσης Ρ(χ) : (χ-1 ), ισούται με -4.

α) Ρ(χ) = (χ3 + 1)π(χ) + (-χ2 - 2χ - 1) , για κάθε χ Ε JR . Το υπόλοιπο της διαίρεσης είναι,

Ρ(- 1 ) = -1 +2-1 = Ο . β) Q(x) = Ρ(χ)+(χ+ 1 )2 = (χ3 + 1)π(χ) + (-χ2 - 2x - l) + (x + l)2 = (χ3 + l)π(χ) = (χ + 1)(χ2 - χ + 1)π(χ) ,

άρα η διαίρεση Q(x) : (χ2-χ+ 1 ) είναι τέλεια. γ) Q( l ) = Ο => Ρ( 1 )+( 1 + 1 / = Ο =>P( l )+4 = Ο

=>Ρ( 1 ) = -4, άρα το υπόλοιπο της διαίρεσης Ρ(χ) : (χ- 1 ), ισούται με -4.

Το πολυώνυμο �χ)=χ4+� +βχ+γ,γ>Ο, έχει

παράγοντα το χ2-1 . α) Να αποδείξετε ότι: γ+α+ 1 = Ο και β = Ο. β) Να αποδείξετε ότι το πολυώνυμο Ρ(χ) έχει

παράγοντα το χ2-γ. γ) Αν το πολυώνυμο Ρ(χ) έχει ρίζα το

χ = J2 ,να υπολογίσετε το πολυώνυμο Ρ(χ).

α) Το Ρ( χ) έχει παράγοντα το χ2-1=(χ+ 1 )(χ- 1 ) , άρα { P(l) = O => { 1 + α + β + γ = Ο => Ρ(- 1) = 0 1 + α - β + γ = Ο

=> { 1 + α + β + γ = Ο=> {α + γ + 1 = Ο 2β = ο β = ο

β) Ρ(χ) = χ4 + αχ2 + γ ,γ > 0 . Έχουμε: � ±fr) =I +αy+γ=){γ+α+1)=γ.0=0. Το Ρ(χ) έχει παράγοντες το χ -JY και το

x+fr , με [γ 1:--[γ, σφ;Jύγ>Ο άρα θα έχει και το

(χ-{γ) (χ+JΥ)=χ2-γ, διότι αν το Ρ(χ) διαιρείται με χ-μ και χ-ν με μ1:-ν τοτε διαιρείται και με ( χ - μ) ( χ - ν) . Πράγματι Ρ ( χ ) = ( χ - μ)Q ( χ ) , για κάθε χ Ε IR => Ρ( ν) = ( ν - μ)Q(ν) => 0 = ( ν - μ)Q ( ν)

v;t μ => Q (ν ) = Ο => Q (χ ) = ( χ - ν) Π (χ ) για κάθε χ εΙR=>Ρ( χ) =( χ -μ)( χ -ν)Π( χ) , για κάθε χ ε JR .

γ) Η ρίζα χ = J2 ,θα είναι η JΎ , άρα γ=2 . Από τη σχέση γ+α+ 1 = Ο, προκύπτει

α = -3 άρα Ρ(χ) = χ4 - 3χ2 + 2 .

:\ φ :ςη ση I) Το πολυώνυμο Ρ(χ) διαιρούμενο με τα

ΕΥΚΛΕΙΔΗΣ Β' 74 τ.2/38

Page 41: Ευκλειδης Β 74

------------ Μαθηματικά για την Β ' Λυκείου -----------­(χ-1 )(χ+1) δίνει υπόλοιπο υ(χ).

αντίστοιχα πηλίκο π(χ) και

α) Να αποδείξετε ότι:

υ(χ) = Ρ(1) - Ρ(-1) χ +

Ρ(1) + Ρ(-1).

2 2 β) Να υπολογίσετε το υπόλοιπο

διαίρεσης του πολυωνύμου

χ2010 _ χ2009 + χ2008 _ 1 με το χ2 _ 1 .

της

α) Ο διαιρέτης (χ-Ι )(χ+ Ι ) = χ2 - Ι είναι δευτέρου βαθμού, άρα το υπόλοιπο υ(χ) θα είναι το πολύ πρώτου βαθμού ή θα είναι το μηδενικό πολυώνυμο.

Έστω υ(χ) = αχ+β, α, β Ε R , τότε: Ρ( χ) = (x- l )(x+ Ι ) π(χ)+ (αχ+β), για κάθε

x E IR ( 1 ) . Από την ( 1 ), για χ = Ι , χ = - Ι , έχουμε:

{ Ρ(Ι) = α+β {Ρ(Ι) + Ρ(-Ι) = 2β - 2 {α- P(l) -P(-I)

P(-1) =--il+β � Ρ(Ι) -Ρ(-Ι) = 2α � β P(l) +2P(-I)

Ά ( ) P(l) - P(-I) P(l) + P(- I) ρα, υ χ = χ + . 2 2

β) Θέτουμε Ρ( χ) = χ20 1 0 _ χ2009 + χ2008 - Ι . Από α), το υπόλοιπο της διαίρεσης είναι:

υ( χ) __ P(l) - Ρ( -Ι) χ + P(l) + Ρ( - Ι) 2 2

P( l ) = Ο,Ρ(- 1 ) = 2 .

-χ+ Ι, διότι,

α) Να λύσετε την εξίσωση 2y3-3y+l=O. β) Με τη βοήθεια του α), να λύσετε την

εξίσωση, 8χ3 -6χ+ .fi = Ο.

α) Με τη βοήθεια του σχήματος Homer, 2y3 -3y+ Ι = Ο<=> (y-1 )(2/+2y-I ) = Ο<=>

( Ι , - 1 +J3 , - 1 -J3) y = η y = η y = . 2 2

β) 8χ3 -6χ+ J2 =Ο<=> 8χ3 6χ [4χ3 ) ( 3χ ) fi -fi + Ι = 0<=>2 fi -2 fi + Ι = Ο<=>

z [ 4�'' ) -{ J�x ) + I � O<c:>4J2x3 - 3J2x + l � ο

� 3 � { y = xJ2 <=> 2(χν L ) - 3(χν 2 ) + Ι = Ο <=>

2y3 - 3y + Ι = Ο

!;; (xfi = Ι ή xfi = -Ι +13 ή xfi -Ι -13)<=> 2 2

(χ = J2 ή χ = -J2-J6 ή χ = -J2 + J6 ) . 2 4 4

;\σκψιη � � Το πολυώνυμο Ρ(χ) διαιρούμενο με τα x,x-

.fi , χ+ .fi δίνει αντίστοιχα υπόλοιπα 1 , .fi , -.fi .

α) Να υπολογίσετε το υπόλοιπο της διαίρεσης του Ρ( χ) με το χ3 -2χ.

β) Να αποδείξετε ότι το πολυώνυμο

υ { υ ( χ)) - χ , έχει παράγοντα το χ2-2 . . \ \Jση α) Το υπόλοιπο υ(χ) θα είναι το πολύ δευτέρου

βαθμού ή θα είναι το μηδενικό πολυώνυμο. Έστω υ(χ)=αχ2+βχ+γ, α, β, γ Ε R , τότε : Ρ(χ)=(χ3-2χ)π(χ)+υ(χ)= =(x-J2 )(x+J2 )π(χ)+αχ2+βχ+γ, για κάθε χ Ε IR ( I ) .

Από υπόθεση , Ρ (Ο) = Ι ,Ρ( J2 ) = J2 και P(­J2 ) = -J2 . Από την ( Ι ) , αν θέσουμε, όπου χ = Ο, J2 ,-J2 αντίστοιχα έχουμει:

Ρ( J2) = J2 � 2α + βJ2 + γ = J2 =>

{ Ρ( Ο) = I { γ = Ι

Ρ( -J2) = -J2 2α - βJ2 + γ = -J2 { γ = 1 => 4α + 2γ = 0

� 2α + βJ2 + γ =J2

Άρα, υ( χ) = _ _!_χ2 + χ + 1 . 2

Ι α = --2

β = I γ = I

β) Ρ( χ) = (χ3 -2χ)π(χ)+υ(χ), για κάθε χ Ε IR . Άρα: Ρ( J2 ) = υ( J2 ) = J2 και P(-J2 ) = υ(-J2 ) = -J2 . Θέτουμε Q(x) = υ ( υ ( χ )) - χ .

Q( J2 ) = υ(υ( J2 ))-J2 =J2 -J2 = Ο. Q(-J2 ) = υ(υ(-J2 ))+J2 =-J2 +J2 = Ο και J2 * -J2 . Άρα, το Q(x), έχει παράγοντα το

(χ -J2 )(x+J2 ) = χ2 -2 .

\ σ"ηση 1 2 Δίνονται τα πολυώνυμα P(x)=(k-1)x3 +px2

+mx+n, Q(x) = χ2 -1 , k,p,m,n E R . Το πολυώνυμο P(x).Q(x) είναι πέμπτου

βαθμού και το Ρ( χ) έχει παράγοντα το Q(x). α) Να εξετάσετε αν είναι δυνατόν k = 1 και

να αποδείξετε ότι η = -p, k-1 = -m. β) Να εξετάσετε αν είναι δυνατόν m=O και

ΕΥΚΛΕΙΔΗΣ Β ' 74 τ.2/39

Page 42: Ευκλειδης Β 74

------------ Μαθηματικά για την Β ' Λυκείου ------------

να λύσετε την εξίσωση Ρ(χ) = Ο (1). Να βρείτε το υπόλοιπο της διαίρεσης Ρ(χ) : (χ2 γ) Αν το πηλίκο της διαίρεσης P(x) :Q(x) -2 ) .

ισούται με χ-3, να αποδείξετε ότι: Ρ(χ) = χ3 -3χ2

-χ+3.

α) Το πολυώνυμο P(x).Q(x) είναι πέμπτου βαθμού και το Q(x) είναι δευτέρου βαθμού, άρα το Ρ(χ) θα είναι τρίτου βαθμού. Συμπεραίνουμε ότι: k - 1 :;t 0 => k :;t l .

Το Ρ(χ) έχει παράγοντα το Q(x) = χ2 - Ι = (χ- Ι )(χ+ 1 ) , άρα, { P(l) = Ο { k - Ι + p + m + η = Ο

Ρ( -1) = Ο=>

-(k - 1) + p - m + η = Ο=> { 2p + 2η = Ο { η = -p

=> 2(k - 1) + 2m = 0 => k - 1 = -m β) Έστω m = Ο. Τότε: k - 1 = -m => k - 1 = Ο

=> k = Ι , άτοπο . Άρα, m :;t Ο , οπότε. (Ι) � -mx3 +px2 +mx-p = Ο � -mx(x2 - l )+p(x2 -1 )=0

� (p-mx)(x2 - I )=O� (χ=- Ι ή x= l ή χ=..Ε_ ). m

γ) Ρ(χ) = Q(x)(x-3) = (χ2 -Ι )(χ-3) = =χ3 -3χ2 -χ+3 .

1 ) Έστω το πολυώνυμο R:χ)=�+�-5χ+4,α,βεR. Το Ρ(χ) διαιρούμενο με χ+2,χ-1 δίνει αντίστοιχα υπόλοιπα 6,2.

2) α) Να αποδείξετε ότι: α = � και β = � . 3 3

3) β) Να λύσετε την εξίσωση Ρ(χ)-6 = Ο. 4) γ) Να λύσετε την ανίσωση Ρ( χ) :::=: 2 . 5) Το πολυώνυμο

Ρ( χ) = αχ4 + (α - Ι 6)χ3 - αχ2 + α - 1 3, α ε R ,

, . Ι εχει παραγοντα το χ - - . 2

6) α) Να αποδείξετε ότι: α = 1 6 . 7) β) Να λύσετε την ανίσωση Ρ( χ) < Ο. 8) γ) Να λύσετε την εξίσωση Ρ(συνχ) = Ο. 9) α) Έστω το πολυώνυμο Ρ(χ). Να αποδείξετε

ότι το υπόλοιπο της διαίρεσης Ρ(χ) : (χ2 -k2 ) , όπου k :;t Ο , είναι:

υ( χ) = P(k) - Ρ( -k) χ + P(k) + Ρ( -k) . 2 2 Ι Ο) β) Έστω το πολυώνυμο Ρ( χ) = 3χ6 -4χ4+5χ2+ 1 .

1 1 ) α) Να λύσετε την εξίσωση 2χ3 -4χ2 +χ+ 1 = Ο. 1 2) β) Με τη βοήθεια του α), να λύσετε την

εξίσωση, 1 8/ -1 2J3 /+3y+J3 = Ο. 1 3) α) Να υπολογίσετε πολυώνυμο Ρ(χ) δευτέρου

βαθμού, τέτοιο ώστε: Ι 4) Ρ(χ+ 1 ) - Ρ(χ) = χ.

* 1 5) β) Να βρείτε το άθροισμα 1+2+ 3+ . . +ν, ν ε Ν .

1 6) Το πολυώνυμο Ρ(χ) =χ3 +αχ2 +βχ+γ,α,β,γ εR , έχει παράγοντα το (χ - 1)2 .

1 7) α) Να βρείτε την τρίτη ρίζα του Ρ(χ),ως συνάρτηση του α.

Ι 8) β) Α ν Ρ( -20 1 1 ) = Ο, να βρείτε το πολυώνυμο Ρ( χ).

1 9) γ) Αν Ρ(χ), το πολυώνυμο που βρήκατε στο β), να λύσετε την εξίσωση JP(x) = j x - l j .

1 )

2)

3)

4)

5)

Λ Π Α Ν Τ Η Σ Ε Ι Σ

β) (χ + 2)(� χ2 - 2χ - 1) = 0 3

γ) (χ - 1 )( � χ 2 + 3 χ - 2) ;:::: ο 3

J3 1 , 1 J3 β) -- < Χ < --η- < χ <-2 2 2 2

J3 1 γ) συνχ = ± - , ± -2 2

β) υ(χ) = 1 9

) Ι ' 1 ±J3 α χ = η χ = -- . 2

β) 1 , I ± J3 y = J3 η y = 2J3

1 2 1 α) Ρ( χ) = 2 χ -2 χ + γ, γ ε R

β) ν(ν + 1) 2

6) α) χ = -α -2 402 1 χ+20 1 1

β) Ρ(χ) = χ3 +2009χ2 -γ) χ = 1 ή χ =-20 1 1 .

ΕΥΚΛΕΙΔΗΣ Β ' 74 τ.2/40

Page 43: Ευκλειδης Β 74

------------- Μαθηματικά για την Β' Λυκείου -------------

Εμ�αδά των Ευθυγράμμων Σχημάτων Γιώργος Τσαπακίδης

Α. Π ερ ι ληπτ ι κή Θεωρ ία Το εμβαδόν ενός σχήματος είναι ο θετικός αριθμός, που μας λέει πόσες φορές πρέπει να

επαναλάβουμε την επιφάνεια ενός τετραγώνου, που έχει πλευρά ίση με το μοναδιαίο ευθύγραμμο τμήμα, για να πάρουμε την επιφάνεια του σχήματος.

• Τύποι ψβαδών.

·D · ,@-·'

l Ε= ι /2αυα= ι /2βυβ= ι /2yυ,

β L..L..--[ _] Ε•οβ

α β + Β E= - · U

2 .. Β

Ε= ι /2βγημΑ= ι /2γαημΒ= ι /2αβημΓ

�----. • γ{\ Ε= -:�-γ Ε= �'τ (;-τ-- α-;)-;-( τ---::-β-;-:)(,-τ --γ-::-) ε�"� � ε-..,iμ«ρος.ρ Ε= ι /2αυα= ι /2βυβ= ι /2yυ,

Χρήσι μες Προτάσε ι ς • Εμβαδά και λόγοι

Α

Αν ΑΔ=Α ' Δ ' , τότε: (ΑΒΓ ) (ΑΈ 'Γ)

Γ ΒΓ

Β 'Γ

ΑΔ Αν ΒΓ=Β 'Γ , τότε: -:-'('--ΑΒ

---'Γ )'-:- =

(Α Έ 'Γ) Α'Δ '

Γ

Αν Α Β Γ ::::: Α 'Β Τ ' , τότε: ((ΑΒΓ )

) =(-Β

_Γ )2 Α Έ Γ Β 'Γ

(ΑΒΓ) _ ΑΒ · ΑΓ

(ΑΉΓ) -ΑΉ' · ΑΓ

Β. Τα Εμβαδά στην Αποδεικτική Διαδικασία Με τη βοήθεια των εμβαδών αποδεικνύονται, με

απλό τρόπο, αρκετά γνωστά θεωρήματα, όπως τα: I " Θεώρημα Διχοτόμων : Σε κάθε τρίγωνο η

εσωτερική (εξωτερική) διχοτόμος μιας γωνίας του χωρίζει εσωτερικά (εξωτερικά) την απέναντι της γωνίας πλευρά σε μέρη ανάλογα προσκειμένων των πλευρών του τριγώνου.

Απόδειξη • Έστω ΑΔ η διχοτόμος της γωνίας Α του

τριγώνου ΑΒΓ, ΑΗ το ύψος του και ΔΕ, ΔΖ οι αποστάσεις του Δ από τις πλευρές ΑΒ και ΑΓ αντίστοιχα.

Α

Β

ΕΥΚΛΕΙΔΗΣ Β ' 74 τ.2/41

Page 44: Ευκλειδης Β 74

-------------- Μαθηματικά για την Β ' Λυκείου ------------­Είναι: ( ΑΒΔ)

(ΑΓΔ)

1 - ΒΔ · ΑΗ 2 1 -ΔΓ · ΑΗ 2

(ΑΒΔ) (ΑΓΔ)

�ΑΒ · ΔΕ ΑΒ -=---- = _!_ΑΓ · ΔΖ ΑΓ 2

ΒΔ ΔΓ ( 1 )

(2)

( αφού κάθε σημείο της διχοτόμου ισαπέχει από τις πλευρές της γωνίας) .

Από τις ( 1 ) και (2) παίρνουμε : ΒΔ = ΑΒ ΔΓ ΑΓ

Έστω ΑΔ ' η εξωτερική διχοτόμος του τριγώνου ΑΒΓ, ΑΗ το ύψος του και ΔΈ, Δ 'Ζ οι αποστάσεις του Δ ' από τις ευθείες ΑΒ και ΑΓ αντίστοιχα.

��--�-- ::__- -- -Δ ' ' ... ... ... ... ... ... �Β Η Γ

( ΑΒΔ') _ �ΒΔ' . ΑΗ __ ΒΔ' Έχουμε : ...:....__..:.... (ΑΓΔ') ..!_ ΓΔ' · ΑΗ ΓΔ' 2

(ΑΒΔ ') - �ΑΒ · Δ 'Ε ΑΒ (ΑΒΓ ') _!_ΑΓ · Δ 'Ζ ΑΓ

2

( 1 )

(2)

Από τις ( 1 ) και (2) παίρνουμε: ΒΔ ' = ΑΒ ΓΔ ' ΑΓ

�' " ' ':' ' ' χ · · . · ;: : : · >: � · : ·•> ' i :λ<:ε .μ η : Αν ευθεία ε τέμνει τις πλευρές ή τις προεκτάσεις των πλευρών ΑΒ, ΒΓ και Γ Α του τριγώνου ΑΒΓ στα

Δ, Ε, και Ζ αντίστοιχα, τότε ισχύει:

ΔΑ . ΕΒ . ΖΓ = 1

ΔΒ ΕΓ ΖΑ

Θα εφαρμόσουμε την πρόταση : Αν μια γωνιά ενός τριγώνου είναι ίση ή παραπληρωματική με μια γωνιά ενός άλλου τριγώνου, τότε ο λόγος των εμβαδών των δύο τριγώνων ισούται με το λόγο των πλευρών που περιέχουν τις ίσες ή παραπληρωματικές γωνίες.

Α

Τα τρίγωνα ΑΔΖ και ΔΒΕ έχουν Μ Ζ + ΕΔΒ = 1 80° , άρα �=� ::: ( 1 )

Τα τρίγωνα ΔΒΕ και ΕΓΖ έχουν την Ε κοινή άρα ( ΔΒΕ) _ ΔΕ . ΒΕ (2) (ΕΓΖ) - ΕΓ · ΕΖ Τα τρίγωνα ΑΔΖ και ΕΓΖ έχουν ΕΖΓ = ΑΖΔ , άρα (ΕΓΖ) _ ΕΖ · ΖΓ (3)

(ΑΔΖ) ΖΑ · ΔΖ Πολλαπλασιάζουμε κατά μέλη τις ( 1 ) , (2), (3)

και παίρνουμε: ( ΑΔΖ)( ΔΒΕ)(ΕΓΖ) (ΔΒΕ){ ΕΓΖ) (ΑΔΖ)

ΔΑ · ΔΖ · ΔΕ · ΕΒ · ΕΖ· ΖΓ ΔΑ · ΕΒ · ΖΓ --------- => 1 = . ΒΔ · ΔΕ · ΕΓ · ΕΖ · ΖΑ · ΔΖ ΔΒ · ΕΓ · ΖΑ 11 <'7 '

Ο Μενέλαος ήταν Έλληνας μαθηματικός και aστρονόμος που έζησε κυρίως στην Αλεξάνδρεια μεταξύ του Ι ου και 2°υ μ.Χ αιώνα. Για ένα διάστημα έζησε και στη Ρώμη . Το σπουδαιότερο έργο του Μενελάου είναι τα «Σφαιρικά», που αναφέρονται στις ιδιότητες των σφαιρικών τριγώνων. δηλαδή τριγώνων που ορίζονται στην επιφάνεια μιας σφαίρας από τρεις μέγιστους κύκλους της. Το διάσημο θεώρημα του Μενελάου δεν αποδεικνύεται στα «Σφαιρικά», αλλά χρησιμοποιείται για την απόδειξη άλλου θεωρήματος. Είναι ενδεχόμενο να υπήρχε στο χαμένο έργο του «Περί τριγώνων».

1 ' Αν Ρ είναι εσωτερικό σημείο του τριγώνου ΑΒΓ και οι ΑΡ, ΒΡ, ΓΡ τέμνουν τις ΒΓ, Γ Α και ΑΒ στα Δ, Ε και

Z , , ΒΔ ΓΕ ΑΖ 1 αντιστοιχα, τοτε: - · - · - = ΔΓ ΕΑ ΖΒ

Δ Αν ΒΒ 'ύψος του Α Ρ Β και ΓΓ 'ύψος του Δ

ΑΡ Γ έχουμε : ΒΔ = ΒΒ ' (από Β Β 'Δ � rr ·Δ) =

(ΑΡΒ) ΔΓ ΓΓ (ΑΡΓ) ( αφού τα τρίγωνα αυτά έχουν την ίδια βάση ΑΡ).

ΕΥΚΛΕΙΔΗΣ Β' 74 τ.2/42

Page 45: Ευκλειδης Β 74

-------------- Μαθηματικά για την Β ' Λυκείου -------------Α

Γ

Έτσι έχουμε: ΒΔ ΓΕ ΑΖ (ΑΡΒ) (ΒΡΓ) ( ΓΡΑ) - · - · - = . . = 1 ΔΓ ΕΑ ΖΒ (ΑΡΓ) (ΒΡΑ) ( ΓΡΒ)

Ο Giovanni Ceva (1648-1734) ήταν Ιταλός γεωμέτρης, το κυριότερο έργο του, στο οποίο περιέχεται το «θεώρημα του Ceνa» ήταν το «De linies rectis se inνicem secantibus statica construction» που εφαρμόζει τη θεωρία των κέντρων βάρους στην απόδειξη γεωμετρικών θεμάτων.

4" Θ α;) ρ η μ α του nω A ιι !J c l : Αν Ρ είναι το εσωτερικό σημείο του τριγώνου ΑΒΓ και οι ΑΡ, ΒΡ, ΓΡ τέμνουν τις πλευρές ΒΓ, ΓΑ και ΑΒ στα Δ, Ε και Ζ αντίστοιχα, τότε ισχύει: ΑΡ ΑΖ ΑΕ - = - +-ΡΔ ΖΒ ΑΓ

Α πό δε ιξη Όπως είδαμε στην απόδειξη του Θεωρήματος

του Ceνa, έχουμε: Α

Β

ΑΖ ΑΕ (ΑΡΓ) (ΑΡΒ) - + - =--- + = ΖΒ ΕΓ (ΒΡΓ ) (ΒΡΓ)

Γ

_ (ΑΡΓ) + (ΑΡΒ) _ (ΑΒΓ) - (ΒΡΓ) _ - (ΒΡΓ) - (ΒΡΓ) -

( ΑΒΓ) ( ΒΡΓ) ( ΑΒΓ) =-- --- =-- -Ι = (ΒΡΓ) (ΒΡΓ) (ΒΡΓ)

Ι - ΒΓ · ΑΗ 2 = ΑΗ - Ι = _!_ ΒΓ · ΡΘ ΡΘ 2

ΑΔ ( Δ Δ ) ΑΔ - ΡΔ ΑΡ = Ρ Δ - Ι από τα Α Η Δ ;:::: Ρ Θ Δ = Ρ Δ = Ρ Δ

5" Θεώ ρη μ α του \f i ' i n n i : Το άθροισμα των αποστάσεων τυχαίου εσωτερικού σημείου ισόπλευρου τριγώνου από τις πλευρές του είναι σταθερό.

Απόδειξη Έστω Ρ εσωτερικό σημείο του ισόπλευρου

τριγώνου ΑΒΓ πλευράς α και Ρ Δ, ΡΕ και ΡΖ οι αποστάσεις του από τις πλευρές ΒΓ, ΓΑ και ΑΒ αντίστοιχα. Α

Β Γ Είναι: ( ΡΒΓ) + (Ρ ΑΓ) + (Ρ ΑΒ) = ( ΑΒΓ) �

Ι Ι Ι Ι � -α · ΡΔ + -α · ΡΕ + -α · ΡΖ = -α · υ � 2 2 2 2

� ΡΔ + ΡΕ + ΡΖ = υ (υ το ύψος του ισοπλεύρου τριγώνου )=σταθερό.

Π αρ ατi1 ρ η σ η : Η πρόταση αυτή εύκολα γενικεύεται για οποιοδήποτε κανονικό πολύγωνο.

Ι στο ρ ι κό

Ο Vincenzo Viviani (1622-1 703) ήταν Ιταλός μαθηματικός και μηχανικός, που από τα 1 7 του χρόνια έγινε μαθητής, γραμματέας και βοηθός του Γαλιλαίου. Μελέτησε τους aρχαίους Έλληνες γεωμέτρες και έγραψε σχόλια στα «Κωνικά» του Απολλωνίου και τα

«Στοιχεία» του Ευκλείδη .

6" Θ ι:ιο ρη μ α Π τολ::μυ ίου Σε κάθε εγγεγραμμένο τετράπλευρο ΑΒΓ Δ ισχύει:

ΑΓ ΑΒ · ΑΔ + ΓΒ · ΓΔ = -------ΒΔ ΒΑ · ΒΓ + ΔΑ · ΔΓ

ΕΥΚΛΕΙΔΗΣ Β' 74 τ.2/43

Page 46: Ευκλειδης Β 74

-------------- Μαθηματικά για την Β ' Λυκείου -------------Απόδειξη Έστω R η ακτίνα του περιγεγραμμένου

κύκλου του τετραπλεύρου ΑΒΓ Δ, έχουμε:

Δ

Β

(ΑΒΓ) = ΒΑ · ΒΓ · ΑΓ } 4R

(ΑΓΔ) = ΔΑ · ΔΓ · ΑΓ 4R

( ) ( ΒΑ · ΒΓ + ΔΑ · ΔΓ)ΑΓ άρα ΑΒΓ Δ = --'------------'--4R

( Ι )

Ομοια: (ΑΒΓΔ) = (ΑΒ · ΑΔ + ΓΔ · ΓΔ) ΒΔ (2)

4R Από τις ( Ι ) και (2) παίρνουμε : (ΒΑ · ΒΓ + ΔΑ · ΔΓ) ΑΓ = ( ΑΒ · ΑΔ + ΓΒ · Γ Δ) ΒΔ

ΑΓ ΑΒ · ΑΔ + ΓΒ · ΓΔ => - = -------ΒΔ ΒΑ · ΒΓ + ΔΑ · ΔΓ Π αρατη ρι1σης. Στην Ευκλείδεια Γεωμετρία

αναφέρονται και άλλα δυο θεωρήματα με το όνομα του Πτολεμαίου.

Ι " Σε κάθε εγγεγραμμένο τετράπλευρο ΑΒΓ Δ ισχύει: ΑΒ.Γ Δ=ΑΒ.Γ Δ+ΒΓ.ΑΔ

2" Για κάθε μη εγγράψιμο τετράπλευρο ισχύει: Α Β .Γ Δ<ΑΒ.Γ Δ+ΒΓ.ΑΔ

• Είναι προφανές ότι οι δυο προηγούμενες προτάσεις μπορούν να συμπτυχθούν στην: Για κάθε τετράπλευρο ισχύει: ΑΒ.Γ Δ::ΞΑΒ.Γ Δ+ΒΓ.ΑΔ Ι στορ ι κό

Ο Κλαύδιος Πτολεμαίος ήταν μαθηματικός, αστρονόμος και γεωγράφος που έζησε στην Αλεξάνδρεια το 2° μ.Χ. αιώνα. Ο Πτολεμαίος συγκέντρωσε όλες τις αστρονομικές γνώσεις των αρχαίων Ελλήνων και ιδίως του Ιππάρχου ( \ 90-1 20 π.Χ.) στο μεγάλο έργο του «Μαθηματική Σύνταξις», στο οποίο θεμελίωσε το γεωκεντρικό σύστημα και

ερμήνευσε τη φαινόμενη κίνηση των πλανητών με τη βοήθεια της Γεωμετρίας.

Στο έργο αυτό, που είναι πιο γνωστό με το Αραβικό του όνομα «Αλμαγέστη» , ο Πτολεμαίος έκανε ακριβείς μετρήσεις εφαρμόζοντας τη «γεωμετρική τριγωνομετρία» βασική πρόταση της οποίας είναι το 1 ο θεώρημα, που αναφέρεται στις παρατηρήσεις.

r. rl ροβλι1 ματα Ι . Α ν ο Θεσσαλικός κάμπος έχει έκταση

5. 1 00 τετραγωνικά χιλιόμετρα, ο πληθυσμός της γης είναι 7 δισεκατομμύρια και για να σταθεί όρθιος ένας άνθρωπος απαιτείται χώρος ενός τετραγώνου πλευράς 50 εκατοστών, χωράει όρθιος ο ανθρώπινος πληθυσμός στο Θεσσαλικό κάμπο;

Λύση • Πληθυσμός γης=7 . Ι 09 κάτοικοι • Μέρος της επιφάνειας της γης, που απαιτείται

, , ' θ ' θ Ι Ι Ι 2 απο εναν ο ρ ιο αν ρωπο=- · - = - m 2 2 4

• Μέρος της επιφάνειας της γης, που απαιτείται για να σταθούν όλοι οι άνθρωποι

7 · 1 09 • .!. όρθιοι= 7 · 1 09 _!_ m2 = 4 Κm2

4 1 06 (αφού

= 7 · 1 03 . _!. = Ι 750Κm2 • 4

Επομένως ο Θεσσαλικός κάμπος χωράει όλους τους ανθρώπους της γης ακόμη και ξαπλωμένους !

2 . Στο τρίγωνο ΑΒΓ είναι ΑΒ=7, ΒΓ=ΙΟ και Γ Α=8. Το τρίγωνο ολισθαίνει πάνω στην ευθεία ΒΓ και παίρνει μια νέα θέση Α 'Β 'Γ, όπως φαίνεται στο παρακάτω σχήμα. Ποιο πρέπει να είναι το μήκος του τμήματος ΒΒ 'έτσι, ώστε το κοινό μέρος των τριγώνων ΑΒΓ και Α 'Β 'Γνα έχει εμβαδόν ίσο με το μισό του εμβαδού του ΑΒΓ;

Λύση

Α Α'

Επειδή τα τρίγωνα ΑΒΓ και ΔΒ Τ είναι όμοια, έχουμε:

\:;; =(�;)2

��

=(�;)2

� ��

= �

� ΒΤ = sJ2, έτσι : ΒΒ ' = ΒΓ - ΒΤ = 1 0 - sJ2 ΕΥΚΛΕΙΔΗΣ Β' 74 τ.2/44

Page 47: Ευκλειδης Β 74

------------- Μαθηματικά για την Β' Λυκείου ------------­Α Α '

Β ' Ένα ισόπλευρο τρίγωνο είναι

εγγεγραμμένο σε τετράγωνο πλευράς α, όπως φαίνεται στο παρακάτω σχήμα. Ποιο είναι το εμβαδόν του ισοπλεύρου τριγώνου;

Α\Jση Επειδή ΑΒ=ΒΓ και ΒΕ=ΒΖ τα ορθογώνια

τρίγωνα ΑΒΕ και ΒΓΖ είναι ίσα, άρα: ΑΒΕ = ΓΒΖ = 30° ( αφού ΕΒΖ = 60° )

Α Β

Δ Ζ Γ

Από το ορθογώνιο τρίγωνο ΑΒΕ έχουμε: ΑΒ α 2α , ΒΕ = = - = - ετσι συν30° J3 J3 '

2 ( ΒΕΖ) = _!_ ΒΕ · ΒΖημ60ο = 2

1 2α 2α J3 α2J3 2 ' J3 .

J3 · 2 =-3-

4 . Στο χάρτη απεικονίζεται η Κρήτη. Μπορείτε να υπολογίσετε προσεγγιστικά την έκτασή της,

(( ....Lru..,y:; J ��--."-��� . �--�"'·:, _ro, � � ) .:..... \··-

ι _ _

όταν 0,5 cm του χάρτη αντιστοιχούν σε απόσταση 20Km στην πραγματικότητα;

Λ ι'Jt�η

Χωρίζουμε το χάρτη σε ορθογώνια παραλληλόγραμμα όπως φαίνεται στο σχήμα:

Εμβαδόν 1 ου ορθογωνίου : Ε ι= 1 44 ·30=3420Κm2

Εμβαδόν 2ου ορθογωνίου : E2=94-44=4 1 3 6Km2 Εμβαδόν 3ου ορθογωνίου : Ε3=52 ·20= 1 400Κm2

Προσεγγιστική έκταση Κρήτης: 8956 Κm2

Ι Ι rφαηΊρψιη : Η ακριβής έκταση της Κρήτης είναι: 826 1 km2

5 . Διπλώνουμε ένα ορθογώνιο παραλληλόγραμμο, διαστάσεων α, β ( α>β) κατά μήκος μιας διαγωνίου του όπως φαίνεται στο παρακάτω σχήμα. Ποιο είναι το εμβαδόν του κοινού μέρους των δυο τριγώνων που σχηματίζονται;

Δ Δ Επειδή ορθ.Α Β Γ = ορθ.Δ Β Γ , είναι ' ' ΑΒΓ = ΔΓΒ, έτσι το τρίγωνο ΕΒΓ είναι

ισοσκελές, άρα ΕΒ=ΕΓ οπότε ΕΔ=ΕΑ=χ, επομένως ΕΒ=ΕΓ=α-χ. Από το Πυθαγόρειο

Δ Θεώρημα στο ορθ.Δ Ε Β , έχουμε :

? β2 2 ο 2 ( z 2 ο α- + ΕΒ = ΕΔ- + ΔΒ <:::> α - χ) = χ + β- <:::> χ = --=--2α Δ

Β

Γ ΕΥΚΛΕΙΔΗΣ Β ' 74 τ.2/45

Page 48: Ευκλειδης Β 74

------------ Μαθηματικά για την Β ' Λυκείου------------1 1 (ΕΒΓ) = (ΑΒΓ) - (ΑΕΓ) = 2αβ - 2ΑΕ · ΑΓ =

1 1 α2 + β2 2αβ - ( α2 + β2 ) β = -αβ - - · · β = . 2 2 2α 4α 6. Στο παρακάτω τετράγωνο, που έχει

εμβαδόν 1 , το Μ είναι το μέσο της αντίστοιχης πλευράς του. Ποιες οι τιμές των εμβαδών Ει , Ε2, Ε3, και Ε4;

Μ

Λ\)ση

Σε κάθε τρίγωνο ισχύουν: Ε = _!_βγημΑ 2 α2 = β2 + γ2 - 2βγσυνΑ (2)

( 1 )

β2 + γ2 - α2 Από τη (2) παίρνουμε: συνΑ = 2βγ ,

οπότε: η μ 2 Α = 1 - συν2 Α = 1 - [.:__β_2 _+ 2--'-γβ-2γ_-_α_' J'

( 2βγ)2 - (β2 + γ2 - α2 )2 = = 4β2γ2 ( 2βγ + β2 + γ2 - α2 ) (2βγ - β2 _ γ2 + α2 )

4β2γ2 [(β + γ )2 _ α2 J - [ α2 - (β - γ )]2 = =-----=�---------

Επειδή �=45" =Μ, θα είν ι: 4β2γ2 (β + γ + α) (β + γ - α) ( α + β - γ) ( α - β + γ) 1 ξ =-ΑΒ· ΒΕ · ημ45" = 2

1 J2 J2 1 = - 1 · ΒΕ · - = -ΒΕ και � =-ΒΕ · ΒΜ·ημ45" 2 2 4 2 = _!_ΒΕ_!_ · J2 = J2 ΒΕ . 2 2 2 8 Έτσι αρκεί να υπολογίσουμε το ΒΕ.

Α Β

Μ

Γ

Είναι: ΒΕ = � ΒΟ ( αφού το Ε είναι το κέντρο 3 , Δ 2 J2 J2 βαρους το Α Β Γ ) = - · - = - . 3 2 3

Επομένως ξ = fi42 . fi

32 =-6

1 , � = fi . fi =J_ 8 3 12 '

7 . Αποδείξτε τον τύπο του Ήρωνα:

Ε = �τ { τ - α ) { τ - β) { τ - γ ) για το εμβαδόν του τριγώνου.

= 4β? ? (3) -γ-Αν θέσουμε α+β+γ=2τ, τότε:

α + β + γ - 2α = 2τ - 2α β + γ - α = 2 ( τ - α) α + β + γ - 2β = 2τ - 2β έτσι β + γ - β = 2 ( τ - β) α + β + γ - 2γ = 2τ - 2γ β + γ - γ = 2 ( τ - γ) Οπότε η (3) γράφεται:

2 2τ · 2 ( τ - α) 2 ( τ - γ) 2 ( τ - β ) ημ Α - -- 4β2γ2 -4τ ( τ - α) ( τ - β ) ( τ - γ) , = 2 2 ' αρα β γ

ημΑ = ��τ ( τ - α) ( τ - β) ( τ - γ) β γ Και έτσι η ( 1 ) γίνεται:

Ε = �r-� (,--τ --α-) (.,---τ -_ β-,-)-( τ---,-γ) Ι στορικό ση μtίωμο : Ο Ήρων ο Αλεξανδρεύς ήταν Έλληνας

μαθηματικός, φυσικός και μηχανικός, που έζησε και έδρασε στην Αλεξάνδρεια μεταξύ του 1 ου π. Χ. και του 1 ου μ.Χ. αιώνα.

Ήταν διευθυντής της Ανώτατης Τεχνικής Ελληνικής Σχολής, του πρώτου Πολυτεχνείου στον

ΕΥΚΛΕΙΔΗΣ Β ' 74 τ.2/46

Page 49: Ευκλειδης Β 74

-------------- Μαθηματικά για την Β ' Λυκείου-------------

κόσμο. Το συγγραφικό έργο του Ήρωνα, ήταν μεγάλο σε έκταση και πολύπλευρο, σώθηκαν δέκα τρία έργα και θεωρούνται τέσσερα χαμένα. Ο Ήρωνας στα έργα

του περιγράφει την κατασκευή πρωτοποριακών μηχανών για την εποχή του, καθώς και εφαρμογές των μαθηματικών στην Τεχνολογία και τη ζωή .

Υποθέτουμε ότι το παρακάτω σχήμα παριστάνει ένα τοπογραφικό διάγραμμα ενός αγρού, που βρίσκεται κοντά σε μια ερημική παραλία. Μπορείτε να βρείτε μια μέθοδο προσεγγιστικού υπολογισμού του εμβαδού του αγρού;

Χωρίζουμε τη βάση ΒΓ σε ν ίσα τμήματα και κατασκευάζουμε τα αντίστοιχα τραπέζια, όπως φαίνεται στο σχήμα.

Α�Δ ' ' ' ' ' ' β ' ' ' ' Β : β ι : β2 ' : β,· ' ' ' ' ' h

B h h h h h r Το εμβαδόν του σχεδιαγράμματος κατά

προσέγγιση είναι: Ε ' = β + β ι h β ι + β2 h β ν + Β h = + + . . . + 2 2 2

h = (β + 2β 1 + 2β2 + . . . + 2βν + Β)- . 2 Α ν το τοπογραφικό διάγραμμα έχει κλίμακα

1 :κ, τότε για το εμβαδόν Ε του αγρού έχουμε : � = (�)2 <:::::> Ε = κ2 · Ε ' Ε ' 1

λ Σε κάθε τρίγωνο δείξτε ότι ισχύει: 1 1 1 τ - + - + - < -α β γ Ε

1 1 1 Είναι: Ε = -βγημΑ � -βγ1 = -βγ, 2 2 2 έτσι: Ε � _!_ βγ <:::::> _!_ � _l_ ( 1 ) 2 β 2Ε , , 1 α (2) 1 β (3) ομοια εχουμε: - � - και - � -γ 2Ε α 2Ε Με πρόσθεση κατά μέλη των ( 1 ) , (2) και (3)

1 1 1 α + β + γ παίρνουμε : - + - + - < ( αφού δεν α β γ 2Ε

μπορεί να ισχύει η ισότητα στις ( 1 ) , (2) και (3) συγχρόνως, γιατί αν ίσχυε θα είχαμε, , , , 2τ τ Α = Β = Γ = 90ο άτοπο)= - = - . 2Ε Ε

Ι!l ιφ :.: r:� ιηι;�· " ii ' Επειδή Ε=ρτ η ανισότητα που δείξαμε 1 1 1 τ 1 1 1 1 - + - + - < - <:::::> - + - + - < -α β γ τρ α β γ ρ

γράφεται:

, fli .. Στον πάπυρο του Rhind φαίνεται ότι οι αρχαίοι Αιγύπτιοι υπολόγιζαν το εμβαδόν τετραπλεύρου πολλαπλασιάζοντας τα ημιαθροίσματα των απέναντι πλευρών του. Ήταν σωστός ο υπολογισμός του; Αν δεν ήταν, τότε πώς συνδέεται το εμβαδόν του τετραπλεύρου με το γινόμενο των ημιαθροισμάτων των απέναντι πλευρών του;

δ

α - - - - - --�/' Α

_:ος; .... .........

,. ,. "' ,.

..... ..... ..... , "" "' ' "' ' "' ' , "' '

Β

"' ' ,. "' ..... ..... .....

β i , "" ..... ..... ..... j ; "' ..... ..... .... Δ-�-� -- -- --- ___ _::_:·Ξ�� γ Γ

Έστω Ε το εμβαδόν του τετραπλεύρου ΑΒΓ Δ και α, β, γ και δ τα μήκη των πλευρών του . Έχουμε :

2Ε = (ΑΒΓ) + (ΑΔΓ) + (ΑΒΔ ) + ( ΓΒΔ) = 1 1 1 1 = -αβημΒ + -γδημΔ + -αδημΑ + -βγημΓ 2 2 2 2 1 1 � 2( αβ + γδ + αδ + βγ ) = 2( α + γ ) (β + δ ) .

Ά Ε α + γ β + δ , , , ρα � -- · -- , οπου το = ισχυει, οταν : .2 2 ημΑ=ημΒ=ημΓ=ημΔ= 1

<:::::> Α = Β = Γ = Δ = 90° <:::::> ΑΒΓΔ ορθογώνιο .

Ο πάπυρος του Rhind, που γράφτηκε το 1 650 π.Χ. , είναι αντίγραφο ενός άλλου πρωτότυπου πάπυρου παλαιότερου κατά δυο αιώνες. Στον πάπυρο περιλαμβάνονται 87 προβλήματα με τις λύσεις τους γραμμένα σε κοινή ιερατική γραφή και όχι σε

ΕΥΚΛΕΙΔΗΣ Β ' 74 τ.2/47

Page 50: Ευκλειδης Β 74

------------- Μαθηματικά για την Β ' Λυκείου-------------ιερογλυφική . Ο πάπυρος ανακαλύφθηκε στα μέσα του 1 9ου αιώνα και αγοράστηκε από το Βρετανό Α.Χ. Rhind και πουλήθηκε από τους εκτελεστές της διαθήκης του στο Βρετανικό Μουσείου, όπου βρίσκεται

σήμερα.

! 1 . Αν τετραπλεύρου

Ε είναι το εμβαδόν ενός πλευρών α, β, γ και δ, τότε

ισχύει: Ε � �(α + β) (γ + δ) .

Λίιση Με πλευρά τη ΒΔ κατασκευάζουμε τρίγωνο

ΒΓ Δ, τέτοιο, ώστε ΒΓ=γ και ΔΓ=β, έτσι Δ Δ

Β Γ Δ = Β Γ ' Δ , άρα (ΒΓΔ)=(ΒΓΔ)

Δ

Α α Β

Γ

β

Γ

I I Έχουμε: -( α+β)( γ + δ) = - ( αγ+ αδ +βγ+βδ) = 4 4 1 ( 1 I I 1 ) = - -αδ + -βγ + -αγ + -βδ 2': 2 2 2 2 2

1 I =>-z( α - y)ημ60° +-χ ( α - z)ημ60° + 2 2 +.!. y ( α - χ ) � .!. α · α · ημ60° 2 2 <=> z (α - y) + χ (α - z) + y (α - χ ) � α2 • 1 3 . Στο τρίγωνο ΑΒΓ τα Δ και Ε είναι

σημεία των πλευρών του ΑΒ και ΑΓ αντίστοιχα. Α ν Κ είναι το σημείο τομής των ΒΕ και Γ Δ, (ΒΔΚ)=5, (ΒΚΓ)=7 και (ΓΕΚ)=3, ποιο είναι το εμβαδόν του τετραπλεύρου ΑΔΚΕ;

Λίιση Θα χρησιμοποιήσουμε την πρόταση : Αν μια

γωνία ενός τριγώνου είναι ίση ή παραπληρωματική μια γωνιά ενός άλλου τριγώνου, τότε ο λόγος των εμβαδών των δυο τριγώνων ισούται με το λόγο των γινομένων των πλευρών που περιέχουν τις ίσες ή παραπληρωματικές γωνίες.

Δ

Α Λ / ., I '

Γ • Τα τρίγωνα ΒΚΓ και ΒΕΓ έχουν κοινή την

, 7 ( ΒΚΓ) ΒΚ · ΒΓ ΒΚ ΕΒΓ ' άρα 1 0 = (ΒΕΓ ) = ΒΕ · ΒΓ = ΒΕ ( I )

• Τα τρίγωνα ΒΔΓ και ΒΑΓ έχουν την Β κοινή, , 1 2 (ΒΔΓ) ΒΔ · ΒΓ ΒΔ

2': - - αδημΑ +-βyημΓ +-αyημΑΒΓ +-βδημΑΔΓ = 1 ( 1 I I , I , 2 2 2 2 2

αρα: ( ΑΒΓ) = (ΒΑΓ ) = ΒΑ · ΒΓ = ΑΒ (2)

=�[{ΑΒΓ) +(ΓΔΒ) +(ΑΒΓ) +(Γ'Μ)] = .!_(Ε + Ε) = Ε 2 2 12 . Αν α>Ο και χ, y, zε (Ο,α] , δείξτε ότι :

χ( α-z)+y( α-χ)+z( α-y)::::;α2

Λίιση Κατασκευάζουμε ισόπλευρο τρίγωνο πλευράς

α και πάνω στις πλευρές του ΒΓ, Γ Α και ΑΒ παίρνουμε σημεία Κ, Λ και Μ αντίστοιχα έτσι, ώστε:

Α

Γ

ΒΚ=χ, Γ Λ=y και AM=z αντίστοιχα, οπότε: ΚΓ=α-χ, ΛΑ=α-y και ΜΒ=α-z. Προφανώς: (ΑΛΜ)+(ΒΜΚ)+(ΓΚΛ)::::;(ΑΒΓ)

• Τα τρίγωνα ΒΔΚ και ΒΑΕ έχουν την ΑΒΕ κοινή, άρα:

(ΒΔΚ) ΒΔ · ΒΚ ΒΔ ΒΚ ( ι ) 1 2 7 , �-...:... = = - · - = · - ετσι: (ΒΑΕ) ΒΑ · ΒΕ ΑΒ ΒΕ (2 ) (ΑΒΓ) 1 0 (ΒΔΚ) 42 5 42 -'----...!... = => = => (ΒΑΕ) 5 (ΑΒΓ) (ΑΒΓ) - 1 0 5 (ΑΒΓ)

=> 25 (ΑΒΓ) = 42 (ΑΒΓ) - 420 => -1 7 (ΑΒΓ) = = -420 <=> ( ΑΒΓ) = 420

1 7 Άρα: (ΑΔΚΕ) = (ΑΒΓ) - (ΒΔΚ) - (ΒΚΓ) - ( ΓΕΚ) =

= 420 - 1 5 = 1 65 . 1 7 1 7 Π α ρ ατή ρηση : Το πρόβλημα μπορεί να λυθεί

και με την εφαρμογή της πρότασης: Ο λόγος των εμβαδών δυο τριγώνων που έχουν

το ίδιο ύψος ισούται με το λόγο των βάσεών τους 1 4 . Στο παρακάτω τετράπλευρο το Κ είναι

το μέσο της πλευράς ΑΒ και το Λ το μέσο της Γ Δ. Με Ει , Ε2, Ε3, και Ε4 συμβολίζουμε τα

ΕΥΚΛΕΙΔΗΣ Β' 74 τ.2/48

Page 51: Ευκλειδης Β 74

------------- Μαθηματικά για την Β ' Λυκείου ------------εμβαδά των αντίστοιχών τριγώνων. Δείξτε ότι Ει+ E3=Ez+E4

ΛίJση Η λύση θα στηριχθεί στη γνωστή πρόταση :

Μια διάμεσος ενός τριγώνου το χωρίζει σε δυο -" ισεμβαδικά τρίγωνα. Στο Κ Γ Δ το ΚΛ είναι διάμεσος, άρα (ΚΔΛ)=(ΚΛΓ) ( 1 )

Γ -" Στο Λ Α Β το ΛΚ είναι διάμεσος, άρα (ΛΑΚ)=(ΛΚΒ) (2)

( Ι ) Είναι : Ε , + Ε3 + (ΚΡΛΜ) = (ΚΑΛ) + (ΚΓΛ) (η

(ΚΛΒ) +(ΚΛΔ) =� +Ε4 +(ΚΡΛΜ) άρα ξ +Ε; =Ε2 +Ε4

1 3 . Ποιο είναι το μέγιστο εμβαδό που μπορεί να έχει ένα τρίγωνο με μήκη πλευρών α,β,γ για τις οποίες ισχύει α:51:5β:52:5γ:53 ;

. \ iJση Σε κάθε τρίγωνο ισχύει: Ε = ±βγημΑ � ±βγ · 1 = ±βγ , γιατί Ο<ημΑ:5 1 , αφού Ο<Α< 1 80°. Το = ισχύει όταν Α=90°. Για το τρίγωνο του προβλήματος έχουμε : Ε � _!_βγ � _!_ 2 · 3 = 3 , το = όταν β=2, γ=3 και 2 2 Α = 90° , τότε όμως α2 = β2 + γ2 = 22 + 32 = 1 3 και α= Jl3 , άτοπο, αφού α � 1 . Ε � _!_γα � _!_3 · 1 = � το = όταν γ=3 α= 1 2 2 2 ' ' και Β = 90° , τότε όμως β2 = α2 + γ2 = 1 2 + 32 = 1 0 , οπότε β= Μ , άτοπο, αφού β � 2 . Ε � _!_ αβ � _!_ 1 · 2 = 1 , το = όταν α= 1 , β=2 και 2 2 r = 90° , οπότε γ2 = β2 + α2 = 22 + 1 2 = 5 , άρα

γ= J5 δεκτό, γιατί 2 � γ = J5 � 3. Έτσι Ε,113,= 1 . Ι 6 . Δυο αδέλφια κληρονόμησαν ένα χωράφι

σχήματος κυρτού τετραπλεύρου. Ο ένας από τα αδέλφια πρότεινε να ενώσουν τα μέσα των απέναντι πλευρών του χωραφιού και ο καθένας τους να πάρει δυο μη διαδοχικά τεμάχια από τα τέσσερα στα οποία χωρίζεται το χωράφι. Ήταν δίκαιη η προτεινόμενη μοιρασιά;

ΛίJση Έστω Κ, Λ, Μ και Ρ είναι τα μέσα των πλευρών

του τετράπλευρου ΑΒΓ Δ, που αναπαριστάνει το χωράφι. Όπως γνωρίζουμε το ΚΛΜΡ είναι παραλληλόγραμμο του οποίου οι πλευρές είναι παράλληλες προς τις διαγώνιες του ΑΒΓ Δ.

Β

Μ Γ Φέρνουμε το ΑΘ .1 ΒΔ , που τέμνει την ΚΡ

στο Η. Επειδή ΚΡ//ΒΔ και Κ μέσο ΑΒ, θα είναι και Η μέσο ΑΘ. Έχουμε :

1 1 1 1 ( ΑΚΡ) = 2 ΚΡ · ΑΗ = 2 · 2 ΒΔ 2 ΑΘ =

=�ΘΒΔ· ΑΘ ) =� (ΑΒΔ) , όμοια

( ΓΛΜ) =_!_(ΓΒΔ) , έτσι (ΑΚΡ)+(ΓΛΜ) =_!_{ΑΒΓΔ) ( 1 ) 4 4 όμοια ( ΒΚΛ) + (ΔΜΡ) = _!_(ΑΒΓΔ ) (2) 4 Οι διαγώνιες ενός παραλληλόγραμμου το χωρίζουν

σε τέσσερα ισοδύναμα τρίγωνα, οπότε έχουμε: • (ΑΚΟΡ ) + (ΓΛΟΜ) =

( Ι ) = (ΑΚΡ) + (ΟΚΡ) + ( ΓΛΜ) + ( ΟΛΜ) = = _!_ (ΑΒΓΔ) + 2 (0ΚΡ) (3) 4

• ( ΒΚΟΛ) + (ΔΜΟΡ) = ( 2 ) = (ΒΚΛ) + ( ΟΚΛ) + ( ΔΜΡ) + ( ΟΜΡ) =

= _!_(ΑΒΓΔ) + 2 (0ΚΡ) (4) 4 Οι (3 ) και (4) βεβαιώνουν ότι η προτεινόμενη

μοιρασιά είναι δίκαιη . I 7. Α ν Ε είναι το εμβαδόν και τ η

ημιπερίμετρος ενός τριγώνου, δείξτε ότι: E �J3(�)z ΕΥΚΛΕΙΔΗΣ Β' 74 τ.2/49

Page 52: Ευκλειδης Β 74

------------ Μαθηματικά για την Β ' Λυκείου------------

Γνωρίζουμε ότι για χ, y, z θετικούς ισχύει: x + y + z "?_ � ( 1 ) 3

(τ-α) +(τ-β\ +(τ-γ) ,1 * Είναι 1 >�(τ-α) ( τ-β) ( τ-γ)=> 3

3τ - ( α+β+γ) , 1 => 3 > �{ τ-α)( τ-β) { τ - γ) =>

=> 3τ -2τ "?_ ψ τ-α) ( τ-β){ τ - γ) => 3 � "?_ψ τ-α)( τ-β)( τ-γ) � "?_( τ-α)( τ-β)( τ-γ) 3 27

� "?_τ( τ-α) ( τ-β) ( τ-γ) => {l "?_�τ( τ-α) ( τ-β) ( τ-γ) => n νn � "?_Ε=>Ε::;;J3Θ J2 . Αποδεικνύουμε την ( 1 ) Από την ταυτότητα του Euler έχουμε :

α' +β3 +y' -3αβγ=� ( α+β+γ}[( α-β)2 +(β-γ)2 +( γ-α)2] Για α,β,γ θετικά το δεύτερο μέλος της προηγούμενης είναι μη αρνητικό, άρα:

α3 + β3 + γ3 - 3αβγ "?_ Ο <=> α3 + β3 + γ3 "?_ 3αβγ , το = ισχύει όταν α=β=γ. Αν στην προηγούμενη θέσουμε: α = if;.., β = ifY, και γ = if;, παίρνουμε :

x + y + z "?_ 3� <=> x + y + z "?_ � , όπου 3 το = ισχύει όταν x=y=z.

Η προηγούμενη ανισότητα γενικεύεται για ν το πλήθος θετικούς αριθμούς α1 , αυ . . . , αν :

αι + α, + . . . + αν ν ! ν - ν "?_ ν α ι α2 . . . αν "?_ 1 1 1 - + - + . . . + -αι α2 αν Η περίφημη αυτή ανισότητα ονομάζεται

ανισότητα αριθμητικού γεωμετρικού aρμονικού μέσου και την απέδειξε πρώτος ο μεγάλος Γάλλος Μαθηματικός A.L . Caychy ( 1 789- 1 875) στο βιβλίο του : Cours d' Analyse de I' Ecole Polytechnique, το 1 82 1 . Από τότε έχουν δοθεί πάνω από 70 αποδείξεις της σπουδαίας αυτής ανισότητας. '� Όπως δείξαμε, σε κάθε τρίγωνο ισχύει:

Ε ::;; .J3 ( � J 2 , το = ισχύει μόνο όταν ισχύει το = στην * , δηλαδή μόνο όταν:

τ - α = τ - β = τ - γ και τελικά α = β = γ ,

αυτό σημαίνει ότι: από όλα τα τρίγωνα που έχουν την ίδια περίμετρο, το ισόπλευρο έχει το μέγιστο εμβαδόν.

c-; Στο πρόβλημά μας δείξαμε ότι σε κάθε τρίγωνο ισχύει:

� "?_ Ε <=> _!_ ( α + β + γ J 2 "?_ .J3E <=> 3.J3 3 2

Ισχύει

<=> (α + β + γ)2 "?_ 4.J3E ( 1 ) 3 α2 + β2 + γ2 "?_ ( α + β + γ)2

3 γράφεται ισοδύναμα

γιατί

3α2 + 3β2 + 3γ2 "?_ α2 + β2 + γ2 + 2αβ + 2βγ + 2γα <=> <=> 2α2 + 2β2 + 2γ2 - 2αβ - 2βγ - 2γα "?_ Ο <=> <=> ( α - β )2 + (β - γ )2 + (γ - α)2 "?_ Ο , που ισχύει

Έτσι από τις ( 1 ) και (2) παίρνουμε: Σε κάθε τρίγωνο ισχύει: α2 + β2 + γ2 "?_ 4.J3E Η προηγούμενη ανισότητα στο τρίγωνο ήταν θέμα στη Διεθνή Μαθηματική Ολυμπιάδα του 1 96 1

6 . Στο τετράπλευρο ΑΒΓΔ τα Κ, Ρ, Μ και Ρ είναι τα μέσα των πλευρών ΑΒ, ΒΓ, ΓΔ και ΔΑ αντίστοιχα. Α ν η ΑΛ τέμνει τις ΔΚ και ΒΜ στα Ε και Ζ αντίστοιχα και η ΓΡ τέμνει τις ΔΚ και ΒΜ στα Θ και Η, δείξτε ότι:

(ΕΖΗΘ)=(ΑΕΚ)+(ΒΖΛ)+(ΓΗΜ)+(ΔΘΡ)

Έχουμε: ( ΑΛΓΡ) = ( ΑΛΓ) + ( ΑΓΡ) = = _!_(ΑΒΓ) + _!_ (ΑΓΔ) (αφού ΑΛ διάμεσος 2 2

Γ

του τριγώνου ΑΒΓ και ΓΡ διάμεσος του ΓΑΔ)

= _!_( ΑΒΓ Δ ) . Όμοια: ( ΒΜΔΚ ) = _!_ ( ΑΒΓ Δ ) . 2 2 Έτσι ( ΑΛΓΡ) + ( ΒΜΔΚ) = (ΑΒΓΔ) <=>

<=> (ΑΕΘΡ ) + (ΕΖΗΘ) + (ΓΛΖΗ) + (ΒΖΕΚ) + + (ΕΖΗΘ) + ( ΔΜΗΘ) = = ( ΑΚΕ ) + (ΒΖΕΚ) + (ΒΖΛ) + ( Γ ΛΖΗ) + (ΓΗΜ) + + (ΔΜΗΘ) + (ΔΘΡ) + (ΑΕΘΡ) + (ΕΖΗΘ) <=> <=> (ΕΖΗΘ) = ( ΑΕΚ) + (ΒΖΛ) + ( ΓΗΜ) + ( ΔΘΡ) .

ΕΥΚΛΕΙΔΗΣ Β ' 74 τ.2/50

Page 53: Ευκλειδης Β 74

-------------- Μαθηματικά για την Β ' Λυκείου--------------1 9 . Από το εσωτερικό σημείο Ρ του

τριγώνου ΑΒΓ φέρνουμε παράλληλες προς τις πλευρές του, οπότε σχηματίζονται τρία τρίγωνα, όπως φαίνεται στο παρακάτω σχήμα, εμβαδών Εα, Ep και Εγ.

Δ Α ν Ε το εμβαδόν του Α Β Γ δείξτε ότι:

1 1 1 9 -- + -- + -- > -JE: JE; jE.; - JE

Β

Α

.l Δ

Γ

Εί\'αι Ρ Θ Η - Α Β Γ , άρα: :" � ( e�} �Jf� e�� .k�e� � (ι) , I α I α I ομοια : - = - · - = - · - (2 ) jE; ΡΕ JE ΗΓ JE

I α I α I jE; = ΔΡ . JE = ΒΘ . JE (J) Α

Β Γ

( 1 1 1 ) 1 = ( ΒΘ + ΘΗ + ΗΓ ) ΒΘ + ΘΗ + ΗΓ . JE

( ΒΘ ΒΘ ΘΗ ΘΗ ΗΓ ΗΓ ) 1 = 1 +

ΘΗ + ΗΓ + ΒΘ + I + ΗΓ + ΒΘ +

ΘΗ + 1 . JE =

= [3 + ( ΒΘ + ΘΗ ) + ( ΒΘ + ΗΓ ) + ( ΒΗ + ΗΓ )] · -1 > ΘΗ Β Θ ΗΓ ΒΘ ΗΓ ΘΗ JE -

;::: ( 3 + 2 + 2 + 2 ) .JE =}Ε (αφού για x,y>o είναι

� + 2::. ;::: 2 <:::::> χ 2 + y2 ;::: 2 xy <:::::> χ 2 + y2 - 2 xy ;::: Ο <:::::> Υ χ <:::::> ( χ - y ) 2 ;::: Ο ισχύει)

2 0 . Ένα τετράγωνο είναι χωρισμένο σε ορθογώνια παραλληλόγραμμα των οποίων οι πλευρές είναι παράλληλες προς τις πλευρές του τετραγώνου όπως φαίνεται στο παρακάτω σχήμα. Δείξτε ότι το άθροισμα των λόγων των πλατών προς τα μήκη των ορθογωνίων είναι τουλάχιστον ένα.

,------ --ι ' � ·-·-ι

� ,r- - � Λί1ση Έστω ότι:

--·-1 • το τετράγωνο έχει πλευρά μήκους α, οπότε το

εμβαδόν του είναι α2

• το τετράγωνο χωρίζεται σε ν το πλήθος ορθογώνια

• τα μήκη των ορθογωνίων είναι α ι , α2, α3 , . . . , αv και τα πλάτη τους β 1 , β2 , β3 , . . . , βν αντίστοιχα. Έχουμε:

; +;_ +;_ + . . . + βν = αι�ι + α2�2 + α3�3 + . . . +

αν�v ;::: αι α2 α3 αν αϊ α2 α:; α�

α β α β α.β α β ;::: _ι_ο ι + 2/ +4 + . . . +� (γιατί: α ;::: α ; με

α- α- α- α-ί= 1 ,2, . . . ,ν αφού κάθε ένα από τα ν ορθογώνια περιέχεται στο τετράγωνο)

α2 α2 ( αφού ο αριθμητής του προηγούμενου

κλάσματος είναι το άθροισμα των εμβαδών των ορθογωνίων στα οποία χωρίζεται το τετράγωνο)= 1

Βιβλιογραφ ία I . Ν. ΒΑΣΙΛΙ ΕΦ - Α. Γεγκόροφ, Πανενωσιακές

Μαθηματικές Ολυμπιάδες της ΕΣΣΔ 1 96 1 - 1 99 1 , Κάτοπτρο, Αθήνα 1 998

2. Ο. Bottema, Topics in Eleιηentary Geometry, Springer, USA 2008

3 . R . Hosberger, l n Polya ' s Footsteps, ΜΑΑ, USA 1 997 4 . R. Johnson, Advanced Euc l idean Geometry, Dover, New

York, 1 960 5. P .Viu, Notes of Euc l idean Geometry, 1 998,

lu.ΙJ:ι,_ιη,ι ι l ι , Ιlι_ιL',}1ιlι ι \ ί ι ι �ι:ο_ι ι ι c ι ι :-..J.J ι ιιι_Ι 6. Crux Mathematicorum, Canadian Mathematical Society 7 . Gazeta Mathematica, Societatea de Sti inte Matematice

din Romania 8 . 1 1\1[1_: _ __\1 \Ι_\Ι . ι ι ι ι ι l_ι c Ι, π οι . ι , ι ·�

9. ilι.lJ2_:___\I_II_��L;�1 1 l l L' \ l \ • .Ιc'()_lll 1 0 . 1 Ί ι l[2:____\Ι:ΙΙ � ι ι ιη � ω ι η . i � ι ιι,·<J.ιι

ΕΥΚΛΕΙΔΗΣ Β ' 74 τ.2/51

Page 54: Ευκλειδης Β 74

-------------- Μαθηματικά για την Β ' Λυκείου-------------

Εξ ίσωση Ευθε ίας Φανέλη Ά ννυ - Καρδαμίτσης Σπύρος

Στην δεκαετία του 1 630 ο Καρτέσιος και ο Fermat ανακαλύπτουν την αναλυτική γεωμετρία η οποία δίνει νέες διαστάσεις στην απόδειξη γεωμετρικών προβλημάτων. Αναλυτική γεωμετρία σημαίνει ότι κάθε καμπύλη μπορεί να αναπαρασταθεί με μία αλγεβρική εξίσωση, αλλά και το αντίστροφο, κάθε εξίσωση προσδιορίζει μια καμπύλη. Έτσι στις αρχές του 1 7ου αιώνα οι μαθηματικοί της εποχής εκείνης, ήρθαν αντιμέτωποι με ένα εκρηκτικό αριθμό καμπύλων προς εξέταση. Για τις νέες καμπύλες οι πw.ιές αποδεικτικές διαδικασίες ήταν πλέον ανεπαρκείς, έτσι διατυπώθηκαν αποδείξεις με την χρησιμοποίηση νέων εργαλείων: της συμβολικής άλγεβρας και της αναλυτικής γεωμετρίας.

Χ Ι> Η l: Ι \ Ι ΕΣ Ε Π Ι Σ Η Μ Λ :\ l: Ε Ι Σ ll . Ένα σημείο M(x0,y0) ανήκει στην γραμμή C

που έχει εξίσωση f(x, y) = Ο αν και μόνο αν οι συντεταγμένες του σημείου επαληθεύουν την εξίσωση f(x, y) = Ο, δηλαδή f(X0,y0) = Ο

2 . Για τις γραμμές: C , : fι (χ, y) = Ο, C2 : f2(x, y) = Ο ξέρουμε ότι τα κοινά τους σημεία δίνονται από { f1 (x , y) = O την λύση του συστήματος: (Σ)

f0 (x , y) = Ο

ειδικότερα: Α ν το σύστημα (Σ) είναι αδύνατο, τότε οι γραμμές C 1 και C2 δεν έχουν κοινά σημεία. Α ν το σύστημα (Σ) είναι ταυτοτικό, τότε οι γραμμές c , και c2 συμπίπτουν.

3 . Αν (ε ι ) : α , χ+β , y+γ ,=Ο, (ε2) : α2χ+β2y+γ2=Ο τότε:

(ε ι ) j_ κ, = (α ι , β , ) και (ε ι ) // ν , = (β , , -α ι )

επίσης (ε2) j_ κ2 = (α2 , β2) και (ε2) // ν2 = (β2, -α2) άρα (ε ι ) //(ε2) <=> κ , // κ2 ' (ε ι ) j_ (ε2) <=> κ , j_

όπου φυσικά κ 1 , κ2 ::j:. Ο

4. Αν η ευθεία (ε 1 ) είναι παράλληλη με μια ευθεία ( ε2), τότε ο συντελεστής της είναι: λε , = λε, , ενώ αν η ευθεία (ε ι ) είναι κάθετη με μια

ευθεία ( ε2) , τότε ο συντελεστής της είναι: 1 λ,. • λ,. = - I <=> λr = - -' 1 ., •• , λ ε,

5 . Ο συντελεστής διεύθυνσης λ χρησιμοποιείται

είναι γνωστός ο συντελεστής της λ και αυτή ικανοποιεί μια ιδιότητα, τότε αυτή έχει μορφή y = λχ+β και ο αριθμός β υπολογίζεται με βάση την δοσμένη ιδιότητα. Α ν για την ζητούμενη ευθεία γνωρίζουμε ότι διέρχεται από γνωστό σημείο Α(χο,Υο) , τότε αυτή έχει μορφή y - Υ σ = λ( χ - Χο) .

7 . Για τον υπολογισμό της οξείας γωνίας δύο ευθειών (ε 1 ) και (ε2) έχουμε τα παρακάτω :

!Ε> Αν λ1 και λ2 είναι οι συντελεστές των παραπάνω ευθειών και ισχύει: λ ι λ2 = -1 τότε, η γωνία θ των ευθειών είναι 90°.

!Ε> Αν ένας από τους συντελεστές λ1 , λ2 δεν ορίζεται, τότε η οξεία γωνία θ των ευθειών (ε ι ) και ( ε2) υπολογίζεται από το ορθογώνιο τρίγωνο που σχηματίζουν οι ευθείες και ο άξονας χχ '

!Ε> Αν λ1 λ2 ::j:. - 1 , τότε η οξεία γωνία υπολογίζεται με την βοήθεια των διανυσμάτων. • Θεωρούμε τις ευθείες (ε ι ) : α , χ + β ,y + γ , = Ο, (ε2) : α2χ + β2y + γ2 = Ο

• Θεωρούμε τα διανύσματα: ν , = (β 1 , -α1 ) και ν 2 = (β2, -α2) που έχουν ίσους συντελεστές με

τις ευθείες (ε 1 ) και (ε2) αντίστοιχα και είναι με τις ευθείες παράλληλα.

για να δείξουμε ότι τρία σημεία Α, Β και Γ είναι • Τότε η οξεία γωνία των ευθειών (ε 1 ) και (ε2) συνευθειακά. Δείχνοντας ότι: λΑ13 = λΑr έχουμε:

-> -> ΑΒ/1 ΑΓ , οπότε τα σημεία Α, Β, Γ είναι συνευθειακά.

6. Αν αναζητάμε την εξίσωση ευθείας που

θα είναι ίση ή παραπληρωματική με την γωνία φ των δύο διανυσμάτων που υπολογίζεται με

την βοήθεια της σχέσης: συνφ = α . βι�ι ι αι · β

ΕΥΚΛΕΙΔΗΣ Β ' 74 τ.2/52

Page 55: Ευκλειδης Β 74

------------- Μαθηματικά για την Β ' Λυκείου -------------8 . Δίνεται η παραμετρική εξίσωση (ε) : Α(λ)χ+Β(λ)y+Γ(λ) = Ο ( 1 ) όπου λ

πραγματικός αριθμός (παράμετρος), και ζητείται να δειχθεί ότι διέρχεται από σταθερό σημείο .

!Ε> Για να δείξουμε αν η παραπάνω σχέση : παριστάνει ευθεία, προσδιορίζουμε ένα σύνολο Δ στο οποίο μεταβάλλεται η παράμετρος λ. Για τον

{Α(λ) = ο

} σκοπό αυτό θεωρούμε το σύστημα: το Β(λ) = Ο οποίο και λύνουμε. " Αν το σύστημα είναι αδύνατο τότε Δ = R • Α ν L είναι το σύνολο λύσεων του συστήματος,

τότε η σχέση Α(λ)χ+Β(λ)y+Γ(λ)= Ο παριστάνει ευθεία για κάθε λ � L , οπότε Δ=R-L !Ε> Για να δείξουμε ότι όλες οι ευθείες με

εξίσωση ( 1 ) διέρχονται από το ίδιο σημείο, θεωρούμε δύο διαφορετικές ευθείες (ε 1 ) , (ε2) για τυχαίες τιμές λ 1 και λ2 του Δ. και προσδιορίζουμε την τομή M(x0,y0) των παραπάνω ευθειών. Διαπιστώνουμε ότι οι συντεταγμένες του σημείου Μ επαληθεύουν την εξίσωση ( 1 ) αυτό δηλώνει ότι όλες οι ευθείες διέρχονται από το Μ.

Σε σύστημα αξόνων xOy δίνεται η ευθεία

χ + y = 4 (ε). Από τα σημεία Α(3, Ο) και 0(0, Ο)

φέρνουμε ευθείες παράλληλες που τέμνουν την

(ε) στα σημεία Γ και Δ τέτοια ώστε ΓΔ = 3. Να

βρεθεί ο συντελεστής διεύθυνσης των δύο

παραλλήλων ευθειών.

Λi>ση : • Α ν ορίζεται συνευθειακές διευθύνσεις λ ο

συντελεστής διεύθυνσης των παραλλήλων ευθειών, τότε η ευθεία ΑΓ έχει εξίσωση :

y = λ(χ - 3) και η ευθεία ΟΔ έχει εξίσωση y = λχ, με λ i- λε = -1 . Οι συντεταγμένες του σημείου Γ προσδιορίζονται από την λύση του συστήματος: {y=λ(χ-3)} άρα Γ (3λ+4 ,__!:_) , x+y=4 λ+ 1 λ+ 1 αντίστοιχα οι συντεταγμένες του σημείου Δ προσδιορίζονται από την λύση του συστήματος: { Υ = λχ } , άρα Δ (__i__ , 4λ ) x + y = 4 λ+ 1 λ+ 1 όπου και για τις δύο περιπτώσεις έχουμε λi--1 .

Όμως είναι ΓΔ = 3 , επομένως ΓΔ2 = 9 � ( 4 4+3λ)2 ( 4λ λ )

2 J

λ+ 1 - λ+ Ι + λ+ l - λ+ Ι =9�λ--2λ-1= Ο � λ= 1+ J2 ή λ= 1-J2 που είναι δεκτές και οι δύο.

• Α ν δεν ορίζεται συντελεστής διεύθυνσης τότε: χ = 0 } οι κατακόρυφες ευθείες ky = 3

Λύνω (Σ) χ = Ο } � χ = Ο Γ (Ο, 4) x + y = 4 y = 4

Λύνω (Σ) χ = 3 } � χ = 3 Δ (3, 1 ) x + y = 4 y = 1

Τότε ΓΔ =�(Ο - 3)2 + (4 - 1)2 = Jϊ8 = 3J2 -::;: 3 . χ = 0

} Άρα οι απορρίπτεται. χ = 3 Λσι<ηση 2 '� Δίνεται η εξίσωση : 2χ2 - 7xy + 3y2 = Ο

α) Να αποδείξετε ότι παριστάνει δύο

ευθείες, για τις οποίες να βρείτε τις εξισώσεις

τους.

β) Να βρείτε την οξεία γωνία που

παριστάνουν οι παραπάνω δύο ευθείες.

Λύση : Θεωρώ την εξίσωση 2χ2 - 7xy + 3/ = Ο ( 1 )

δευτεροβάθμια με άγνωστο το χ οπότε αυτή έχει διακρίνουσα: Δ = 49/ - 24/ = 25y2 �Ο

( l )�x= 7y ± J251 = 7y ± 5y � 4 4 {χ = Ίy: Sy} άρα x=3y, x=y/2, δηλαδή 7y - 5y χ = _..:...._..:....

4 παριστάνει δύο ευθείες τις (ε 1 ) και (ε2) με εξισώσεις : x-3y=O και 2x-y=O αντίστοιχα.

ΕΥΚΛΕΙΔΗΣ Β ' 74 τ.2/53

Page 56: Ευκλειδης Β 74

------------- Μαθηματικά για την Β ' Λυκείου -------------β) Έχουμε (ε ι ) // v 1 = (3 , 1 ) και (εz) // v2 = ( 1 ,

2), οπότε η οξεία γωνία των ευθειών είναι ίση ή παραπληρωματική με την γωνία φ των δύο αυτών διανυσμάτων. Για τον υπολογισμό της γωνίας φ

ii · β 3 · 1 + 1 · 2 έχουμε: συνφ= ---- = = Ιa l · lβ l �32 + 1 2 · ,Ν + 22

( 1 )

5 J2 ' 450 ----==----= = - , επο μεν ως φ = JIO . JS 2

Δίνεται η εξίσωση (1-2λ)χ+(λ+l)y+8λ-1=0

α) Να δείξετε ότι η εξίσωση (1) παριστάνει

ευθεία για κάθε λ πραγματικό αριθμό.

β) Να δείξετε ότι όλες αυτές οι ευθείες

διέρχονται από σταθερό σημείο Μ του οποίου

να προσδιορίσετε τις συντεταγμένες.

α) Θεωρούμε το σύστημα { 1 - 2λ = 0} δηλαδή

{λ = 1 1 2} το οποίο είναι

1 + λ = 0 λ = - 1

αδύνατο, συνεπώς η ( 1 ) παριστάνει ευθεία για κάθε λ Ε IR .

β) Για λ = 1 /2 προκύπτει η ευθεία: 3 (ε ι ) : Οχ + - y +3 = 0 <::::> y = - 2 2

και για λ = - 1 προκύπτει η ευθεία (εz) : 3χ + Oy - 9 = Ο <=> χ = 3 οι ευθείες αυτές τέμνονται στο σημείο Μ(3 , -

2) . Αρκεί να δείξουμε ότι όλες οι ευθείες διέρχονται από το σημείο Μ, δηλαδή αρκεί οι συντεταγμένες του σημείου Μ να επαληθεύουν τον τύπο της ( 1 ) . Πράγματι είναι: ( 1 - 2λ) - 3 + ( I + λ) · ( -2) +8λ - 1 =

=3 - 6λ - 2 - 2λ +8λ - 1 = Ο για κάθε λ Ε IR, άρα ο ι ευθείες διέρχονται από το σταθερό σημείο Μ για κάθε λEIR .

Να βρεθούν οι εξισώσεις των ευθειών που

διέρχονται από το σημείο Μ(1, 1) και

σχηματίζουν με τους άξονες τρίγωνο με εμβαδό

ίσο με 2 τετραγωνικές μονάδες.

Από το σημείο M( l , 1 ) διέρχονται οι ευθείες χ

= 1 και y - 1 = λ( χ - 1 ) με λ =F Ο. Η πρώτη ευθεία δεν σχηματίζει τρίγωνο με τους άξονες, επομένως δεν είναι λύση του προβλήματος.

Η ευθεία y - 1 = λ( χ - 1 ) τέμνει τους άξονες χχ ' και yy ' στα σημεία Α και Β αντίστοιχα που προσδιορίζονται από τις λύσεις των συστημάτων: {y - 1 = λ(χ - 1)}

και {y - 1 = λ(χ - 1)},

y = O χ = Ο

επομένως έχουμε: Α ( λ� 1 , Ο) και Β( Ο, λ-1 ) .

Το σχηματιζόμενο τρίγωνο ΟΑΒ έχει εμβαδό Ε που είναι ίσο με :

Ε = _!_ · IOA I · IOB I = _!_ ι λ - Ι I · Ιλ - 1 1 = (λ - 1 )2

2 2 λ 2lλ l

(λ - 1 )� ) 1 1 Επομένως Ε=2<=> 2lλl 2<=>(λ-1 )-=4 λ ( 1 )

Α ν λ > Ο τότε: ( 1 ) <=> λ 2 - 6λ + I = Ο <=> (λ = 3 + 2 J2 ή λ = 3 - 2 J2 ) που είναι και οι

δύο δεκτές τιμές. Αν λ < Ο τότε: ( I ) <=> λ2 + 2λ + 1 = Ο που έχει

διπλή ρίζα την λ = - 1 που είναι επίσης δεκτή, επομένως οι ζητούμενες ευθείες είναι οι :

(ε ι ) : y - 1 = (3 + 2 J2 )(x - 1 ) (ε2) : y - 1 = (3 - 2 J2 )(χ - 1 ) και (ε3) : y - 1 = - (χ - 1 )

Λ σ κηση 5η Δίνεται τρίγωνο ΑΒΓ με Α= (2, -1) . Αν η

εξίσωση του ύψους ΒΚ είναι 7χ - 10y + 1 = Ο

και η εξίσωση της διχοτόμου ΒΛ είναι 3χ - 2y

+5 = Ο να βρεθούν:

α) Οι συντεταγμένες της κορυφής Β του

τριγώνου.

β) Ο συντελεστής διεύθυνσης της πλευράς

ΑΓ.

ΕΥΚΛΕΙΔΗΣ Β ' 74 τ.2/54

Page 57: Ευκλειδης Β 74

-------------- Μαθηματικά για την Β ' Λυκείου-------------γ) Οι εξισώσεις των πλευρών ΑΒ και ΑΓ.

δ) Την εξίσωση της πλευράς ΒΓ.

λ �Ίση : α) Οι συντεταγμένες της κορυφής Β

προσδιορίζονται από την λύση του συστήματος των ευθειών του ύψους ΒΚ και της διχοτόμου ΒΛ του τριγώνου, δηλαδή του συστήματος {7χ - l Oy = - 1}

' 3χ - 2y = -5 άρα Β (-3 , -2)

β) Είναι λ8κ = }____ i- Ο. Επειδή το ύψος ΒΚ 1 0

είναι κάθετο στην πλευρά ΑΓ έχουμε : λ8κ ·λΑr = - 1 1 0 άρα λυ = -- . . 7

. . . . . . . \

Γ

γ) Ο συντελεστής διεύθυνσης της πλευράς ΑΒ

' ' 1 . ξ ' θ ' ειναι : Λ ,_ R

= S , οποτε η ε ισωση της ευ ειας ΑΒ

είναι : y + 1 = _!_ (χ - 2) δηλαδή χ - 5y - 7 = Ο και 5

η εξίσωση της ευθείας ΑΓ είναι:

y + 1 = _ .!_Q (x - 2) δηλαδή l Ox +7y - 1 3 = Ο 7

δ) Επειδή η ΒΛ είναι διχοτόμος του τριγώνου το συμμετρικό Α ' της κορυφής Α ως προς την ΒΛ

ανήκει στην πλευρά ΒΓ. έχουμε λ8Λ % i- Ο,

2 ΟΠότε λΒΛ 'λΑΑ ' = -1 Q λΑΑ ' = - -3

Η εξίσωση της ΑΑ ' είναι: y + 1 = -� (χ - 2) 3

δηλαδή 2χ +3y - 1 = Ο. Αν Ο είναι η τομή της διχοτόμου ΒΛ και της

ΑΑ ' , τότε οι συντεταγμένες του Ο προκύπτουν

από την λύση του συστήματος : {3x-2y=-5}

, άρα 2x+3y = 1

0(-1 , 1 ) . Αν ονομάσουμε (α, β) τις συντεταγμένες του

σημείου Α ' , επειδή το Ο είναι το μέσο του ΑΑ '

, α + 2 β - 1 εχουμε : -- = - 1 <:::::> α = -4 και -- = l <:::::>β= 3 , 2 2

άρα Α ' (-4, 3) . τέλος από τα σημεία Α ' (-4, 3 ) και Β (-3 , -2) ορίζεται η ΒΓ, ο συντελεστής διεύθυνσης της ευθείας ΒΓ είναι λ8r = -5 η εξίσωση της ευθείας ΒΓ είναι: y + 2 = - 5 (χ +3 ) δηλαδή , 5χ + y + 17 = Ο .

Η Λ Ν Α Λ Υ τ Ι Κ i-1 ί\Ι ΕΘΟΔΟΖ.: (Η απι'ιόειξη μιας γα•J μ ::τρ ικ1)ς πρότασης με

την �olj i�: :ιu. της ανιι!λυηυ<:J�ς γ::ωμ::τρ ίας) 1:8> Ορίζουμε κατάλληλο σύστημα αξόνων Oxy

και σ' αυτό εκφράζουμε όλα τα σημεία του σχήμα­τος με την βοήθεια των συντεταγμένων. Η εκλογή του συστήματος αναφοράς θα γίνεται με τέτοιο τρόπο, ώστε να έχουμε όσο το δυνατόν περισσότερα σημεία με τετμημένες ή τεταγμένες μηδέν ώστε να μένουν αναλλοίωτες οι ιδιότητες των σχημάτων .

1:8> Αντί λοιπόν να αποδείξουμε μια ζητούμενη γεωμετρική πρόταση δείχνουμε την αντίστοιχη αλγεβρική .

r υη ι ι η> ι κ Η !l � >ΟΤ \ 2: Η

Η ευθεία (ε) διέρχεται από σημείο Μ(χο ,y0)

Το σημείο Μ(Χο ,yo) ανήκει στην ευθεία (ε)

Οι ευθείες (ε ι ) και (ε2) τέμνονται

Οι ευθείες (ε ι ) και (ε2) είναι παράλληλες με Ευκλείδεια έννοια, δηλαδή (ε ι )n(ε2) = 0. Οι ευθείες (ε ι ) και (ε2) συμπίπτουν.

Οι ευθείες (ε ι ) και (ε2) είναι κάθετες.

Οι ευθείες (ε ι ) , (ε2) και (ε3) συντρέχουν.

Η ευθεία (ε ι ) σχηματίζει με την (ε2) οξεία γωνία θ .

Τα σημεία Α, Β, Γ είναι συνευθειακά.

Δύο ευθύγραμμα τμήματα είναι ίσα.

\ . \. ! �� Β Ι> � Ε Η � ΙI II' O Γ \ C> ' � j

Οι συντεταγμένε (Χσ,Υο) του σημείου Μ

επαληθεύουν την εξίσωση τηι:: ευθείας. Οι συντεταγμένες (ΧωΥσ) του σημείου Μ

επαληθεύουν την εξίσωση της ευθείας. Το σύστημα (Σ) των ευθειών (ε ι ) και (ε2) έχει μοναδικiι λύση .

Το σύστημα (Σ) των ευθειών (ε ι ) και (ε2) είναι αδύνατο.

Το σύστημα (Σ) των ευθειών (ε ι ) και (ε2) είναι ταυτοτικό.

α . β = Ο ε 1 ε�

Μ ι α από τις τρεις ευθείες διέρχεται από την τομή των δύο άλλων.

l a · β l -- = συνθ

Ι a l · l β l Γ ε ΑΒ ή Β ε ΑΓ ή

Γ ε ΒΓ iι ΑΒ/1 ΑΓ ή (ΑΒΓ)=Ο Έχουν τα ευθύγραμμα

τμJ'ιματα ίσα μέτρα.

ΕΥΚΛΕΙΔΗΣ Β' 74 τ.2/55

Page 58: Ευκλειδης Β 74

------------- Μαθηματικά για την Β ' Λυκείου-------------Άσκη ση 6η Θεωρούμε ορθογώνιο τρίγωνο ΟΒΓ (Ο =

90°). Εξωτερικά του τριγώνου ΟΒΓ κατασκευ­

άζουμε τα τετράγωνα ΟΒΔΕ και ΟΓΚΛ. Να

δείξετε ότι:

α) Να δείξετε ότι τα σημεία Κ, Ο και Δ είναι

συνευθειακά.

--> --> άρα ΜΒ _ι ΜΓ , δηλαδή το τρίγωνο ΜΒΓ είναι

ορθογώνιο στο Μ. Επιπλέον έχουμε:

ΜΓ� (ο -�)2 + -7)2 � Jβ2 ; γ2

ΜΒ� (β - β � γ )2 + (Ο - γ � β )2 �Jβ2 ; 12 ' β) Αν Μ είναι το μέσο της ΚΔ, να δείξετε ότι άρα ΜΒ = ΜΓ, συνεπώς το τρίγωνο ΜΒΓ είναι και

το τρίγωνο ΜΒΓ είναι ορθογώνιο και ισοσκελές. ισοσκελές.

Λ.ίJση ; Λ σrc(ψ;η 7 '1 α) Θεωρούμε ορθοκανονικό σύστημα αξόνων Στην διαγώνιο ΒΔ ενός ορθογωνίου

- ___,. - -t xOy με i tt ΟΒ , j tt ΟΓ , τότε Β(β, Ο ), Γ(Ο,γ) παραλληλογράμμου ΑΒΓΔ θεωρούμε ένα

με β > ο, γ > ο , επομένως έχουμε: Δ(β, σημείο Ρ. Αν Ε είναι το συμμετρικό του Γ ως

-β) Ε(Ο, -β) Κ(- γ, γ) και Λ(- γ, Ο) προς το Ρ και Η, Ζ είναι οι προβολές του Ε στις

ΑΒ και ΑΔ αντίστοιχα, να δειχθεί ότι τα σημεία

Ζ, Η και Ρ είναι συνευθειακά.

Λ ί1ση : --> -

Θεωρούμε σύστημα αξόνων xAy με ΑΒ tt i , --> -

ΑΔ tt j τότε τα παρακάτω σημεία έχουν

συντεταγμένες: Α(Ο, 0), Β(β, 0), Γ(β, δ) και Δ(Ο, δ) με β, δ > Ο . Η ευθεία της διαγωνίου ΒΔ έχει συντελεστή

λΒΔ = -� και εξίσωση y - Ο = β

δ -- (χ - β) β

Θεωρούμε την ευθεία που διέρχεται από τα δηλαδή δχ + βy - βδ = Ο . σημεία Κ και Ο αυτή έχει συντελεστή διεύθυνσης

λκο = -γ = -1 καθώς και την ευθεία που γ

διέρχεται από τα σημεία Ο και Δ που έχει

συντελεστή διεύθυνσης λοΔ = -δ = - 1 . Επειδή λκο δ

= λοΔ , οι ευθείες ΚΟ και ΛΟ είναι παράλληλες, οπότε τα σημεία Κ, Ο και Λ είναι συνευθειακά.

β) Επειδή το σημείο Μ είναι το μέσο του ΚΔ

αυτό έχει συντεταγμένες Μ ( β ; γ , γ; β ) Άρα ΜΒ = (β - β;γ , 0 - γ;β) = (β;γ

, β;γ) Μr = (ο - β - γ

γ - γ - β ) = (- β - γ β + γ ) 2 ' 2 2 ' 2 ' --> --> β+γ β-γ β-γ β+γ επομενως J\ι1Β Ι\ι1Γ = --- · -- +-- ·-- =Ο

2 2 2 2

Έστω Ρ(χ 1 , y 1 ) οι συντεταγμένες του τυχαίου σημείου Ρ, επειδή ανήκει στην ευθεία ΒΔ είναι:

δχ 1 + βy 1 - βδ = Ο, ( 1 ) επιπλέον το σημείο Ρ είναι το μέσο της ΓΕ οπότε ισχύει: {

Χε +β} Χ =--1 2 Χε =2χ -β � _ 1 } άρα Ε (2χ ι-β, 2y ι-δ) ΥΕ +δ LYE -2yι -δ Υι =--

2

ΕΥΚΛΕΙΔΗΣ Β ' 74 τ.2/56

Page 59: Ευκλειδης Β 74

-------------- Μαθηματικά για την Β ' Λυκείου -------------

Από το σχηματιζόμενο ορθογώνιο ΑΗΕΖ έχουμε: Η(2χ 1 - β, Ο) και Ζ(Ο, 2y 1 - δ) .

Τέλος βδ - δχ !

β

από την σχέση ( 1 ) έχουμε : Υ ι = και αφού χ 1 i- β οι συντελεστές

διεύθυνσης των ευθειών ΡΗ και ΡΖ είναι: βδ - δχ ι

λrΗ = _-_Υ_ι = _ ____,_β __ = δ(β - χ ι ) = � και Χ ι - β Χ ι - β β(β - χ ι ) β βδ - δχ ι - δ

λrz = Υ ι - δ = β = δχ ι = � -Χ ι -Χ ι βχ ι β άρα λr11 = λrz � ΡΗ ΡΖ , δηλαδή τα σημεία

Ζ, Η και Ρ είναι συνευθειακά.

Α.σκη ση 8 11 Έστω :\1 το μέσο της πλευράς ΒΓ

τετραγώνου ΑΒΓ Δ πλευράς α. Η ΑΓ τέμνει την

Δi\1 στο σημείο Ε και Ζ είναι η προβολή του Γ

στην ΔΜ. Να δείξετε ότι: ΓΖ = 3ΖΕ. . \ ίι ση : Έστω ορθοκανονικό σύστημα αξόνων xAy με

-) � ΑΒ ίί i . ΑΔ ίί J τότε τα παρακάτω σημεία έχουν συντεταγμένες: Α(Ο, 0), Δ( Ο, 2α), Γ(2α, 2α), Β(2α, 0), Μ(2α, α) . Η εξίσωση της ευθείας ΑΓ είναι προφανώς η y = χ και η ευθεία ΔΜ έχει

y - ! α -α εξίσωση : ---- = - ή χ + 2y = 4α. χ 2α

ΑΙΟ Ο ι

Η τομή τους Ε έχει συντεταγμένες που προσδιορίζονται από την λύση του συστήματος

, ξ , , Ε ( 4α 4α ) των παραπανω ε ισωσεων και ειναι -,- . 3 3 ΓΖ ..l ΔΜ <=> λrz · λΔΜ =-1 <=> λrz = 2, οπότε η

ευθεία ΓΖ έχει εξίσωση : y - 2α = 2(χ - 2α), δηλαδή 2χ - y = 2α. Οι συντεταγμένες του σημείου

Ζ προκύπτουν από την λύση του συστήματος των ευθειών ΔΜ και ΓΖ οπότε Ζ (

Sα , 6α ) . 5 5

2α = J5 2 2

3ΖΕ = 3 ( 8; _ �α ) + ( 65α _ �α ) = . . . = Ys επομένως είναι ΓΖ = 3ΖΕ.

Άσκη ση 9'1

Α ν σε τρίγωνο ΑΒΓ είναι μp=μγ να δείξετε

β=γ.

ΛίJση :

Έστω ορθοκανονικό σύστημα αξόνων xBy με --> -ΒΓ// i , τότε τα παρακάτω σημεία έχουν

συντεταγμένες: Β(Ο, 0), Α( κ, λ) , Γ(μ, Ο) όπου κ, λ θετικοί.

Τα μέσα Δ και Ε των ίσων πλευρών του ισοσκελούς τριγώνου ΑΒΓ έχουν συντεταγμένες: Δ( κ + μ � ) και Ε ( � � ) 2 ' 2 2 ' 2

Έστω μβ = !1γ τότε είναι ΒΔ2 = ΓΕ2 επομένως έχουμε: ( κ;μ -ο)2 + (� - o)z = (� -μ)2 + (� -o)z

(κ + μ)2 = (κ - 2μ)2 � 6κμ = 3μ2 � 2κ = μ αφού ο μ είναι θετικός αριθμός. Επομένως :

β = ΑΓ = J(μ - κγ + (Ο - λ)2 = Jκ2 + λ2 και γ = ΑΒ = Jκ2 + λ2 , άρα β = γ.

ΕΥΚΛΕΙΔΗΣ Β' 74 τ.2/57

Page 60: Ευκλειδης Β 74

Μαθηματ ι κά Γεν ι κής Πα ιδε ίας γ ια τη Γ ' τάξ η του Λυκε ίου

Στατιστι κή Α. Έστω /ιτι οι τιμί;ς y , μιας μεταβλητής Υ, σε ένα

<),· ι ;ιμα μιταβ/ιλλονται σύμφωνα με τον τύπο:

y , =-= αχ , +- μ , τόπκαι τα μέτρα θέσης, διάμεσος δ Υ -

ιωι μ!_ση τιμή y μεταβάλλονται και γίνονται αντί-

στοιχα y = αχ +- β για κάθε α, β ε JR και

<\ "" αδ , + [) , όταν α> Ο (γιατί; ) .

Π α ρ α δ ε ί γ μ α τ α Α I . Δίνονται οι αριθμοί: 2α + β, 3α + β, 4α + β, 7α + β, 9α + β , όπου α, β

θετικοί αριθμοί. ί. Α ν {;χουν μέση τιμή χ = 1 7 και διάμεσο

δ = 1 5 να βpεθούν οι α, β. ί ί . Αυξάνουμε τους παραπάνω αριθμούς

κατά β μονάδες τον καθένα και η διάμεσος γίνε­τω 28.

Λ tιξάνουμε κατά 20% τον καθένα και η ό ιάμισος γίνεται 24. Να βρεθούν οι παραπάνω αριΟμοί .

Λ ύ σ η ί Επι: ι01) οι α,β είναι θετικοί. , οι παραπάνω α­

ρ ι U μ ο ί ι ίνα ι με αύξουσα σειρά , οπότε έχουμε : ι) - 4 α i β . ;Δ, ρα 4α + β -= ι 5 (1 )

/\ κ uμα c ίνα ι : 2α t β + 3α + β + 4α + β + 7α + β + 9α + β

\ <=>

χ

5 2 �α + :'iβ - 5 β · 5 β. 1 7 (2) ---'- <=> χ =- α + . Άρα, . α + = 5

ι\πό τις ( I ) και (2) προκύπτει: α = 2 και β = 7 . ί ί . Αν θέσουμε δ, = 4α + β και δ, , δ, τις τιμές

της δ ιαμέσου μετά τις αντίστοιχες μεταβολές, έ-

χουμc : δ . "" δ + β και δ ο = δ + 20 δ , ' ' ' χ I 00 χ {δ + β = 28

Οπότε έχουμε : ' + 20 _ <=> δχ δ , - 24

1 00 ' )4α + β + β = 28 <=> l 4α + β + ( 4α + β) · Ο, 2 = 24

J 4α + β + β = 28 {β = 8 <=> <=> . l4α + β = 20 α = 3

Γ Τσικαλουδάκης

Α2. Μια αριθμητική πρόοδος έχει πρώτο όρο α 1 = 6 και διαφορά ω = 4

I . Να βρεθούν 5 διαδοχικοί όροι της προ{ι­δου με δ ίαμεσο : δ = 50 ,

2 . Να βρεθούν 6 διαδοχικοί όροι της προό­δου με δίαμεσο : δ = 24

3 . Αν στην παραπάνω πρόοδο αλλάξοιψι: τη διαφορά σε ω = 7 ποια θα ε ίναι η δ ιάμεσος στ ις παραπάνω περιπτώσεις (I ) και (2 ) ;

Α t'J σ η I . Έστω αk , αk -; ι , αk ,- ' , αk . 3 • αk . � , 5

διαδοχικοί όροι της προόδου Επειδ1) ν = 5 , περιττός, θα πρέπε ι : ό = α, , 1

Άρα αk +2 = 50 Αλλά: αk ,2 = 50 <=> α1 + ( l< + 2 - \ )ω c::: 50

<::::> 6 + (k + J )4 = 50 <::::> ( k + \ )4 oo 44 <::;> k := j () Άρα αk = α 1 0 = α1 + 9ω Οπότε οι ζητούμενοι όροι ε ίναι: οι α 1 0 , α1 1 , α 1 2 , α 1 3 • α1 4 δηλαοιΊ ο ι :

42 , 46 , 50 , 54 , 5 8 2 . Έστω αk , α k " ι , α • • : . α, , ; , α , , 4 , α, , .

6 διαδοχικοί όροι της προόδου Ε ίνα ι : δ = αk -• 2 + αk ' ' , οπότε : _?-�+ υ,,_'- "' 2�

2 2 <=> 2α1 + (2k + 3)ω "" 24 ς,

2 12 + (2k+3)4 = 24 <=> 8k = 24 <=> l< = 3

2 Άρα οι ζητούμενοι όροι ε ίνα ι ο ι :

α3 , α4 , α5 • α6 , α7 , αχ 6ηλαοιΊ ο ι : 1 4 , 1 8 , 22 , 26 , 30 , 34

3 . Αλλάζοντας τη διαφορά ω της προόδου από 4 σε 7 , ο κάθε όρος της προόδου αυξ<iνcται κατά 3 , οπότε την ίδια αύξηση θα έχουμc και στη διάμεσο για τις περιπτώσεις ( l ) και (2 ) δηλαδ1) Οα είναι δ = 53 και δ = 27 αντ ίστοιχα

Η ομαδοποίηση παρατηριjσιχu ιJ ι , ι: ιιιJς y_rψα­κτηριστικού Χ , επηρεάζι:ι τα μ{rμα (Jι:-ση.,: ιωΟrι)ς και τα μέτρα διασποράς.

ΕΥΚΛΕΙΔΗΣ Β ' 74 τ.2/58

Page 61: Ευκλειδης Β 74

-------------- Μαθηματικά για την Γ Λυκείου -------------

Δηλαδή αλλάζοντας το πλήθος ή το πλάτος τω ν κλάσεων μπορεί να έχουμε αλλαγή στις τιμές των μέτρων θέσης και διασποράς.

Π α ρ α δ t ί γ μ α τ α Λ3. Οι μισθοί των υπαλλήλων μιας εταιρείας

παρουσιάζουν εύρος R = 2000 € Ομαδο­ποιούμε τους μισθούς σε 4 κλάσεις ίσου πλάτους. Η πρώτη κλάση έχει κέντρο χ, = 1000 Το πλήθος των υπαλλήλων ανά

κλιμάκιο (κλάση) μισθών είναι: 20, 30, 50, 10, από την 1 η έως και την 4ν κλάση αντί­στοιχα. Να βρεθούν οι κλάσεις και το συνολικό ποσό που δίνει η εταιρεία ανά μήνα για μισθούς.

Λ 1ι σ η Έχουμε 4 ι-.:/.άσει.: και το εύρος είναι 2000 ,

οπότε το πι.ατο.: της κάθε κλάσης είναι:

� οοο - οο . . λ . . . c = -- = :- ι-.:αι αφου η πρωτη κ αση εχει κε-4

\'τρο χ = 1 000 . '(ια τα κέντρα χ 2 , χ 3 , χ 4 των άλ-ί.ω\' ι-:ί.άσεωΥ έ;ι:ουμε : χ 2 = Ι 000 + 500 = 1 500

χ , = 1 5 00 + 500 = 2000 και ' � = 2000 + 500 = 2500

Επομέη·)ς ο ι τέσσερις κλάσεις είναι:

[750 ι �:'Ο) .

ι ι � 5 0. ι 7 5 0 ) ' [ 1 750, 2250) ' [2250, 2750)

Ο ι συχνότη τες των παραπάνω κλάσεων είναι: ν1 = 20 , ν 2 = 30 , ν3 = 50 , ν4 = 1 0

Οποτε το συνολικό πόσο για μηνιαίο μισθό τωv ωrυ/./.ι1λων, ανά μήνα είναι:

'

Σ χ , ν , == Χ ι · ν ι + x c · ν 2 + χ 3 · ν3 +Χ4 · ν4 =

' ' ' '

"' 20 . 750 + 30 . ι soo + 50 . 2000 + 1 ο . 2soo = 1 85 .000€

Α4. Οι τιμές χ1 μιας συνεχούς μεταβλητής Χ σε

ένα δείγμαμεγέθους ν έχουν ομαδοποιηθεί σι.; 5 κλάσεις πλάτους 1 Ο η κάθε μια α. Ν α βρεθεί το εύρος του δείγματος β. Αν η τρίτη κλάση έχει κέντρο

χ :� = 27 , να βρείτε τις κλάσεις

Λ 1ι σ η α. Έχουμε 5 κλάσεις πλάτους c = Ι Ο η κάθε

μία, οπότε το εύρος του δείγματος είναι: Ι{ : s . ι ο = so

β. Η τρίτη κλάση έχει κέντρο χ 3 = 27 , οπότε αυηΊ είναι η [22 , 32) Επομένως οι 5 κλάσεις είναι

οι: [2 , 1 2) , [1 2 , 22) , [22 , 32) , [32 , 42) , [42 , 52)

A S . Οι τιμές χ 1 μιας συνεχούς μεταβλητi]ς

χ σε ένα δείγμα μεγέθους ν έχουν ομαδο­ποιηθεί στις παρακάτω κλάσεις:

(5 , 10) ' ( 1 0 ' 15) ' (1 5 , 20) ' ( 20 , 25)

Αν οι τιμές χ 1 αυξηθούν κατά 20% και το

πλήθος των κλάσεων παραμείνει ίδιο : α. Να βρεθεί το εύρος των αυξημένων τι­

μών β . Να βρεθούν τα όρια των νέων κλάσεων γ. Να αποδείξετε ότι οι συχνότητες των

κλάσεων θα παραμείνουν ίδιες (όσο ήταν πριν την αύξηση)

Λ 1ι σ η α. Το εύρος των παρατηρήσεων είνα ι :

R = 25 - 5 = 20 . Σε αύξηση των τιμών κατά 20% και το εύρος

θα αυξηθεί 20% Οπότε το νέο εύρος R' είναι: 20

R' = R + -R = 20 + 0, 2 · 20 = 24 1 00

β. Αν το πλήθος των κλάσεων παραμείνει το ίδιο ( k = 4 ) τότε πρέπει να αλλάξει το πλάτος κλά-

24 σης και να γίνει: c = - = 6 (αύξηση πλάτους 4

20% ) Το κάτω όριο της πρώτης κλάσης 5 αυξάνι: ι

κατά 20% και γίνεται 6 , οπότε οι νέες κλάσεις ι.:ί ­

ναι: [6 , 1 2) ' [ 1 2 , 1 8) ' [ 1 8 , 24) ' [24 . 30) γ. Έστω ότι μια παρατήρηση t , ανήκει στην

κλάση [α , β) . Δηλαδή είναι: α :'Ξ: t1 < β Έστω ότι με την

αύξηση κατά 20% η τιμή t 1 γίνεται y 1 Τότι.: ι.; ίναι Υ ι = t 1 + Ο, 2t1 και ακόμα έχουμε:

α :'Ξ: t 1 < β <=> α + Ο, 2 · α ::;; t1 + Ο, 2 · t1 < β + Ο, 2 · β <::::> α + 0, 2 · α :'Ξ: y1 < β + 0, 2 · β ( \ ) Η κλάση [α , β) μετά την αύξηση κατά 20<%

των τιμών t 1 , γίνεται: [α + 0, 2 · α , β + 0. 2 · β ) Επομένως, λόγω των ισοδυναμιών ( I ), το πλιi ­

θος των παρα τηρήσεων των κλάσεων [α . β ) , [α + 0, 2 · α , β + 0, 2 · β) είναι ίδιο

Γ. • Αν δύο δείγματα Α , Β ενός πί.ηΟuσμuι) μεγέθους ν 1 , ν 2 αντίστοιχα, iχου ν την ίδια ,ιlί:ση τιμή χ (χΑ = Χ8 = χ) ως προς ένα χαραηηριστιι .. :() Χ και διαφορετική τυπική απόκ/. ιση. s λ =Ι= s H . τύτ<-

ΕΥΚΛΕΙΔΗΣ Β ' 74 τ.2/59

Page 62: Ευκλειδης Β 74

-------------- Μαθηματικά για την Γ ' Λυκείου -------------

μπορούμε εύκολα, με βάση τους τύπους Του σχο­λικού βιβλίου, να βρούμε την τυπική απόκλιση του δείγματος που αποτελείται και από τα δύο δείγματα μαζί.

• Α ν όμως είναι χ Α :f:- χ ΙJ , τότε για την εύρεση

της τυπικής s του δείγματος που αποτελείται και

από τα δύο δείγματα μαζί, απαιτείται ο τύπος: ? ?

S 2 = ν ι s :\ + ν2S ίJ η'

οι τύποι : ν ι + ν" ( ν )2 Σι S2 =� Σχ -� ν i=ι ν ν

( 1 ) Οι παραπάνω τύποι, εύκολα, διαμορφώνονται

' ' ? 1 Σν 2 (-) ? ' στην πιο ευχρηστη μορφη : s- = - t; - χ - η ν i = ι

s 2 = _.!._ :Σ:Χ � ν ; - (Χ)2 , αντίστοιχα ν i = ι

Οι παραπάνω τύποι ( 1 ) , συνήθως δίνονται ? ? ο , ο ν 1 s� + ν,Sίj δ , , τυπος s- = - εν ειναι αναγκη να ν ι + ν "

είναι γνωστός, αφού μέσα στην πορεία της λύσης της άσκησης, αναγκαστικά θα προκύψει

Π α ρ α δ ε ί γ μ α τ α Α6. Δύο δείγματα Α, Β ενός πληθυσμού μεγε­

θών 60 και 40 αντίστοιχα παρουσίασαν ως προς ένα χαρακτηριστικό Χ ίδια μέση τιμή : χΑ = χ8 = 5 και διακύμάνση s� = 4 και

s� = 9 αντίστοιχα Να υπολογιστεί η μέση

τιμή χ και η διακύμανση s 2 του συνό­λου των δύο δειγμάτων, θεωρούμενα ως ένα δείγμα. Λ ύ σ η Η μέση τιμή και των δύο δειγμάτων είναι ίδια:

χΑ = xu = 5 , οπότε και το συνολικό δείγμα θα έχει μέση τιμή χ = 5

Τότε έχουμε : s 2 = -1 I Ct ; - 5)2 ( 1 ) 5 0 i = l

Ακόμα είναι: 1 20 s� =-Σ(t; - 5)2 20 i = l

1 20 J Σ , και sίj = - (t ; - 5)-30 i = l

Οπότε έχουμε : I 6ο 2 4 = -Σ cιί - 5) � 60 i = l

60 ΣCt; - 5) 2 = 240 (2) i = l 60 1 40 και 9 = -Σ (t ; - 5)2 � 40 i = l

Σcι; - 5)2 = 36ο (3) i = l

Με πρόσθεση κατά μέλη των (2), (3) έχουμε : 1 00 Σ(t; - 5)2 = 600 , οπότε: i = l

2 _ _ 1 Σι οο ( - 5)2 - 600 - 6 s - ι - -100 i = J I 1 00

Α 7. Μια μεταβλητή Χ παρουσίασε σε ένα δείγμα 40 ατόμων μέση τιμή 2 και τυπική απόκλι­ση 1 Η ίδια μεταβλητή σε ένα άλλο δείγμα 60 α­

τόμων παρουσίασε μέση τιμή 7 και τυπική από­κλιση 4.

α) Να βρείτε ποιο δείγμα παρουσιάζει μεγα-

λύτερη ομοιογένεια.

β) Αν θεωρήσουμε και τα δυο δείγματα μαζί ως ένα δείγμα να υπολογίσετε την μέση τιμή και την τυπική απόκλιση του δείγματος των 1 00 ατόμων.

Λ ύ σ η α) Έστω Α και Β τα δύο δείγματα Τότε εί-

s 1 s 4 ναι: CV =� =- =50 % και CV = ___1L = - � 56 % Λ - 2 lJ - 7 ΧΑ Χ �

Άρα έχουμε : CVA < cvfJ ' που σημαίνει ότι το Α παρουσιάζει μεγαλύτερη ομοιογένεια από το Β

β) Για τη μέση τιμή των δύο δείγματα μαζί (θεωρούμενα ως ένα δείγμα) έχουμε :

χ = 40 . 2 + 60 . 7 = 500 = 5 ( 1 ) 1 00 1 00 , , ? 1 \' ., _, , Απο τον τυπο: s- = - Σ t i - - χ - , εχουμε: ν i = ι

I cu για το Α δείγμα: I = - Σ t 1 c - 2c , άρα 40 i = l

2ο I •ο Σ t ; 2 = 200 για το Β δείγμα : 1 6 = - Σ t ; 2 - 72 , i = l 60 i = l

40 άρα Σ t ; 2 = 3900 οπότε και για τα δύο δείγματα i = l

μαζί (θεωρούμενα ως ένα δείγμα) έχουμε : 60 20 40 Σ t; 2 =Σ t; 2 + Σ t ; 2 = 200 + 3900 = 4 1 00 και συνε-i = l i = l i = l

πώς είναι: co2 I Σ

ν 2 -2 I �� 2 -2 I 2 � =- t. - χ =-L.} -χ =- · 4100-5 ν i=ι 1 100 i=Ι 1 Ι 00 '

Επομένως είναι: S2 = 41 -25 = 16 και άρα S = 4 .

ΕΥΚΛΕΙΔΗΣ Β' 74 τ.2/60

Page 63: Ευκλειδης Β 74

------------- Μαθηματικά για την Γ Λυκείου -------------

Α 8 . Δίνεται ο παρακάτω πίνακας συχνοτή-των:

x i Ι 2 3 4 5 6 7 8

ν i 20 40 40 30 20 10 20 20

I . Να βρεθεί η διάμεσος. 2. Να βρεθεί η μέση τιμή και η τυπική α­

πόκλιση. 3 . Να ομαδοποιηθούν οι παρατηρήσεις σε

τέσσερις κλάσεις ίσου πλάτους, με εύρος παρα­τηρήσεων 8, κάτω όριο κλάσεων Ι και άνω, 9.

4 . Με βάση την παραπάνω ομαδοποίηση να βρείτε αν το δείγμα παρουσιάζει μεγαλύτερη ομοιογένεια από ό,τι πριν την ομαδοποίηση.

Λ ύ σ η

Ι . Από τον παρακάτω πίνακα συχνοτήτων :

xi Ι 2 3 4 5 6 7 8

ν i 20 40 40 30 20 10 20 20

προκύπτει ότι το μέγεθος του δείγματος είναι

ν = 200 , οπότε η διάμεσος είναι το ημιάθροισμα

της I ΟΟης και Ι Ο 1 ης παρατήρησης. Επομένως εί-

ναι: δ = 3 + 4 = 3 5 2 '

2. Για την μέση τιμή και τη τυπική απόκλιση, με βάση τον παραπάνω πίνακα συχνοτήτων, έχουμε: - 1 · 20+2 ·40+3 ·40+4 · 30+5 · 20+6 · 10+ 7 · 20+8 · 30 χ 800 = 4 200

200

ο 9 · 20 + 4 · 40 + I · 40 + 1 · 20 + 4 · I Ο + 9 · 20 + 1 6 · 30 s- =-----------------------200

1 100 =5. 5 Οπότε : s =.J5,5 �2,35 και CV= 2,35 =0,59 . 200 4

3 . Με ομαδοποίηση των παρατηρήσεων σε 4 κλάσεις ίσου πλάτους με κάτω όριο I και άνω ό­ριο 9 , έχουμε τον παρακάτω πίνακα:

[ ' ) [Ι , 3) [3 , 5) [5 , 7) [7 , 9) χ ί 2 4 6 8 νί 60 70 30 40

4 . Με βάση την παραπάνω ομαδοποίηση , έ­

χουμε: χ = 2 · 60 + 4 · 70 + 6 · 30 + 8 · 40 = 900

= 4 5 200 200 '

και 52 6, 25 · 60+ 0, 25 · 70+ 2, 25 · 30+ 12, 25 · 40 200

950 =4 75 200 '

cv = 2• 1 8 = 0 48 4,5 '

οπότε : s = J4, 75 � 2, 1 8 και

Άρα, μετά την ομαδοποίηση έχουμε μεγαλύτε­

ρη ομοιογένεια

Α9. Οι μισθοί των υπαλλήλων μιας εταιρείας ακολουθούν την κανονική κατανομή με μέ­ση τιμή χ και τυπική απόκλιση s Στο διάστημα (χ - 2s , χ + s) είναι οι μισθοί

1 630 υπαλλήλων της εταιρείας. Πάνω από 2400 ε μισθό παίρνει μόνο το 1 6% των υ­

πάλλήλων, ενώ κάτω από 800 ε παίρνουν μόνο 3 υπάλληλοι. I . Να βρεθεί πόσοι είναι οι υπάλληλοι της

εταιρείας. 2. Να βρεθεί ο μέσος μισθός χ και τυπική

απόκλιση s.

3. Να βρεθεί αν υπάρχει ομοιογένεια μισθών.

4. Να βρεθεί η μείωση όλων των μισθών, κατά το ίδιο ποσοστό ε% , ώστε το εύρος τους να μειωθεί στα 1 800 ε .

Λ ί> σ η

I . Στην κανονική κατανομή, στο διάστημα (x - 2s ' x + s) βρίσκεται το 8 1 , 5% ( 1 3, 5 + 68 = 8 1, 5 , βλέπε σχήμα) των παρατηρήσεων.

Οπότε, αν ν είναι το σύνολο των υπαλλήλων,

τότε έχουμε : ν = 1 ΟΟ · 1 630 = 2000 8 1 , 5

34%

: 2, 35%! • .2 , 35%; ο, ι 5% : ' . • i ! ο, ι 5%

. . . . . . . .. χ - 3s χ - 2s χ - s χ χ + s χ + 2s χ + 3s χ 2. Όμοια, στην κανονική κατανομή , μετά την

τιμή χ + s βρίσκεται το 1 6% ( 1 3, 5 + 2, 3 5 + Ο, 1 5 = 1 6 ) των παρατηρήσεων.

Οπότε αφού το 1 6% των υπαλλήλων παίρνει μισθό 2400 ε , θα είναι: χ + s = 2400 ( Ι )

Οι 3 υπάλληλοι είναι το Ο, 1 5% του συνόλου των 2000 υπαλλήλων. Οπότε, επειδή στην κανο­νική κατανομή , πριν την τιμή χ - 3s βρίσκεται το Ο, Ι 5% των παρατηρήσεων και μόνο τρεις παρα­τηρήσεις έχουν τιμή κάτω από 800 θα είναι: χ - 3s = 800 (2) . Από το σύστημα των ( I ) , (2) βρί-σκουμε : s = 400 και χ = 2000

ΕΥΚΛΕΙΔΗΣ Β' 74 τ.2/61

Page 64: Ευκλειδης Β 74

Μαθηματικά για την Γ Λυκείου

s 400 3. Είναι: CV = - = -- = 20% Οπότε οι μι­χ 2000

ρατηρήσεων, έχουμε sy

1 00 - ε · S .

1 00 σθοί δεν έχουν ομοιογένεια.

4 . Το εύρος των παρατηρήσεων είναι: R = 6S , Οπότε: 6SY 1 00 - ε 6 'Ε , --- · S . τσι εχουμε:

1 00 οπότε για να έχουμε R = 1 800 , πρέπει να είναι

s = 300 . Όμως σε μείωση ε% των τιμών των πα-100-ε 1 800 = -- · 2400 <:::> l&X>=(IOO-ε) · 24 <:::> ε = 25%

100

Α World Con]'erence on ΤΗΕ GENIU S OF ARC HIMEDES

23 Centuries ot Ίnfluence οη tl1e F ields of Mathematics, Science, and Engineering 8-ιο June 2 0 1 0 , Syracuse, Sicily (ltaly)

τtιc aίιη of tlιc Coιιferencc is to bήng togetlιer researclιcrs, sclιolars and studcnts froιn tlιe broad ranges of disciplincs rcfcrήng to HisiOΙJ' of Scicncc and Technology. Matlιenιatics, Meclιanics, and Enginceήng, in an intiιnate, collcgial, and stimιιlating

cnvirornnent to cclcbnιte tlιe personality and contήbutions of Arclιiιncdcs in ιlιc 23tlι ccntury anruvcrsary .

TOPICS 01' TIIE (:ONFERENCE \'lat11eιnntiι:iωι�, ..:ιιgin..::en;, phy!-iίι;ίsts, aιιd 5cie111 ist1-i. along \νίt11 ltisιoriιωs ot' scienc�. ιη;ΗIΙ..:ο.m<ιtίι.::s, <ιηιl cnginceriιιg, <ιr� invitcd ιο ρreseιlt r•φers deιηonstrntiιιg tllt: eηdιιrί ιιg �111<1 continιι i n g influence of Λ.n�ltίιncιlcs.

Oι·iginιιl , unpιιblislιeιt papcι� dcnιonstrating tlιc

sc<luence, progression, or cοηtίιιιιιιnι of Λ.rι.:lli· ιηι:ιlι:<Ηl in llιιc-nc� frοιη :ωcienι ιο ntod�rn tiιηes \.\'1 1 1 be COΠ!:>idcrcιt Γοr rrc��ntation. Λ f'tιll list of possilll� topic.s , dra\VΠ fi·onι tl1� \νorks σf Λrclt iιηcdι:s, ίs listed on tltc Conference \V�b sίιι:. 'Πιc)' inclιιdc 1-Iydrostιιtics, Meclιaιιics, Ylathcιη.ιt icHI Plιysic� . lntegral Calι.::ιιlus, Λ.ncient \.1<ιchin�s & Mechanisnιs , f-Ii stol)' of Matheιηatiι.::s & M nclιines. Teaclιing οΓ ι\.J·ι.::Ιιίιηcdeaιι Prίnciples, P)•cnoιηetry. ΛτclιiιHedcan Legcnds and otlιers. Λlso, l>ι'C(tιιs� οΓ tlιe location of thι.:: Conf'erence, t'hcrC' \νί 11 be a spe.ciί\1 session on Syracιιse at the ιίιηc οΙ' Arcltinιedcs. Please qιιery a Conferencι.:: CΌ-Ch;ιίr ίΓ yοιι arc ιιnccrtιιin wlιether or not your subject litι; wiιhίn thc ιlιeιηι.::s o Γ tlιe Confer�ncι.:: .

CO'\'f<'Ι•:RE�<:E (;0-CΠo\IRS Pι-ol�ssor Yfaι·co Ceccarelli is Dίrector of the 1..-aboraιory οΙ' Rol,otics and Meclιntroιιics at the Uniνersit)' of Cassίno ( ltaly) aιιιl lιas a ψecial inteι·est in tlι� 1-Iistory of Mechanisιn Desίgn. He cr.:-ated tlι� f i:'ΊΌ Μ Μ S;Jηιpοsίιωι on the Historγ q_(

Λfacl1ines and A.fechanί.<ϊιn.'> and edίt5 a Spriιιger book scries, l!ί.�·tυιy ojΊι1echanisnι and Afachine

Science. [ceccarelli@ιιnicas.it]

Prof'essor Stcμlιaιιos Λ. Paipetis of tlι� Departιηeιιt οΓ Me<.:lυι.nical Engineeriιιg & Aeronautics at thc l'ni\'crsit�· of Patπιs (Greece), a specialist on Aιlνanced Composίte Matcrials, ί$ activcly en· gaged in re�earclι οΙ' Ancient Science and Tcclι­no log!', especially of tlιe Homeric Era, whiclι has inspired hίιη to organize seνeral successful S)'Ιη­posia on the sιιbject.

[paipetis@)ιηeclι.ιιpatras.gr]

PllliSE:'Π Α τΙ ΟΝ ηιe- official language οΓ the Conterence \νill be Ειιgl ί:;h. The 5peakers should present their contribution in 15 min \\'ith furtlιer 5 ιnίιι for discus:;ίoιι. Sl ide and overhead projectors, rroj ecto� for peτsonal computers, and VHS video systenι are avaίlable for prcsentation.

PROσ:EDINGS Only papers \\'itlι ;.ιt l�a.<;t one autlιor as a regi�tercd part ίc ipant \\• ί l l be inclιιdcd ίι1 tlιc l'roceedings, vvlιίι.:Ιι will ι,e puω i�hed by Springcr ίιι η printed volurne. Partίcipιιnts wilt receiνe οιιe copy of' tlιc Proceedings upon registration.

INSTR\.J(.TION FOI{ AtJτiJOI{S fιιll papcrs slιould he !-:Ubιηittcd by ι.. ... ncιil to οιιc of' the Conference CΌ·CI1airs by tlιe deadlines l i!>tcd hclo\.v. Pl)l: aιιd Miι:rosoH \Vord docιιιnents are th� pref�rred fonnat. l'he paper fonηat is availί\bl� ίη the webpagc. Eaclι paper is liιηited betwccι1 8 to 1 2 pages. Αιιthο� ar� kindl}· asked to ίndiι:atι.:: ιι

pret'ereηce for poster or oral preNentatioιι, and an indication for a 5uίtable tίtl e sι.-ssioιι.

I>Ko\J)LJNES

SuiJmί!-ision of Fιιll Papers: ί'ίotification oJ' Acceptaιιce:

(ncw)l 5 Νον 2009 10 Dc< 2009

10 Jan 2 0 1 0 Final submission ο Γ Fιιll Paρer!' Official Acc�pt<tnι::e ο Γ Papers: 20 Jan 20 1 0

REGISTRAΊ'ION FEES Participants: Oradιιate Stιιdents:

€3 50

€200 (Αdιnί!-:�ίοιι to all ConΓercnce seNsioιιN, Opening H.eception, and Conference Proι:eedings) tJndergradιιate Students: ε50

€ 1 50 Accoιnpanying Pι:rsoιlf-i: (Adιηission to all Conference se:;sions aιιd Opening 1\eception) Payιηent !-:lιοιιld be ιnade by seιιding a clteck or by baι1k transfer as ίndicated at tlι� Conference web page/Registration.

CONFERENCE LOCA'ΠON

S)τacιιse lies on tlιe east coast of Sicily 5 3 kιη soutl1 ot' C'ataniιι (\.\'\.\'\-\o·.conιune.siraι:u:->a. it). Ιι1 early June Syracuse is sιιnny witlι an aver�ιge teιnperatιιre of 24°C i7 5°f.

ACCOMMODA τΙ ΟΝ

Hotels of all classes are avaίlable at reasonable prices ίι1 Syracιιse. Infonnatioιι will be avaίlable in tlιe conference web page. It is hopcd that Confereι1ce participαιιts will stί\)' ίη Ortygia, tlιe ancient city, and walk to all Coιύerence events.

ΕΥΚΛΕΙΔΗΣ Β ' 74 τ.2/62

T RA VEI, INFORMΛ'flON Syracusc i s accessihlc b�' ;ι ίr, CM. train. <ιηιl Ιηιs.

τJ1e (:�ιΙ;.Jnίιι•I:OI\{i\11:\ΓOSSa . \ίmι)n j� 53 kιn Ιlω1.)1 of Syracusι::, about a onι:-lιour dι·i νc. Ί11e �lι:ΠΙJΥ. lnιcnι<ιtion<tl Λiτrort is 2 1 0 kιn 11ortln\·csι ()f Syr;ιcιιsc, a J'our-l1our dri \'t. front C't:ιιtrnl εuropc b)' ι:ar, tιιkό tlι� Λ3 Λutostrcιda to J{cggίo ('alahri<l and cros� t11ι: Straits of M�ss ina h�' JΊ�πγ ι:ο! I Ώ\\' the Λ 18 toll road to <:ataιιia. contin uing on tl1c S S 1 1 4 to Syraι:usc .

SΊΈERING COMMITT""

Prof. Thoιn<L'ί Ο. (.'ltondro� rGreι:c:e) Prol"'. Alexandcr Λ. Oolo\'in (Rιι.<i.rίa)

Prol'. Harry G. llarήs (U8.4) Pro[ Juιιιι lgιιacio Cu<tdrado Iglcsia� (.<\pcιin) Prof. A1ex:ιndcr Jones (U8Λ) t>rot: Τeιιι1 Koeιsier (Nethe,·/and.r) Ρωf. Edward J. Lar>;on (U8A) Prof. Jlorxt Nowacki (Πernιanγ) Pro[ Aganιenon Ιι Ε. dc Oliνciι·a (IJΙ·a=;t)

Prof. D�ιηosthι.:nes Polyzos rGreec·eJ Prof. C.'esare Rossi (ftaly) tvlr. Ρ.ιοlο l >aιιiele Scίrpo ((Jι·eec·(!.J

Prof. P<ιnayiotis IJ. Siafarik.-ιs ((ίι·eece) nr. nennis L. S inuηs (Unitftd Kingdomj Pro[ Panayiotis Sypsns (Oreecι:) Pro[ Ίlιcodo�ios Ρ. Tassios rGι·eec·e) Prof. 1\ogcr V/i lson (Γarιacla)

Mr. MiclJael Wright r United Kingdonι) ΡrοΓ. Haiclιun Zlιang f{'hina)

Organlzed by • Ίlιe Cίty of' S)'racust (ftalγ) • Ίlι� \Vcstern Greece Region (Gr·eece) • Ίlιe lnstitιιte of Cιιlture anιt Qιιalit)' of' J....il'c (Gι·eece) • The University of C'assiιιo (ltal)-� • 'Πιe University of Patras (Gι·eece) • Th� Hellenic Οpeιι Uniνersiιy (Greece) • Ίlιe e·RDA Innoνation Cι.:ntcr (Gι·eece)

tlnder the patronage of • π:τοΜΜ. The Intenιatίonal Fedcration for tl1e Pron1otioι1 ot� Mechanisιn and Machinc Sciι..'11CC • ΊΠe Hellenic Mathematical Soci�ty (CTι·eece) • European Societ)' for th� HistOI}' of Scίeιιc�

\\'eb Page: lιttp:/·'W\\'\ .. ·.arι:l\iιη�d�s20 lO.org/

Page 65: Ευκλειδης Β 74

------------ Μαθηματικά για την Γ Λυκείου ------------

Κατεύθυνση Γ ' Λυκείου Γενι κά θέματα ορ ίων - συνέχε ιας

Α. Ε ΡΩΤΗΣΕΙΣ ΣΩΣΤΟΥ ΛΑΘΟΥΣ

Ι . Εάν έχουμε : l ί ιη f(x0 + h ) = m Ε �11 τότε l im f(x) = m . h -->0 χ _, χ 0

Χριστιάς Σπύρος

2 . [6ν υπ6ρχει το l i m f(x ) και ισχύει ότι l im f(x) < Ο τότε f(x)<O για κάθε χ Ε ο, . . · � � · � �

3 . Ε6ν υπάρχει το Ι ί ιη (f (χ ) + g ( χ )) τότε πάντοτε θα υπάρχουν τα lim f(x) και l im g(x ) . Χ -> .\ 1 � Χ ->Χο X ---t X0

4. [σχίιει πάντοτε η ισοδυναμία l im f(x) = m Ε 91 <=> l im lf(x) l = lml . Χ -> Χ ο X ---t X0

5. Εάν I i ιη ΙΊ χ ) -= () κ α ι f( χ )>0 κοντά στο χ0, τότε lim -1- = -ι-οο . ' -" . . · χ -> χ ο f(x )

6. Ε ά ν Ι i ιη Γ ι χ Ι = - '- τότε l im (-f(x)) = -oo . .\ -) .'\ 1) 7. Ε ά\' ωτcί. ι\ι:ου\· στο �Η τα όρια l im (f(x) + g(x)) και lim f(x) , τότε υπάρχει στο :η και το

χ->Χο χ -> Χ ο

Ι i ιη � i χ J .

8. Ε ά\ ' Ι ι ιη Γ ι χ ) = Ο , τότε Ι ίιη -1- = +οο ή l im _

Ι_ = -oc .

· • -"" f(x) χ -> χ ο f(x) 9. Ε<'Η' υπάρχουν στο �Η τα Ι i ιη f(x) και Ιίιη g(x) και ισχύει ότι f(x)<g(x) κοντά στο χο, τc'nι:

χ -> ."' ο χ -)Χ ο

απαι)αίτητα Οα έχουμε : l im f(x) < l im g(x) . ·' --> .\ ο χ -> Χ ο

I Ο. Μ ία συνάρτηση f ι=: ίναι συνεχής στο σημείο χ0 Ε Ο,. όταν και μόνο lί ιη f(x0 + l1 ) :-:-. f (x 0 ) . ι, --� ο

1 1 . τ::στω συν<'φτηση f' ορισμένη και συνεχής σε διάστημα Δ με f(x ) * Ο για κάθε χ r ι\ . Τ('η ι : η συνάρτηση Γ διατηρεί σταθερό πρόσημο στο Δ.

1 2 . Έστω Γ μία συνεχής συνάρτηση ορισμένη στο (α, β) . Τότε η συνάρτηση f θα παίρνι::ι μιγ ι ­στη κα ι ελάχιστη τιμή στο (α, β) .

1 3 . Έστω συνάρτηση f συνεχ1)ς σε διάστημα Δ. Τότε η εικόνα f(Δ) δεν είναι απαραίτη τα ()ιι'ι. ­

στημα. 1 4. 'Εστω f συνεχ1Ίς συνάρτηση στο διάστημα (α, β) με Α= l im f(χ) και Β= Ι ί ιη f( x ) . Τότι: το

χ --> <ι ·· χ - - - >rι-σl>νολο τ ψ<!)ν τ η ς f θα είναι απαραίτητα το (Α , Β) ή το (Β , Α) .

Α Π Α Ν Τ Η Σ Ε Ι Σ ΣΤΙΣ Ε ΡΩΤΗΣΕΙΣ ΣΩΣΤΟΥ - ΛΑΘΟΥΣ

Γι [ 2 τ-� I 4 5 6' 7 8 9 1 0 1 ι I 1 2 ! L 3 ! 1 4 1 Σ 1 -Λ--1+-: _Λ __ , -r-:- -Σ--+-Σ-+-Σ-+-Λ--1-Λ--1-Σ----/--Σ-· -��f--Λ-.�-Σ--> -Λ ---L_ __ _L _____ L_ __ _L ____ L_ __ _L ____ L_ __ _L ____ �--�----�--�--�

Β . ΓΕΝ Ι ΚΑ Θ ΕΜ ΑΤΑ ΣΤΑ O P I A ΚΑΙ ΣΤΗ ΣΥΝΕΧΕΙΑ ΣΥΝΑΡΤ ΗΣ ΕΩΝ

Θέμα 1° : Δίνεται η συνεχ1]ς συνάρτηση f : 91 --+ 9{ για

την οποία γνωρίζοι>με ότι xf(x) ;::: χ 2 + ημ2χ

για κάΟε χ Ε 9{ •

α) Να βρεΟεί το f(O). β) Ν α δειχθεί ότι υπάρχει ένα τοι>λάχιστον

χ0 Ε 9{ τέτοιο ώστε f(χ0)=π.

Λύση :

ΕΥΚΛΕΙΔΗΣ Β ' 74 τ.2/63

Page 66: Ευκλειδης Β 74

------------ Μαθηματικά για την Γ Λυκείου ------------

α) Έχουμε : xf(x) � χ 2 + ημ2χ για κάθε χ Ε �Η .

Τότε για χ>Ο θα έχουμε : f(x) � χ + ημ2χ χ

( 1 ) . Αφού η συνάρτηση f είναι συνεχής

στο � θα είναι και συνεχής στο σημείο

χο=Ο. Έτσι θα ισχύει: l im f(x) = lim f(x) = ιim f(x) = f(O) . Επίσης

χ �ο �- χ �ο- x ---tO

ι im (χ + ημ2χ ) = Ο + 2 · Ι = 2 . (Για το όριο χ -->0- χ Ι ί ιη ημ2χ

θέτουμε x=2u) . Τότε λόγω της Χ -->0 ' Χ ( Ι ) παίρνουμε : f(O) � 2 . Για χ<Ο θα έχουμε :

f(x) ::o; x + ημ2χ χ με ι im f(x) = f(O) και

x ---tO

l im (x + ημ2χ ) = 2 . Άρα πάλι λόγω της (1) χ-->0" χ θα έχουμε: f(O) � 2 . Επομένως, τελικά

παίρνουμε : f(0)=2 .

β) Από το α) ερώτημα έχουμε ότι f(0)=2 . Έ­

τσι f(Ο)-π=2-π<Ο. Η υπόθεση ισχύει για

κάθε χ Ε �Η άρα θα ισχύει και χ=2π. Τότε

έχουμε : ι 2πf(2π) � 4π2 +ημ4π <=>f(2π) � 2π <=> <=> f(2π) - π � π > Ο . Θεωρούμε τη συνάρ­

τηση g(x)=f(x)-2π, χ Ε [0, 2π] . Τότε η συ­

νάρτηση g είναι συνεχής στο [0, 2π] ως

διαφορά συνεχών συναρτήσεων και

g(O)g(2π)<O. Άρα λόγω θεωρήματος Bol­

zano προκύπτει ότι υπάρχει ένα τουλάχι­

στον χ0 Ε (0, 2π) (άρα και στο 9\ ) τέτοιο ώστε g(x0 ) = Ο <=> f(x0 ) = π .

Θέμα 2° : Έστω η άρτια συνάρτηση f : � � 9{ η ο­

ποία είναι συνεχής στο σημείο χ0=2 και για , , , . f(x) - 1

την οποια ισχυει οτι lιm 2 = 1 • χ--> 2 (χ - 2)

α) Να βρεθεί το f(2). β) Να δειχθεί ότι η συνάρτηση f είναι συνε­

χής και στο σημείο χ1=-2. ) Ν

λ θ

, , . f 2 (x) - l γ α υπο ογισ ει το οριο : lιm 2 •

χ-->2 ημ πχ

ΛίJση :

α) Θέτουμε Μ(χ)= f(x) - 1

. Τότε έχουμε: ( χ - 2)2

f(x) = M(x)(x - 2)2 + 1 με l im M(x) = 1 . ΆΕτσι x ---t2

παίρνουμε: ιimf(x) = lirr(M(x)(x -2)2 + Ι] = ι x---t2 x---t2 Αφού η συνάρτηση f είναι συνεχής στο ση μείο χ0=2 θα έχουμε : l im f(x) = f(2) .

χ -� 1

Άρα f(2)= 1 .

β) Από α) ερώτημα έχουμε :

f(x) = Μ(χ)(χ - 2)2 + ι με f άρτια συνάρτηση .

Άρα και f(-x) = M(x)(x - 2)2 + 1 με

limM(x) = 1 . x---t2 Τότε ι im f( -χ ) = ι im[M(x )(x - 2) 2 + 1] = Ι x-t2 x---t 2 Θέτουμε -x=u και έτσι όταν χ ___, 2 παίρ­νουμε : u � -2 . Έτσι από την ι im f(-x) = l

x ---t 2 συνεπάγεται : l im f(u) = I . Επίσης από α) LΙ---t-2 ερώτημα έχουμε : f(2)= 1 και αφού f άρτια

συνάρτηση θα έχουμε: και f( -2)= 1. Άρα τελικά παίρνουμε : l im f(u ) = f( -2) = 1 . Ε-ιι---)ο-2 πομένως η συνάρτηση f είναι συνεχής και

στο σημείο χ 1 =-2 .

γ) Για το ζητούμενο όριο έχουμε :

ι . f2 (x) - \ - ι · (f(x) - l )( f(x) + 1 ) ιm 1 - ιm , χ --> 2 ημ-πχ χ --> 2 ημ-πχ

(χ - 2)2 = l im[(f(x) + l )M(x) , ] με Χ-->2 ημ-πχ l im(f(x ) + \ ) = f(2) + Ι = 2 και Ι iιη Μ(χ) = I χ -+ 2 x-t2

Γ , ι · (χ - 2)2

θ ' ιι ια το οριο ηη 1 ετουμε χ = 2 - - . χ --> 2 ημ-πχ π

2 Τότε πχ=2π-u, (χ-2)2=� και ιι � Ο όταν π-χ � 2 . Τότε το όριο παίρνει τη μορφή :

ι . u 2 ι · ι ιι 2

ηη , 1 = ιm-, -1- = u-->O π-ημ- (2π - u) ι ι-->Ο π- η μ -u

ι . I = ιm ---ιι-->ο π2 ( ημu ) 2 π2

ll

Ά ι. (χ - 2)2 ι ·ε

λ . ,

ρα ιm 1 = -1 • τσι τε ικα παιρ-χ--+2 ημ-πχ π-

ιίm f2 (x ) - l

= 2 · 1 · -1 = 2

νουμε: 1 1 , • χ --> 2 η μ -πχ π- π-Θέμα 3° : Έστω η συνεχής συνάρτηση

f: ι ο ' +οο) � (0, + 00) με f(x) * χ 2 για κάθε

ΕΥΚΛΕΙΔΗΣ Β' 74 τ.2/64

Page 67: Ευκλειδης Β 74

------------ Μαθηματικά για την Γ ' Λυκείου ------------

χ 2:: 0 .

α) Να δειχθεί ότι f(x)>x2 για κάθε χ 2:: Ο . β) Να βρεθεί το lim f(x) .

χ->+«>

Λύση : α) Έστω η συνάρτηση g(x)=f(x)�x2

, χ Ε [Ο , +οο) . Τότε η συνάρτηση g είναι συ-

νεχής στο [Ο , +οο) ως διαφορά συνεχών

συναρτήσεων και g(x) :;t Ο για κάθε

χ Ε [Ο , +οο) αφού f(x) :;t x2 για κάθε χ 2': 0 .

Άρα η συνάρτηση g διατηρεί σταθερό πρόσημο στο [Ο , +οο) . Επίσης g(O)=f(O)>O

αφού f(x)>O για κάθε χ Ε [Ο . +χ ) . Άρα

g(x)>O για κάθε χ ε [Ο , +χ ) ή ισοδύναμα

f(x)>x2 για κάθε χ ε [Ο . +χ ) . β) Από α) ερώτημα έχουμε: t'( x } > x c για κά­

θε χ Ε [Ο . - :r ) . τότε θα ισχύει ότι

Ο < -- Ι_ < -1_ �ιια κάθε χ ε [Ο , +οο) . 'Εχουμε : t ( χ ) χ ·

. Ι Ο Ά '

' Ι i ιη Ο = Ο και \ ιm -, = . ρα απο κριτη-\ ----'t--Υ χ -

ρω ;ταρεμβολής παίρνουμε ότι

Ι i ιη -_ 1- = Ο και αφού f(x)>O για κάθε . \ ·-> - • t ( χ ) χ ε [Ο . �χ ) τελικά έχουμε: l im f(x) = +οο .

Χ --7+ΎJ

Θέμα _. . , : Έστω συνάρτηση f : 9i � 9i τέτοια ώστε

2f(χ)+ημf(χ)=3χ για κάθε Χ Ε 9i . α) Να δειχθεί ότι η συνάρτηση f είναι συνε­

χής στο σημείο χσ=Ο. β) Εάν επιπλέον η συνάρτηση f είναι συνε­

χής σε ένα κλειστό διάστημα [α, β] με

f(α)f(β)<Ο, να δειχθεί ότι α, β ετερόσημοι.

Λύση : α) Από υπόθεση έχουμε : 2f(χ)+ημf(χ)=3χ για

κάθε χ ε � . Έτσι για χ=Ο παίρνουμε :

2f(Ο)+η μf(Ο)=Ο :::? ημf(Ο)=--2f(Ο) . Τότε θα

ισχύει: Jημf(O) J = 2 Jf(O) j Όμως

Jημf(O) j � jf(O) j Άρα προκύπτει:

2 jf(O) j � jf(O) j :::? jf(O) j � Ο :::? f(O) = Ο . Επίσης

από την υπόθεση παίρνουμε : 2f(χ) = 3χ � ημf(χ ) . Τότε θα έχουμε : και

2 jf(x)J = J3x �ημf(χ)J � 3 Jx J + Jημf(x)J � 3 Jx J + jf(x)J .

Δηλαδή παίρνουμε ότι

2 jf(x) J � 3 Jx J + jf(x) J :::? jf(x ) J � 3 Jx J :::? -3 Jx J � f(x) � 3 Jx J .

Όμως ι im(�3 Jx j) = ι im(3 Jx j ) = Ο . Επομένως Χ --7 0 Χ --7 0

από το κριτήριο παρεμβολής θα έχουμε : και ι im f(x) = O . Άρα l im f(x) = f(O) = O που

Χ --7 0 Χ --70

σημαίνει ότι η συνάρτηση f είναι συνεχής

στο ση μείο χο=Ο.

β) Εάν θεωρήσουμε τη συνάρτηση f ορισμένη

στο [α, β] , τότε η συνάρτηση f είναι συνε­

χής στο [α, β] και f( α)f(β)<Ο. Τότε από το

θεώρημα Bolzano συμπεραίνουμε ότι θα

υπάρχει ένα τουλάχιστον χ0 ε (α, β) τέτοιο

ώστε f(x0)=0 . Αντικαθιστώντας όπου χ το

χ0 στην υπόθεση, παίρνουμε:

2f(χ0)+ημf(χο)=3χο ή ημ0=3χ0 ή τελικά

χ0=0. Άρα α<Ο<β, δηλαδή α, β ετερόσημοι. Θέμα 5° : 'Εστω η συνάρτηση f με τύπο f(x) = xe'

- l . e'

α) Να μελετηθεί η συνάρτηση f ως προς τη

μονοτονία στο πεδίο ορισμού της .

β) Ν α βρεθεί το σύνολο τιμών της συνάρ­

τησης f. γ) Να δειχθεί ότι η εξίσωση f(χ)=α έχει μο­

ναδική ρίζα για κάθε α ε 9i • ΛίJση : α) Έχουμε: 01= � . Η συνάρτηση f(x) γράφε-

Ι 'Ε ται f(x) = x - --;- . στω χ 1 , χ2 ε �)� με e τ , , , ι ι ι ι χ 1 <χ2 . οτε e ' <e ' =>-:- > -. :::?-. <-. . e'' e'' e'' e''

Επίσης έχουμε θεωρήσει ότι χ 1 <χ2 . Προ­

σθέτοντας κατά μέλη τις δύο τελευταίες α-

, , I I νισωσεις παιρνουμε: χ 1 --. < χ? �- :::? e"' - e'' f(x 1 )<f(x2) . Άρα η συνάρτηση f είναι γνη­

σίως αύξουσα στο �Η . β) Η συνάρτηση f είναι συνεχής στο � ως

πηλίκο συνεχών συναρτήσεων και γνησίως

αύξουσα στο �R . Άρα θα έχουμε : f(A) = ( l im f(x) , ι im f(x)) .

X --7 -'YJ Χ -7 +Χ:

ΕΥΚΛΕΙΔΗΣ Β ' 74 τ.2/65

Page 68: Ευκλειδης Β 74

------------- Μαθηματικά για την Γ ' Λυκείου ------------­Για το l i 1η f(x ) έχουμε ότι χ > --ι

l i 1η f(x ) = l i Ιη (χ - �) = -οο - (+οο) = -οο ' · ·) -ι. .\ - ) -'1.- ex ενιί> για το l i 1η f(x ) έχουμε :

.\ -)+'Υ;

l i 1n f(x ) = l i 1η ( χ - �) = +co - Ο = +οο . Άρα , > i - 1_ .\ -H -.f, e · f( A ) = (-oo , +οο) = �Η .

γ) Θεωρούμε τη συνάρτηση g(x)=f(x)-α,

χ Ε �Η . Η συνάρτηση g(x) είναι συνεχής

στο �Η ως διαφορά συνεχών συναρτήσεων. ' Εστω χ ι , χ2 Ε �Η με χ ι <χ2 . Τότε f(x ι )<f(x2)

( αφοί> η συνάρτηση f είναι γνησίως αύ­

ξουσα στο �Η ) άρα και f(χ ι )-α<f(χ2)-α ή

g(x 1 )<g(x2) . Δηλαδ1Ί και η συνάρτηση g

είναι γνησίως αύξουσα στο �Η . Αφού η

συνάρτηση g είναι συνεχής και γνησίως

αύξουσα στο �Η θα έχουμε:

g( A ) = ( l i 1η g(x ) , l im g(x)) = ( Ιίιη (f(x) - α) , Χ ·-'> ··η_· .\ -)-1-Χ

.Χ ->-':1':

I i 111 ( f ( χ ) - α)) = ( --α:J - α , +οο - α) = \ - - -> •- ·ι. = ( -ω , +οο) = �Η . Επομένως, η συνάρτηση g

ως γνησίως αύξουσα στο �Η θα έχει μία το

πολί> ρίζα στο �Η και αφού το σύνολο τι­

μών της είναι το �Η θα έχει και μία τουλά­

χιστον ρίζα στο �Η . Άρα η συνάρτηση g

έχι:ι μία ακριβώς ρίζα στο �Η για κάθε τιμή

του α ε �Η .

Θi:μα 6" : · Ε στ ω η συνάρτηση f με τύπο

f(x) = ln x + -Fx" . α) Να δειχθεί ότι υπάρχει η αντίστροφη

συνάρτηση Γ1 της f. Γ1 ( ) - 1

β) Να βρεθεί το l im Υ . y -> ι y - (ln x + l )

Λi>ση : α) Έχουμε Dι= (Ο , +οο) . Έστω χ ι , χ2 Ε (Ο ,

+rι.J) με χ ι <χ2 . Τότε έχουμε ότι \η Χ 1 < \η Χ 2 καΟ<i)ς και ότι Fι < F . Προσθέτοντας

κατά μέλη τις δύο ανισότητες παίρνουμε \ η χ . + Fι < \η Χ 2 + ;-;;:; . Δηλαδή προκύ-

πτει ότι f(x 1 )<f(x2) . Επομένως η συνάρτη­ση f είναι γνησίως αύξουσα στο (Ο , +οο) άρα και " 1 - I " στο (Ο , +οο) . Έτσι ορίζε-

ται η συνάρτηση Γ1: f(Α) � 0 1 • Η συνό.ρ-

τηση f είναι συνεχής και γνησίως αύξουσα στο �Η Άρα t'(A) =( li 1η f(x)

' ,ο·

lim f(x )) = ( liιη(lηx+Fx) , l i 1η ( \ η χ + J�-)) =' X -'H--.r. .\---)()" Χ > -Ι- -ι

( -οο , +οο) = �Η και παίρνουμε ότι ορ ίζεται η

συνάρτηση Γ 1 : �Η � (Ο , +οο) . β) Για το ζητούμενο όριο θέτουμε y=f(x) . Τό­

τε Γ\y)=χ και y-( lnx+ I )= Γχ -1 . Για να

βρούμε που τείνει το χ όταν y -> I πρέπι:ι να λύσουμε την εξίσωση t(x)= l (η οπο ία

μάλιστα πρέπει να έχει μοναδική λύση στu (Ο , +oc) ) . Θεωρούμε τη συνάρτηση

g( Χ )=f( Χ)- ] με Χ Ε ( 0 , +οο) Τ ότι: g( l )=f( l )- 1 =0. Άρα το χ0= \ είναι μ ία Πf)Ο­φανής ρίζα της g( χ) . Επίσης εάν χ ι , χ2 ε ( Ο , +οο) με χ 1 <χ2 θα έχουμε f(x ι )<f(x2) (αφού

η συνάρτηση f είναι γνησίως αύξουσα στu (Ο , +οο) ) άρα και f( χ 1 )-- 1 <f( xz}-1 . Δ ηλαδιΊ

παίρνουμε g(χ ι )<g(x2) . Άρα και η συνάρ­τηση g είναι γνησίως αί>ξουσα στο ( Ο , +οο) . Επομένως η εξίσωση g(x)"=O Θα ι':χι:1

μία το πολύ ρίζα στο (0 , +οο) . 'Ετσι προ­

κύπτει ότι το χ0= 1 είναι μοναδική ρίζα της

g(x) . Άρα f(x)= l μόνο για χ= \ . Δηλαδ1Ί

χ � Ι όταν y � Ι . Αρκεί πλι:ον να δειχΟι : ί ότι υπάρχει το

I . Γ1 (f(x )) - l 1 111 Γ . χ -> 1 \η χ + ν χ - ( \ η χ + l ) Για το όριο αυτό έχουμε:

I . f- 1 (f(x)) - 1 ι · χ - 1 ιm = 1 111 -- = χ -> Ι \η χ + fx - ( Ιη χ + I ) χ - > ι Γχ -- I

= I im ( χ - l )(fx + I ) = Ι ί ιη (fχ + I ) = 2 . χ -> 1 χ - 1 ) χ ---> 1 Άρα τελικά παίρνουμε ότι

. r- • (y) - 1 . Γ 1 ( t'( χ )) - 1 _ Ι 1 1η = Ι ι 1η - -2 . y-> 1 Υ - ( \η χ + I ) Χ -> 1 \η χ + Γχ - ( \η χ + I ) Θiψο: 7" : Έστω συνάρτηση f : 9t � 9t τέτοια ώση;

χ2 - 2 � f(x) � χ 2 - J για Κάθε Χ Ε 9t • I

α) Να υπολογισθεί το όριο l im x2t" (-) . . χ -> 0 χ

β) Να βρεθεί το lίm f(x) . χ � +οο

ΕVΚΛΕΙΔ ΗΣ Β' 74 τ.2/66

Page 69: Ευκλειδης Β 74

------------- Μαθηματικά για την Γ Λυκείου -------------γ) Να δειχθεί ότι Ιίm(συv-'χ + ημχ - 3)f(χ) = --οο .

Χ--+t<Χ:ι

, \ {J ση : α) Από την υπόθεση , αντικαθιστώντας όπου χ

I το (με χ =ι- Ο ) παίρνουμε ότι

χ I J I I 2

, θ (-)- - 2 � f(-) � (-) - 1 για κα ε χ ε �Η * . χ χ χ

Πολλαπλασιάζουμε την aνίσωση με χ2 και

, , , , ' f I , ετσι προκυπτει οτι Ι - 2 χ - � χ - (-) � Ι - χ - . χ

Όμως l i ιn( J - 2χ 2 ) = l i ιη( l - x c ) = I . Άρα από Χ -7 0 X -t ! 1

κριηΊριο παρεμβολ1Ί ς παίρνουμε ότι

Ι i ιη x 2 f(_!._) = I . \ >0 χ β) Από υπόθεση ισχύει χ 2 - 2 � f(x ) � χ 2 - I

για κάθε χ ε �Η Όμως έχουμε

Ι ί ιη ( χ : - � ) = Ι ί ιη ( χ 2 - l ) = +οο . Άρα λόγω \ ) • · Χ. ιφιτηριου παρεμβολής παίρνουμε και Ι ί ιη t'( χ ) = +χ; .

γ) Γ\'< •φίζουμε <)τι συvχ � I . Άρα και συν3χ � I .

Επίσης ημχ � I . Τότε συν� χ + ημχ < 2 (η

ισι'ηη τα δεν ισχύει ποτέ αφού το συνχ και

το η μ χ δεν μπορούν να γίνουν ταυτόχρονα

μ ο\'ιJ.δα ) . Έτσι παίρνουμε

σι Ιν ' χ + ημχ - 3 < - 1 => -(συν' χ + ημχ - 3) > 1 .

Λ "ό μ η , από την υπόθεση έχουμε ι · ( v.: ) 2 χ 2 - 2 (με f(x)>O και χ2-2>0 όταν

χ -� +CΌ ) . Πολλαπλασιάζοντας κατά μέλη

τις δύο τελευταίες ανισώσεις έχουμε ότι

-(συν' χ + ημχ - 3)f ( x ) > χ 2 - 2 =>

=> (συν' χ + η μχ - 3 )f (χ ) < -( χ 2 - 2) .

' Ετσι παίρνουμε ότι I I - --, - < , < 0 με

χ - - 2 (συν' χ + ημχ - 3)f(χ )

Ι ί ιη - (-J 1-) = Ι ί ιη Ο = Ο . Επομένως από Χ > Η- Χ - _ 2 Χ -·Η-'"1.'

το κριτήριο παρεμβολής συνεπάγεται :

Ι ί ιη , I

= Ο με ' " ' (συν ' χ + ημχ - 3)f( χ )

(rn >v\ + ημχ - 3)f(χ) < Ο όταν χ � +οο . Άρα

πλικά έχουμε : Ιίιn(σw'χ+ημχ-3)f(χ) =-ω. x----+trrι

Θ , {)Ο εμα ο : Έστω συνάρτηση f: � � � τέτοια ώστε f 2 (x) + f(f"(x)) = 4 για κάθε χ ε � και f(2)= 1 .

α) Εάν η συνάρτηση f είναι συνεχής στο

σημείο χ0=1 να υπολογισθεί το όριο

lim(f(x) - 3)ημ(-1-) .

χ -> 1 χ - 1

β) Να δειχθεί ότι η συνάρτηση f δεν είναι συνεχής στο [ 1 , 2 1 .

Λ ίJση : α) Από την υπόθεση , αντικαθιστιί)ντας όπου χ

το 2 θα έχουμε ότι f'2 (2) + f (f (2 ) ) = 4 ll

1 + f( I ) = 4 � f( l ) = 3 . Τότε αφού η συνcφ­

τηση f είναι συνεχής στο σημείο χο= I παίρνουμε : Ι ί ιη f(x ) = f( l ) =3 . χ --� 1

Έστω Κ(χ) = (f(χ) -3)ημ(-1-) . χ - \ Τότε IK(x) l = Ι (f(χ) -3)ημ(-1-� =

χ - 1 1 = lf(χ) -� ιημ(-� )I � �f(x) -� .

χ - 1 Άρα IK(x ) l � lf(x ) - 3 1 � � - lf( x ) - 3 1 � Κ(χ ) � l f( x ) - 3 1 με

�ί ι�Ί ( - lf(x ) - 3 1 ) = Ι} ι�ψ( χ ) - 3 1 ) = Ο .

Επομένως λόγω κριτηρίου παρεμ βολ1Ίς Ωα έχουμε : και Ι ίιη Κ(χ ) = Ο όπου

.'\ · ·-> 1

I Κ( χ ) = (f( x ) - 3)ημ(-) . χ - I

β) Έστω ότι η συνάρτηση f είναι συνεχ1Ίς στο

[ 1 , 2] . Θεωρούμε τη συνάρτηση g( x)=2

f(x) με χ ε [ l , 2] . Τότε η συνάρτηση ιs ι : ί ­ναι συνεχής στο [ 1 , 2] ως διαφορά συνι:­

χών συναρτήσεων με g( I )=2- -f( 1 )=2--3=

-1 <Ο και g(2)=2-f(2)=2- - l = 1 >0. Άρα έχου­

με: g( 1 )g(2)<0. Τότε από το θεώρημα Bol­

zano συμπεραίνουμε ότι υπάρχει ένα του­

λάχιστον χ 0 ε ( 1 ,2) τέτοιο ώστε g( χ , ) = Ο .

δηλαδή f(x 0 ) = 2 . Αντικαθιστούμε στην υ­

πόθεση όπου χ το χ0. Τότε έχουμε: 4+f(2 )=4, δηλαδή f(2) = Ο . Άτοπο αφού f(2 )= \ . Επο-

μένως η συνάρτηση f δεν είναι συνεχ1Ίς στο

[ I , 2] .

ΕΥΚΛ Ε ΙΔ Η Σ Β' 74 τ.2/67

Page 70: Ευκλειδης Β 74

Η στήλη αυτή έχει ως στόχο την ανάπτυξη μαθηματικού διαλόγου. Φιλοδοξούμε να συμμετάσχουν όλοι όσοι έχουν ένα γενικότερο ενδιαφέρον για τα Μαθηματικά και έχουν πρωτότυπα θέματα να

προτείνουν. Με χαρά περιμένουμε νέες συνεργασίες. Επιμέλεια: Γιάννης Στρατής - Βαγγέλης Ευσταθίου

Εφαρμογές του πρώτου Θεωρήματος του Πτολεμαίου

I . Να υπολογισθούν οι διάμεσοι τριγώνου από

τις πλευρές του.

Λύση Έστω το τρίγωνο ΑΒΓ στο οποίο ισχύουν:

ΒΜ = ΓΜ = � . Θεωρούμε το ισοσκελές τραπέζιο 2

ΑΒΜΔ και ΓΕ// = ΔΜ = ΑΒ = γ

Γ

ΑΕ-ΑΔ = ΔΕ = � ( 1 ) 2

Το ΑΒΜΔ είναι ισοσκελές τραπέζιο άρα εγ­

γράψιμο σε κύκλο. Εφαρμόζουμε το Θ. του Πτο­

λεμαίου . Ήτοι:

(ΑΔ) ·� + γ2 = δ� ΑΔ = 2(μ�α- γ2 )

(2)

Ομοίως το ΑΒΓΕ είναι ισοσκελές τραπέζιο και

εγγράψιμο σε κύκλο.

Άρα: (ΑΕ) · α + γ2 = β2

Οι 1 , 2 και 3 μας δίνουν.

Βασίλειος Φρούντζος

β2 - γ2 2(μ� - γ2 ) α .:....____;__ - = - � α α 2

μ� = ��2(β2 + γ2 ) - α2

μ� = l_�2(α2 + / ) - β2 2

? 1 I ' ' ' μ� = 2 ν 2(α- + β- ) - γ-

2. Να υπολογισθούν οι διχοτόμοι τριγώνου από

τις πλευρές του.

Λύση Έστω το τρίγωνο ΑΒΓ στο οποίο ισχύουν :

Λ Λ ΓΕ//ΔΜ = ΑΒ = γ, ΓΑΜ = ΒΑΜ , ΑΜ είναι δ ιχο-

Γ ΓΜ = � = ΕΔ

β + γ

ΒΜ = _!!]__ β + γ

Β

( 1 )

(2)

ΕΥΚΛΕΙΔΗΣ Β' 74 τ.2/68

Page 71: Ευκλειδης Β 74

Το Βήμα του Ευκλείδη

Το ΑΒΜΔ είναι ισοσκελές τραπέζιο, άρα εγ-γράψιμο σε κύκλο.

Εφαρμόζουμε το Θ. του Πτολεμαίου, οπότε βρίσκουμε :

δ2 = (ΑΔ) · (ΒΜ) + γ2 , δ: = (ΑΔ) · _!!]__ + γ2 α β + γ

ΑΔ = (δ� - γ2 ) · (β + γ) (3) α γ Το ΑΒΓΕ είναι ισοσκελές τραπέζιο, άρα εγ­

γράψιμο σε κύκλο. Επομένως:

(ΑΕ) · α = β2 - γ2 :::::> ΑΕ = β2 - γ2 (4) α

� = ΔΕ = ΑΕ - ΑΔ (5 ) Από τις 1 , 2, 3 , 4 και β + γ . αβ β2 - γ2 ( 2 2 ) (β + γ) 5 βρισκουμε : -- = - δα - γ --β + γ α αγ

� : γ [ (β + γ)2 (β - γ) + γ(β + γ)z _ α2β] ο = = α (β + γ)2

βγ [ (β + γ)z - α2 ] (β + γ)2

δ2 = βγ(α + β + γ) (β + γ-α) θέτουμε α (β + γ)2

α + β + γ = 2τ - α + β + γ = 2(τ - α) ') δ� = ---Jβγτ(τ - α) ομοίως β + γ

δμ = -2-Jαγτ(τ - β) δγ = -2-Jαβτ(τ - γ) α + γ · α + β

3 . Να υπολογισθούν τα ύψη τριγώνου από τις

πλευρές του.

Λύση Έστω το τρίγωνο ΑΒΓ. Στο σχήμα το ΑΓΒΔ

είναι ισοσκελές τραπέζιο εγγράψιμο σε κύκλο. Εφαρμόζουμε το Θ. του Πτολεμαίου οπότε βρί-σκουμε :

Ε<ΑΒΓΔ> = _!_ · (ΒΓ)(ΑΔ) = �(s - α)(s - χ)(α - γ)2 (2) 2

Β όπου

βz _ 2 α2 + β2 + 2αγ _ γz 2s = α + χ + 2γ = α + γ + 2γ = _ ___:__ __ .:._____:__ α α ? β2 ? 2 ? β ? ? α- + - γ- + αγ α- + - - γ - (3) s - γ = 2α - γ = 2α

? ? ? β" ? ? 2 s - α = α- + β- - γ- + 2αγ - α = - - α- - γ- + αγ (4) 2α 2α

s - x α2 + βz - γ2 + 2αγ β2 - γz 2α α

? β? ? = α- - - + γ- + 2αγ

(5 ) 2α Η (2) γράφεται :

2α 2α 2α

uα = 21α J(α + β + γ)(α + β - γ)(α - β + γ)( - α + β + γ) ,

2τ = α + β + γ

uα = �Jτ(τ - α)(τ - β)(τ - γ) α uβ = �Jτ(τ - α)(τ - β)(τ - γ) β

u y = �Jτ(τ - α)(τ - β)(τ - γ) . γ

4 . Να υπολογιστούν οι εξωτερικές διχοτόμοι

τριγώνου από τις πλευρές του ( ΑΒΓ , α, β, γ) .

ΕΥΚΛΕΙΔΗΣ Β ' 74 τ.2/69

Page 72: Ευκλειδης Β 74

ΛίJση Το Βήμα του Ευκλείδη

Θεώρημα Stewart.

Έστω το τρίγωνο ΑΒΓ στο οποίο ισχύουν: Λ Δ ΑΔ = εξωτερική διχοτόμος της Α του ΑΒΓ

που τέμνει την ΓΒ στο Δ. ΓΔ // ΑΖ, ΑΒ // = ΔΖ = γ, ΑΓ // ΕΖ = ΔΗ = β, ΔΕ = α = ΒΓ, ΒΔ = ΑΖ = ΑΗ+ΖΗ ( Ι )

Δ

Στο σχιΊμα ισχύουν: Α Β ΒΔ γ - - --Α Γ ΓΔ ' β

ΒΔ , ΒΔ = _!!]__ (2) α + ΒΔ β - γ Το ΛΓΔΗ είναι ισοσκελές τραπέζιο . Άρα εγ­

γράψιμο σε κύκλο . Εφαρμόζουμε το Θ του Πτολε­μα ίου ι) το ι :

( ΑΗ ) · ( α + ΒΔ ) + β2 = μ� => ΑΗ = μ� - β2 (3 ) α -t- ΒΔ

Ομοίως για το εγγράψιμο σε κύκλο ισοσκελές τραπέζιο ΔΕΗΖ ήτοι :

( ΖΗ ) (ΔΕ ) + γ2 = β2

ΛπίJ τις ( I ) , (2) , (3 ) , ( 4) βρίσκουμε : Δ�, - β 2 + β2 - γ2 = _!!]__ , απ ' όπου προκύπτει: α + α_y_ _ α β - γ

β - γ

Ομοίως: Δα = ιβ � γ Ι Jβγ ( τ - β ) ( τ - γ ) Δ

β =

-I 2-

1 Jαγ ( τ - α ) ( τ - γ ) α - γ

5. Δίνεται τρίγωνο ΑΒΓ και τυχαίο σημείο

Μ της ΒΓ. Να δείξετε ότι ισχύει η σχέση :

ΑΓ2 • ΒΜ + ΑΒ 2 • ΓΜ = ΑΜ 2 • ΒΓ + ΒΓ · Β Μ · ΓΜ

ΛίJση Έστω το τρίγωνο ΑΒΓ στο οποίο ισχύουν:

ΓΜ = ΔΕ ΓΕ // = ΔΜ = ΑΒ = γ ΔΕ = ΑΕ - ΑΔ ( I )

Το ΑΒΜΔ είναι ισοσκελές τραπέζιο εγγράψ ι ­μο σε κύκλο.

Γ Μ

Εφαρμόζουμε το Θ. του Πτολεμαίου, 1Ίτο ι : (ΑΜ) ( ΒΔ) = (ΑΔ)ΒΜ + (Α Β) (ΔΜ)

(ΑΜ)2 - (ΑΒ)2 = ΑΔ (2) (ΒΜ)

Β

Ομοίως το ΑΒΓΕ είναι ισοσκελές τραπi:ζιο Ι :γ­

γράψιμο σε κύκλο . Άρα:

(ΑΓ) 2 = (ΑΕ)(ΒΓ) + (Λ Β) 2 ΑΕ = ΑΙ�� - Α Β 2 ( J ) ΒΓ

Οι Ι , 2 και 3 δίνουν : ΑΓ2 - ΑΒ2 ΑΜ2 - (Α Β) 2

ΓΜ = --

--'--

-'--

-(ΒΓ)(ΓΜ)(ΒΜ) =

ΒΓ Β Μ

=(ΑΓ)2 · (ΒΜ) - (ΑΒ)2 · (ΒΜ) - (ΑΜ)2 • ΒΓ + (ΛΒ ) -' · 1 3 1 .

ΒΓ · ΓΜ · ΒΜ -t- ΑΜ2 · ΒΓ = ΑΓ2 · ΒΜ + ΑΒ2 [ΒΓ ·-- BMJ ΑΓ 2 · ΒΜ + ΑΒ2 • ΓΜ = ΑΜ 2 · ΒΓ + ΒΓ · ΓΜ · Β Μ

ΕΥΚΛΕΙΔΗΣ Β ' 7 4 τ.2170

Page 73: Ευκλειδης Β 74

Σκέψεις πάνω σε μια ανισότητα Σταύρος Κ. Σταυρόπουλος (Αμαλιάδα)

Η Μαθηματική διαπραγμάτευση του θέματος που ακολουθεί γίνεται με αφορμή το 3" πρόβλημα του διαγωνισμού ΑΡΧΙΜΗΔΗΣ 2009 . Θα χρησιμοποιήσουμε την ανισότητα Schur και μερικές χρι']σψι.:..; ταυτότηπς από τις οποίες προκύπτει η πρόταση : Αν α, β, γ>Ο και α+β+γ=l , τότε ισχύει:

J + 9αβγ αβ + βγ + γα � .

4 Μ ε την βο1)θεια αυτής της αντεπαγωγής που έχει προταθεί και ως άσκηση στον Ευκλείδη Β ' (τεύχος 69-2008 ,σελίδα 79, άσκηση 1 44), έχουμε μια μέθοδο απόδειξης ασκήσεων της μορφής του 3°" προ[)λι) ­ματος ΑΡΧΙΜΗΔΗΣ 2009 .Στην συνέχεια δίνουμε μια βελτιστοποίηση του προβλήματος αυτού . Λ ) Οι δύο πρώτες ταυτότητες από τις τρεις που ακολουθούν αποδεικνύονται εύκολα. Θα αναφερθούμε μόνο στην απόδειξη της τελευταίας. α) Αν Α=(α+β)(β+γ)(γ+α), τότε: α 1 ) Α=α\β+γ)+β2(γ+α)+γ2(α+β)+2αβγ α3) Α=αβ(α+β)+βγ(γ+α)+γα(γ+α)+2αβγ α2 ) Ac= α(β2+/)+β(/+α2)+γ( α2+β2 ) -+-2αβγ α4) Α = α2 ( β + γ ) + α (β2 + γ2 ) + βγ (β + γ ) + 2αf{γ . β) ( α+β+γ )( αβ+βγ+γα )=( α+β)(β+γ )( γ+α )+αβγ.

γ) ( α + β - γ) ( α - β + γ) ( -α + β + γ ) = α 2 ( β + γ ) + β2 (α + γ) + γ 2 ( β + α) - α3 - β3 - β3 - 2αβγ

Λπ{Jδε ιξη : ( α + β - γ ) ( α - β + γ ) ( -α + β + γ) = (α + β - γ) [γ2 - (α - β )2 ] = (α + β - γ) {γ2 - α2 - β2 + 2αβ ) =

"' ( α + β ) / - ( α -r β ) ( α c + βc ) + 2αβ (α + β ) - / + γ (α2 + β2 ) - 2αβγ = (α, ) '--= γ : ( α + β ) + γ ( α c + β 2 ) + α β ( α + β ) - (α + β ) ( α2 + β2 - α β ) - γ3 - 2αβγ = ( α , ) = αc ( β + γ ) + β 2 (α + γ) + γ2 ( β + α ) - α3 - β3 - γ3 - 2αβγ.

R ί Θα παραθέσουμε ένα θέμα που περιέχει την βασική ανισότητα του Schur (της οποίας η απ6δαξη α­ιrοη:ί.εί τεχνικι) για τη λύση πολλών ασκήσεων) και μια ισοδύναμη μορφή της για ν= Ι η οποία πλαισιω-μi:νη με την υπόθεση α+β+γ= l μας δίνει την ανισότητα αβ + βγ + γα :::; 1 + :αβγ , Θ{μ: . :\ α αποδειχθούν οι ανισότητες που ακολουθούν:

α) Αν α,β,γ>Ο, νε Ν τότε ισχύει: α•(α-β)(α-γ)+β.(β-α)(β-γ)+γ.(γ-α)(γ-β)2:0 (Schur)

β ) Α ν ο,β, γ>Ο, τότε ισχύει: ( α+β-γ )(α-β +γ)( -α+β+γ ):Sαβγ

Ί ι Α ν α,β, γ>Ο και α+β+γ= 1 , τότε ισχύει: α β + β γ + γα � 1 + :αβγ

.

ι j·Jση : α) Ονομάζουμε S το πρώτο μέλος. Έστω γ :::; β :::; α, τότε : (α-β)(α-γ)2:0 και (γ--α)(γ- β )2:0 οπι'>η; S2:αv( α--β)( α-γ)+βν(β-α)(β-γ)2:βν( α-β)( α-γ)+βν(β-α)(β-γ)=βν( α-β)( α-γ-β+γ)=βv( α--β )22:0.

1 1 ανισότητα του Schιιr για ν= Ι γίνεται: α(α-β)(α-γ)+β(β--α)(β-γ)+γ(γ-α)(γ-β)2:0 q α[α'- -α(β+γ) ι βγ ]+β[β2---β( γ+α)+γα ]+γ[ γ2 -γ( α+β)+αβ]2:0qα3-α2(β+γ)+αβγ+β3 -β2( α+γ )+αβγ--ιy1-γ2( α+β )+αβγ2:0 <: ->αβγ2:α2 - (β Ι-γ )+β2( α+γ)+/( α+β)-α3-β3 -γ3-2αβγqαβγ2:( α+ β-γ)( α-β+γ)( -α+β+γ) ( Ι) . γ) Θα χρησιμοποιήσουμε τους aξιοσημείωτους πολλαπλασιασμούς: i ) ( x + α)( χ + β) = χ2 + ( α + β) χ + α/�. i i ) ( χ + α) ( χ + β ) ( χ + γ ) = χ 3 + ( α + β + γ ) χ 2 + ( α β + β γ + γα) χ + αβγ. ( Ι ) c::> ( Ι - 2γ ) ( ι - 2β ) ( ι - 2α) :::; αβγ =::;, 1 3 - ( 2α + 2β + 2γ ) 1 2 + ( 2β2γ + 2β2α + 2α2γ ) Ι - 8αβγ :::; αβγ =::;,

1 - 2 ( α + β + γ) + 4( αβ + βγ + γα) - 8αβγ :::; αβγ =::;, 1 - 2 - I + 4( αβ + βγ + γα) :::; 9αβγ =::;, αβ + βγ + γα :::; 1 + 9αβγ _ 4

Ιl αρ ατη ρ ι) σης : I ) Η ισότητα και στις τρείς προηγούμενες ανισότητες ισχύει όταν και μόνον α=β=γ 2) Λ ν υποθέσουμε ότι α,β,γ2:0 τότε η ισότητα ισχύει όταν και μόνον: α=β=γ 1) ένας από τους α,β,γ είναι το μηδί:ν και οι άλλοι δυο είναι ίσοι \ 'Ι ε αν{ι.λογο τρ6πο i:zουμι: : Αν α,β,γ>Ο και α+β+γ= l , να αποδειχθούν οι ανισι'>τψες: α) 5 ( αβ + βγ + γα) + 9αβγ :::; 2, β) αβ + βγ + γα - 2αβγ � ;7 , γ) s ( α3 + β3 + γ3 ) + l 2αβγ 2 3 ( αβ + βγ + γα) -

ΕΥΚΛΕΙΔΗΣ Β ' 74 τ.217 1

Page 74: Ευκλειδης Β 74

------------ Εφαρμογές της ανισότητας Schur ------------' δ ):. ) Α ' αβ βy 2 -9αβγ Ι ' ' αβ βy l +9αβγ ' ' δ θ ' Απο ειι.,η : α ρκει + +γα::; 5 . σχυει, ομως + +γα::; 4 . Άρα αρκει να απο ειχ ει ότι:

l +9αβy < 2-9αβy, ή 5( 1 +9αβγ) ::s;4(2-9αβγ) , ή αβγ:::;_!_, που ισχύει καθότι: l = α+β+γ ;::: 3{αβΥ ::::>αβγ ::s;_2_, 4 5 27 27 Για τις περιπτώσεις β, γ, εργαζόμαστε ομοίως λαμβάνοντας επί πλέον υπ' όψη την ταυτότητα:

( α + β + γ)3 = α3 + β3 + γ3 + 3 (α + β) (β + γ) (γ + α) = α3 + β3 + γ3 + 3 ( α + β + γ) (αβ + βγ + γα) - 3αβγ. Γ) Χρησιμοποιώντας τα προηγούμενα θα αποδείξουμε το 3° θέμα του διαγωνισμού της Ε .Μ .Ε . ΑΡΧΙ­ΜΗΔΗΣ 2009 και θα παρουσιάσουμε μια βελτιστοποίηση του . Το 3° θέμα του διαγωνισμού της Ε.Μ.Ε. ΑΡΧΙΜΗΔΗΣ 2009 Αν x,y, z�O και x+y+z=2, τότε να αποδείξετε ότι x2y2+y2z2+z2x2+xyz::;1 . Για ποιες τιμές των x,y,z ι­σχύει η ισότητα;

Λί>ση : x + y + z = 2 <=> � + _r + � = 1 ( 1 ) . Θέτουμε : � = α,Ι = β,� = γ , οπότε 2 2 2 2 2 2 χ = 2α, y = 2β, z = 2γ και ( 1 ) <=> α + β + γ = Ι , άρα αβ + βγ + γα :::; l + 9αβγ ( i) 4 'Εχουμε : x2y2 + y2z2 + z2x2 + xyz = 1 6( α2β2 +βγ + γ2α2 ) +8αβy = 1 6[( αβ +βγ + γα)2 - 2αβy( α+ β+ γ) ]+ 8αβγ =

( )2 ( )2 ( 1 + 9αβγ )2 = 16 αβ + βγ + γα - 32αβγ + 8αβy = 16 αβ + βy + γα - 24αβy :::; 1 6 4 - 24αβγ =

= ( 1 +9αβγ)2 -24αβy=81α2βγ -6αβy+ 1 = 3αβγ(27αβγ-2) + I ::; ι ( i i ) , αφού (αβΥ < α+β+γ =� =>αβγ:::;_2_ <2. 3 3 27 27 Η ισότητα εξασφαλίζεται μόνο όταν οι ( i ) , ( i i ) ισχύουν ως ισότητες, δηλαδή :

? 2 ? 2 2 ? χ = 0 y=O z = Ο ( α = Ο J ( β = Ο J ( γ = Ο J x-y +y-z +z x-+xyz= 1 <=> 1 ή Ι ή 1 <=> ή ή . β = γ = z γ = α = z α =β =z ι = z = l) (z = x = l) (x = y = l ] • Βελτιστοποίηση του προηγούμενου θέματος. Αν x,y,z�O και x+y+z=2, τότε να αποδείξετε ότι

2 2 2 2 2 2 1 1 < l Γ

, ' ' χ y + y z + z χ + - xyz _ . ια ποιες τιμες των χ, y, z ισχυει η ισοτητα; 8

Λύση : Θέτουμε χ = 2α, y = 2β, z = 2γ , οπότε: α+β+γ= 1 . 'Ε τ σι έχουμε : α β + βγ + γα ::::; 1 + 9αβγ ( i) , οπότε : 4 ? ? ? ? ? 2 1 1 ( ) ? ? ) ? ? ) [( )2 ( )] x -y- + y-z- + z -x + S xyz = 1 6 α-β- + β-γ- + γ-α- + 1 1 αβγ = 16 αβ + βγ + γα - 2αβγ α+ β + γ + l lαβγ

( ) 2 ( Ι + 9αβγ )2 ( ) 2 , , , = 1 6 αβ + βγ + γα - 2 Ιαβγ ::s; 1 6 4 - 2 1αβγ = 1 + 9αβγ - 2 1αβγ = 8 Ια-β-γ- - 3αβγ + 1 =

3αβγ ( 27αβγ - 1 ) + 1 ::::; 1 ( i i ) , αφού αποδείξαμε ότι: αβγ ::::; -1 . Ομοίως η ισότητα εξασφαλίζεται μόνο ό-27 ( α = Ο ] ( β=Ο J ? ? ? 2 ? ) 1 1 ' ταν οι ( i) , ( i i ) ισχύουν ως ισότητες, δηλαδή : x - y- + y-z + z- x - + - xyz = Ι <=> 1 η 1 8 β = γ = - γ=α=-2 2 ( γ = Ο ] 1 ( χ = Ο ) ( y = O ) ( z = O ) 2 ή Ι ή α = β = γ = - <=> ή ή ή x = y = z = -α = β = l 3 y = z = l z = x = 1 x = y = 1 3

Σ ' λ

Ε δ ' 1 1 Ο ' ' β λ ' ' ' χο ιο : πει η xyz ::::; -xyz, oταν x , y, z ;::: , εχει γινει μια ε τιστοποιηση στην προηγουμενη ανισοτη-8

λ ' ' ' ' ' 2 τα με επιπ εον το οτι η ισοτητα ισχυει και οταν χ = y = z = - . 3

ΕΥΚΛΕΙΔΗΣ Β ' 74 τ.2/72

Page 75: Ευκλειδης Β 74

•ο -' . i\\"

Ε υ κλε. i δ n ς n ρ οτ ε. i νε. ι ' \_

«Η καρδιά των μαθηματικών είναι τα προβλήματα και οι λύσεις και ο κύριος λόγος ύπαρξης του μαθη ματικού είναι να λύνει προβλήματα>>. Ρ. R. HALMOS

Επιμέλεια : Γ. Κ. Τ Ρ Ι ΑΝΤΟΣ - Ν . ΑΝΤΩΝ Ο Ω Ο Υ ΛΟΣ

ΑΣΚΗΣ Η 149 (τΕΥΧΟΥΣ 7 1 ) ? ?

Δίνεται η έλλειψη C 1 : χ: + Υ: = I , α > β > Ο και ο α· β-κύκλος C� : χ " + y� = α� . Να βρεθεί σημείο Μ του κύκλου c� τέτοιο, ώστε οι εφαπτόμενες που άγο-νται από το Μ προς την έλλειψη C 1 να είναι τέ-τοιες, ώστε η απόσταση της αρχής Ο( ο, ο) των αξόνων από την ευθεία που ορίζουν τα σημεία ε­παφής τους με την C 1 να είναι ελάχιστη . (Προτά­θηκε από τον ΘΑ Ν Α Σ Η ΚΥ Ρ Ι .--\ ΚΟ Π ΟΥ ΛΟ -Αθήνα) Λ ΥΣΗ ( από τον Γ Ι ΑΝΊ\ Η �ΤΑΜΑΤΟ­

Γ Ι ΑΝ Ν Η - Δροσιά Αττικής). Έστω M(x0 , y0 ) E C2 και P(x 1 , y 1 ), N (x � , y� ) τα σημεία επαφής των εφαπτομένων της έλλειψης από το σημείο Μ .. Τότε έχουμε χ � + y� = α" και Μ Ρ : χχ , β 2 + yy , α2 = α2β2 ΜΝ : χχ 2β

2 + yy2α2 = α2β2 Υ

χ

Επειδή το σημείο Μ ανήκει στις ευθείες αυτές οι συντεταγμένες του επαληθεύουν τις εξισώσεις

' β? ? 2β2 τους, οποτε : χ0χ 1 - + y0y, α· = α και β? ' 2β ? Σ , ξ' Χ 0Χ 2 - + y0y2α- = α - . υνεπως, η ε ισωση της

ευθείας ΡΝ ( πολική του σημείου Μ ως προς την έλλειψη) είναι: β2 χ0χ + α2y0y = α2β2 και η από-σταση της αρχής των αξόνων από την ΡΝ :

Θεωρούμε την συνάρτηση α2β2 f(x 0 ) = , - α ::s; χ0 ::s; α , με �(β4 _ α4 )χ � + α6

2β2 ( 4 β4 ) f ' (x ) = α α - Χο Εί-0 [(β4 - α4 )χ � + α6 ]�(β4 - α4 )χ � + α6 ναι: f'(x0 ) = Ο <=> χ0 = Ο , ενώ f'(χ0 ) > 0 <::::> α 2 χ0 > 0 και f'(x0 ) < 0 <::::> -α ::s; x0 < 0 . Άρα η f παρουσιάζει ελάχιστη τιμή για

χ 0 = 0 , ίσο με f(O) = � και y0 = α ή y0 = -α . α Συνεπώς, το ζητούμενο σημείο είναι είτε το Μ(Ο, α) , είτε το Μ(Ο, -α) . Λύσεις έστειλαν επίσης οι συνάδελφοι: Ροδόλφος Μ πόρης - Δάφνη , Β ασίλειος Ν ικολά.κη .;

- Θες/νίκη , Κων/νος Τζαγκαράκη ς - Χανιά.. Β αγ­

γέλης Μ ουρούκος - Αγ ρίνιο, Γιώργος Μ 1Ί τσιο.; -

Ρ άμια Ά ρτας, Γιάννης Η λιόπουλος - Καλαμά.τα,

Φ ίλιππος Σερέφογλου - Μ ελίσια Α πικιΊ ς, Ο ρέ­

στης Κατσάνος - Πρέβεζα, Γιώργος Δελη στάθης ­

Κ άτω ΠαηΊ σια, Γιώργος Τσαπακίδη ς - Κυψέλη

Α ι τολοκαρνανίας.

ΑΣ Κ Η Σ Η 1 50 (τΕΥΧΟΥΣ 7 1 ) Κατασκευάζουμε διαδοχικά τετράγωνα κοινής κο-ρυφής Ο, ώστε η διαγώνιος του προηγουμένου να είναι πλευρά του επομένου. Α ν Sv είναι το εμβα­δόν του επιπέδου χωρίου που καταλαμβάνουν τα ν πρώτα τετράγωνα, να δειχθεί ότι ισχύει:

ΕΥΚΛΕΙΔΗΣ Β' 74 τ.2/73

Page 76: Ευκλειδης Β 74

-------------- Ο Ευκλείδης προτείνει . . . --------------

S . 1 = 2S . + cl και να εξαχθεί ο τύπος του S,. συναρ-1+ Ι· 2

τΙ1σει των ν,α, όπου α είναι το �11Ίκος της πλευράς

του πρώτου τετραγιί)νου . (Προτάθηκε από τον συ­

νάδελφο Ν Ι ΚΟΛΑΟ Β Α Δ Ι ΒΟ Υ Λ Ι Ι · - Άρτα )

ΛΥΣΗ ( Από τον συνάδελφο ΒΑ Σ Ι Λ Ε Ι Ο Ν Ι ΚΟ­

ΛΑΚ Η - Θεσ/νίκη )

Συμφά)νως προς το πρόβλημα, διαπιστ<ί)νεται ότι

κάθε νέο τετράγωνο καλύπτει την μισή επιφάνεια

του αμέσως προηγουμένου τετραγώνου . Άρα, για

να υπολογίσουμε το εμβαδόν Sv του επιπέδου χω­

ρίου που δημιουργείται από τα ν πρώτα τετράγω­

να, αρκεί να προστεθεί στο εμβαδόν του τελευ-

- Ράμια Ά ρτας, Ροδόλφος Μ πίφη ς - Δ άφνη .

ΑΣΚΗΣΗ 1 5 1 ( ΤΕΥΧΟΥΣ 7 1 )

Σε τρίγωνο Α ΒΓ σημείο Δ χωρίζει την πλευρά Β Γ

σ ε μέσο και άκρο λόγο με ΒΔ<ΔΓ . Ομοίως, ση­

μείο Ε την Γ Α με ΓΕ<F:Α . Να δειχθεί ότι η ευθεία

ΑΔ διχοτομεί το ευ01ιγραμμο τ�ο1 μα ΒΕ.

( Προτάθηκε από τον συνc1δελφο Ν Ι ΚΟΛΑΟ ΒΑ­

Δ Ι Β ΟΥ Λ Η - Άρτα ) .

Β

I I

I

I I

I I

Α I

I I

Γ

ταίου τετραγώνου το ήμισυ του αθροίσματος των Λ ΥΣΗ ι Β : (Από τον συνάδελφο Ο Ρ ε : τ Η ΚΑ-εμβαδών όλων των προηγουμένων. Αν είναι α η ΤΣΑΝΟ - Π ρέβεζα )

πλευρά του πρώτου τετραγά>νου, τότε οι πλευρές Από το θεώρημα του Μενελάου στο τρίγωνο Β Ε Γ

των άλλων τετραγιονων είναι : με διατέμνουσα την ευθεία ΔΜΑ έχουμε : ( I )

' I ' ' 4 ' 2 '' ' ' 2 " ·I ? Άρα, S, = - (α- + 2α- + α- + . . . + · α - ) + α-

2

I 2 '. _ 2 • 2 - I 2 2,. _ 1 2 _ I

(3 2,. _ 1 I ) 2 = - · α + α - - · - α

2 2 - 1 2

. 3 · 2 " - 1 2 ν - 1 2 α2 Ετσι , Sv , 1 =

2 α = 3 · 2 α - τ =

Λ ίισεις έστειλαν };π ίσης οι συνάδελφοι:

Κων/νος Τζαγκαράκη ς - Χ ανιά, Γιιο ργος Μ 1Ίτσιος

ΔΒ ΑΓ Μ Ε , ΔΒ J5 - I - · - · - = ι . Όμως - = -- = φ (2 ) Δ Γ Α Ε Μ Β ΔΓ 2

ΑΓ και - = - ( 3 ) . Από την ( I ) βάσει των ( 2) , ( 3 )

ΑΕ φ

. I M C Β λ προκυπτει φ · - · - = 1 => ΜΕ = Μ . Δη . το

φ Μ Β

ζητοίιμενο .

Α

Β Γ

Λ ΥΣΗ 2Β : ( Από τον συνάδελφο Γ Ι Ω Ρ ΓΟ Μ Η ­

ΤΣ Ι Ο - Ράμια Άρτας ) .

, ΔΓ ΑΕ Α Γ J5 - I Ισχυει: - = - = - = -- = φ ( l ) . 'Εστω

ΔΒ ΕΓ Α Ε 2

(ΒΑ Μ)=κ, ( Α Μ Ε)=λ, (ΔΓΕΜ)=μ, ( ΒΜΔ)=ν. ( ΑΔΓ) ΔΓ λ + μ

Είναι: = - => -- = φ (2) . (ΑΒΔ) ΔΒ κ + ν

ΕΥΚΛΕΙΔΗΣ Β' 74 τ.2174

Page 77: Ευκλειδης Β 74

-------------- Ο Ευκλείδης προτείνει . . . --------------(ΑΒΔ) ΑΔ κ + ν ΑΔ __:__--'-- = -- � -- = -- (3) Επειδή τα τρί-(ΑΒΜ) ΑΜ κ ΑΜ

γωνα ΑΔΓ και ΑΕΜ έχουν κοινή την γωνία Α

, (ΑΔΓ) ΑΔ ΑΓ , ισχυει = -- · - και λογω των ( I ),(3 ) (ΑΕΜ) ΑΜ ΑΕ

λ + μ κ + ν , λ , , -- = -- · φ . Απο την τε ευταια ισοτητα και λ κ

την σχέση (2) προκύπτει: λ + μ κ + ν λ + μ -- = -- · --

δηλ. κ = λ . Συνεπώς, ΒΜ=ΜΕ.

λ κ κ + ν

Λύσεις έστειλαν επίσης οι συνάδελφοι:

Ροδόλφος Μ πc') ρη ς - Δάφ, ·η . Β α σίί .ε ι '-• : '\ ιωί.άr-:ΊΕ - Θες/νίκη , ΓιιΊννη -; Σ τ υ μ α ω·. · ι cίη η ; - _' φ ο σ ι ά Α τ-

ΑΣ ΚΗΣ Η 1 52 ( ΤΕΥΧΟΥΣ 7 1 ) Να βρεθεί ποί.υι;),·υμο (( χ ) , 2ν- 1 βαθμού, όπου

ν Ε "Ν • . ι:)στε το ποί.υι!)\'υμο f(x) + I να διαιρείται

με το ( χ - 1 ) ', και το f( x ) - 1 να διαιρείται με το

( χ + I ) ' . Σ τη συνέχεια να βρεθεί ένα τέτοιο, πο­

λυcJ)\'υμο βαθμού I , ένα τρίτου βαθμού και ένα ε­

βδόμου βαθμού .

( Π ροτάθηκε από τον συνάδελφο Ν Ι ΚΟΛΑ Ο Β Α­

Δ Ι Β Ο Υ Λ Η - Άρτα)

Λ Υ Σ Η ( Από τον ίδιο ). Από την υπόθεση έχουμε :

t'( x ) + I = ( χ - l)" p(x) ( I ) και

Γ( χ ) - I = (χ + I ) " q (x) (2). Από τις ισότητες

( I ) , (2) με παραγώγιση έχουμε :

f ' ( x ) = (x - l)" - 1 [νp(x ) + (x - l )p '(x )] (3)

f '( x ) = (χ + l )" - 1 [νq (x) + (χ + l )q ' (x)] (4) .

Από τις σχέσεις (3 ), ( 4) προκύπτει ότι το πολυώ­

νυμο f ' ( x ) , που είναι 2ν-2 βαθμού, έχει παράγο-

ντα το γινόμενο (x - 1) "- 1 (x + l ) v- ι = (χ 2 - l ) v- ι .

Άρα, ισχύει f '(x ) = c 1 (χ 2 - I ) ν- ι , c 1 Ε R , οπότε

Οι σταθερές c 1 , c 2 προσδιορίζονται από τις αρχι-

κές συνθήκες: f(l) = - l και f(- 1) = 1 (6)

Για ν= Ι η (5) δίνει : f(x) = c 1 Jdx + c2 = c1 x + c

Λόγω των σχέσεων ( 6) προκύπτει για λύση το σύ-{c1 + c = -1 {c1 = -1 στημα: <:::> , οπότε -C1 + C = 1 C = 0

f(x) = -χ . Από την (5) , για ν=2, προκύπτει ότι:

f(x) = c1 J(x2 - 1)dx + c2 = c1 (� - x) + c

Οι αρχικές συνθήκες (6) οδηγούν στο σύστημα {-�C 1 + C = -1 { _ 3 3 C ι - -2 <:::> 2 , οπότε θα είναι - c + c = 1 c = O 3 1

f( ) 3 ( χ 3 ) I 3 3 Μ 'δ , χ = - - - χ = - χ - - χ . ε τον ι ιο τροπο

2 3 2 2

για ν=4 προκύπτει το πολυώνυμο

f(x ) = _!_ (5 x 7 - 2 1χ 5 + 35χ 3 - 3 5χ) . Εύκολα δια-1 6

πιστώνεται ότι τα παραπάνω πολυώνυμα ικανο­

ποιούν τις υποθέσεις του προβλήματος.

Λύσεις έστειλαν επίσης οι συνάδελφοι: Ροδόλ­

φος Μ πόρη ς - Δ άφνη , Β αγγέλη ς Μ ουρούκος -

Αγρ ίνιο , Κ ωνστ αντίνος ΤζαγκαριΊκη ς - Χανιά . ΑΣ ΚΗΣΗ 1 53 ( ΤΕΥΧΟΥΣ 7 1 )

Ν α δειχθεί ότι δεν υπάρχουν θετικοί ακέραιοι α­

ριθμοί χ,y,z,α,β,γ που να ικανοποιούν τις συνθ11 -

κες : x 3 + yz = α3 , / + zx = β3 και z3 + xy = γ3 •

(Προτάθηκε από τον συνάδελφο Α ΓΙ ΟΣΤΟΛΟ

ΚΑΖΑΚΟ Π ΟΥ Λ Ο - Θεσ/νίκη) .

Λ ΥΣΗ ( από τον ίδιο).

Υποθέτουμε ότι υπάρχουν θετικοί ακέραιοι x,y,z,α

,β,γ που ικανοποιούν τις συνθήκες του προβλήμα­

τος. Έστω χ = max(x, y, z) . Τότε

χ3 < χ3 + yz < χ3 + χ2 < (χ + 1)3 και λόγω της πρώ­

της των σχέσεων θα είναι: χ 3 < α3 < (χ + 1 )3

ΕΥΚΛΕΙΔΗΣ Β ' 74 τ.2/75

Page 78: Ευκλειδης Β 74

-------------- Ο Ευκλείδης προτείνει . . . -------------­Άτοπο, αφού μεταξύ διαδοχικών ακεραίων δεν υ- μεταβλητά σημεία Γ, Δ, Ε εκτός της ευθείας ΑΒ.

πάρχει ακέραιος. Στο ίδιο άτοπο καταλήγουμε είτε Θεωρούμε σημείο Ζ του ΑΓ τέτοιο, ώστε

δεχθούμε ότι y = max(x , y, z) και χρησιμοποιή- ΖΑ = 2 · ΖΓ , σημείο Η του ΒΖ τέτοιο, ώστε

σου με την δεύτερη συνθήκη είτε δεχθούμε ότι ΗΒ = 3 · ΗΖ . Η ευθεία ΓΗ τέμνει την ΑΒ στο ση-

Ζ = max(x , y, z) και χρησιμοποιήσουμε την τρίτη μείο Κ . Θεωρούμε σημείο Θ της ΑΔ τέτοιο, ώστε

συνθήκη . Λύσεις έστειλαν επίσης οι συνάδελφοι: ΘΑ = 3 · ΘΔ , σημείο Ι της ΚΘ τέτοιο, ώστε

Σάββας Π ιπ ίνος- Ρόδος,

Ά Σ Κ Η Σ Η ι 54 του (τεύχους 7 1 ) στην αποδεικτέα

. . ι θ . ι ι σχεση αντι του - να τε ει το - . 2 27

Π ΡΟΤΕ Ι ΝΌΜ ΕΝΑ Θ Ε ΜΑ Τ Α

ι 68. Να βρεθούν οι τιμές των ακεραίων αριθμών

x , y που επαληθεύουν την εξίσωση

2 2 (Π , , , δ λ χ + y = χ - y . ροτεινεται απο τον συνα ε φο

Γ Ι Ω Ρ ΓΟ Α Π ΟΣΤΟΛ Ο Π Ο Υ ΛΟ - Μεσολόγγι )

ι 69 . Σημείο Κ βρίσκεται στο εσωτερικό ενός

τριγώνου ΑΒΓ. Φέρουμε την ΑΔ// ΒΓ έτσι,

/ Κ = 4 · /Θ , σημείο Λ της ΑΕ τέτοιο, ώστε

ΛΑ = 3 · ΛΕ και σημείο Μ της ΚΛ τέτοιο, ώστε

ΜΚ = 4 · ΜΛ . Να δειχθεί ότι οι ευθείες ΔΙ και ΕΜ διέρχονται από το αυτό σημείο της ΑΒ . (Προτείνε­

ται από τον Χημικό Δ Η Μ Η Τ Ρ Ι Ο Κ Α Ρ Β ΕΛΑ -

Πεύκη )

ι 73 . . Να δειχθεί ότι για οποιουσδήποτε θετικούς πραγματικούς αριθμούς α, β, γ ισχύει ότι:

α2 β: v2 α + β + γ - + � + � + 3 2 2 · --::Ξ==-'-β2 γ- α- �

(Προτείνεται από το\' συ\'άδε/.φο Γ Ι Ω Ρ Γ Ο Λ Π Ο­ΣΤΟΛ Ο Π ΟΥ Λ Ο - Μεσοl.ότιι )

ώστε να είναι ΑΔ = ΒΓ . Αν Ε είναι η τομή της 1 74 . Να δειχθεί ότι αν οι θετικοί ακέραιοι αριθμοί

ΚΔ με την ΑΓ, τότε να δειχθεί ότι ισχύει:

(ΑΕΔ) = (ΑΒΚ) + ΓΕΚ) .

(Προτείνεται από τον Χημικό Δ Η Μ Η Τ Ρ Ι Ο Κ Α Ρ -

8 ΕΛΑ - Πεύκη )

ι 7 0 . Να βρεθούν οι τιμές των θετικών πραγματι­

κών αριθμών α, β, γ που ικανοποιούν τις ισότητες:

(Προτείνεται από τον συνάδελφο Γ Ι Ω Ρ ΓΟ Α Π Ο­

ΣΤΟΛ Ο Π Ο Υ Λ Ο - Μεσολόγγι )

1 7 1 . Να δειχθεί ότι για οποιουσδήποτε θετικούς

πραγματικούς αριθμούς α, β , γ ισχύει ότι:

α β γ -(α - β) + - (β - γ) + - (γ - α) 2 Ο . β γ α

(Προτείνεται από τον συνάδελφο Γ Ι Ω Ρ ΓΟ N I K H ­TAKH - Σητεία Κρήτης )

ι 7 2 . Δίνονται δύο σταθερά σημεία Α, Β και τρία

α, β , ρ ικανοποιού\· ισότητα

α2 + β2 - α - β = ρ(l + αβ) ( I ) . τότε ο ρ είναι γι-

νόμενο διαδοχικών ακεραίω\ ' .

( Προτείνεται από τον συ\·άδε/.φο λ Π ΟΣΤΟΛΟ

ΚΑΖΑ ΚΟ Π ΟΥ Λ Ο - Θεσ \'ίκη )

ι 7 5 . Σε τρίγωνο ΑΒΓ με ΑΒ=γ και ΑΓ=β , η διχο­τόμος της γωνίας Α τέμ\'ει την ΒΓ στο Δ και τον περιγεγραμμένο κύκλο του τριγώνου ΑΒΓ στο σημείο Ε. Από το Ε θεωρούμε ευθεία (ε) κάθετη στην ΑΒ. Να αποδειχθεί ότι η απόσταση της κο-

ρυφής Α από την ευθεία (ε) είναι ίση με _!_ (β + γ) . 2

(Προτείνεται από τον συνάδελφο ΒΑΣ Ι Λ Ε Ι Ο Ν Ι -

ΚΟΛΆ Κ Η - Θεσ/νίκη ) .

1 76. Έστω μια συνάρτηση f: (0, +oo)-JR τέτοια

ώστε για κάθε χ>Ο να ισχύει f(x)>O και

f(f(x)+y)=xf( l +xy) . Να βρείτε τον τύπο της συ­

νάρτησης f. (Προτείνεται από τον συνάδελφο Ν Ι ­

Κ Ο ΑΝΤΩΝ Ο Π Ο Υ Λ Ο -Ίλιον)

ΕΥΚΛΕΙΔΗΣ Β' 74 τ.2/76

Page 79: Ευκλειδης Β 74

�ε αφορμή το άρθρο του συναδέλφου Σταύρου Σταυρόπουλου που δημοσιεύεται στο τεύχος 74, παραθέτουμε δύο ακόμη αποδείξεις της πρότασης:

1 +9αβγ Αν α,β,γ > Ο καια+β+γ = 1, τοτεαβ+βy+γα:::; .

4 (Ευκλείδης Β ' τ .69, 2008 . σελίδα 79) 1 ' 1 Απόδειξη (Σωτήρη Ε. Λουρίδα) (�ε ομογενο­ποίηση των δύο μελών) Αρκεί: 4( α+β+γ) ( αβ+βy+γα) :=:;( α+β+γγ +9αβγ , ή

Ο :::; (α + β + γ ) [( α + β + γ)2 - 4αβ - 4βγ - 4γα J + 9αβγ

ή ο::; ( α+β+γ) ( α2 +β2 +γ2 -2αβ-2βy-2γα) +9αβγ , ή ο :::; α3 + β) + γ) + 3αβγ - α2β - αβ2 -β2γ -βy2 - γ2α-γα2

ή Ο :::; α3 - α2 ( β + γ ) + αβγ + β3 - β2 (α + γ ) + +αβγ + γ3 - γ2 ( β + α) + αβγ

ή Ο:::;α( α-β) ( α-γ) +β(β-α) {β-γ) +γ( γ-α) ( γ-β) ( *) . Η τελευταία ισχύει, όπως είδαμε, γενικά με την εξής μορφή :

r-....:-_:-"' .• Ελλην ι κη Μαθηματ ι κη Ετα ιρεία Καλοκαιρινές Δραστηριότητες

15-30 Ιουλίου 2010

Πληροφορίες : από 20 Δεκεμβρίου 2009 στον Ιστοχwρο της ΕΜΕ: www.hms.gr και στα τηλ. 210 3616532, 210 3617784

αν(α-β)(α-γ)+βν(β-α)(β-γ)+γν(γ-α)(γ-β)� Ο και ονομάζεται ανισότητα Ι . Schur. (Για ν= 1 έχουμε την ισχύ της σχέσης (*)) 2 '1 Απόδειξη (Γιώργου Σ. Τασσόπουλου) (Στοι­χειώδης)) Αρκεί: 4[ ( 1 -β-γ )β+βγ+γ( 1 -β-γ) ]�9( 1-β-γ )βγ+ Ι , ή ( 4-9γ )β2+( 1 3γ-4-9γ2)β+(2γ-1 )2�0, ή ( 4 - 9γ ) β2 + ( 4 - 9γ ) ( γ - 1 ) + (2γ - 1 )2 2:: Ο. Υποθέ­τουμε χωρίς βλάβη της γενικότητας ότι:

min { α, β, γ} = γ < .i, διαφορετικά θα είχαμε 9

β 1 2 4 I

, , α + + γ 2:: - = - > , πραγμα ατοπο.

9 3 Επομένως αρκεί Δ�Ο, όπου Δ = (4-9γ)

2(γ - 1)2 -4(4-9γ) (2γ- l( Πράγματι :

Δ = ( 4 - 9γ) ( -9γ3 + 6γ2 - γ) =

-γ (4 - 9γ ) ( 3γ - 1 )2 ::; ο, αφού Ο < γ < .i . 9

ΕΛΛΗΝ ΙΚΗ ΜΑΘΗΜΑΤΙΚΗ ΕΤΑ ΙΡΕ ΙΑ - - · · - . . 27• ΠΑΜΕΛΛΗΝΙΟ ΣΥΜΕΔΡΙΟ

ΜΑΘΗΜΑΤ Ι ΚΗΙ Π Α Ι Δ Ε Ι Α Σ

Ο ΔΑΣΚΑΛΟΣ ΤΩΝ ΜΑθΗΜΑΤΙΚΩΝ • Οίην Εκrια!i:νο; • V�ί1\• ΈGiVV« · σ�ην Koiνwvtα

ΧΑΛΗΙΔΑ 5·6 ·7 IOEitBPIOY 20 1 0

ΕΛΛΗΝΙΚΗ ΜΑΘΗΜΑτJΚΗ ETAIPEIA

27. ΠΑΝΕΛΛΗΝΙΟ ΣΥΝΕΔΡΙΟ ΜΑΘΗΜΛτΙΚΗΣ ΠΑΙΔΕΙΑΣ ΧΑΛΚΙΔΑ , 5-6-7 NOEMBPIOY 2010

Ο Δάσκαλος των μαθηματικών • Στην εκπαίδευση • Στην έρευνα • Στην κοινωνία.

KPil:I M H Ι Ι Μ Ε ΙΌ Μ Η Ν Ι Α :

Η τι:λική ημερομηνία υποβολής εργασιών για το συνέδριο είναι η 1 0' ΣΕΠΤΕΜΒΡΙΟΥ 201 0.

Γlαρακαλούμ1-: να τηρηθεί αυστηρά το χρονικό περίγραμμα που τίθεται .

ώστε η t:πιστημονικιί επιτροπή να αξιολογήσει χωρίς πίεση χρόνου τις

εισηγήσεις που θα υποβληθούν και να προχωρήσει στην έγκαιρη έκδοση

των πρακτικών του συνεδρίου.

Για πληροφορίες μπορεί τι.: να επικοινων..:ίτε στις διεηθt'>νσεις :

pap-look(a)sch.gr, [email protected] www.emepne.gr

* * * * ι.Jί..ι:κτρονικ() τυχυδμομ�:ίο σU\'t:δρίωι [email protected] - 27synedrί[email protected]

ΕΥΚΛΕΙΔΗΣ Β ' 74 τ.2/77

Page 80: Ευκλειδης Β 74

Τα Μαθηματικά μας διασκεδάζουν

Παναγιώτης Π. Χριστόπουλος

•:• Το πληκτρολόγιο Η θέση των γραμμάτων στο πληκτρολόγιο των Η/Υ είναι ίδια με αυτή που είχαν στ ις γραφομηχανές.

Η θέση τους αυτή εξυπηρετούσε το τυφλό σύστημα γραφής QWERTY. Η συχνότητα που eπαναλαμβάνεται στα κείμενα π. χ. το Α είναι μεγαλύτερη από αυτή του Ξ. Έτσι έπρεπε το Α να πατηθεί μe το μικρό δάκτυλο του αριστερού χεριού και το Ξ με το δείκτη του δεξιού, αντίθετα δηλαδ 1Ί από την eυκολία λειτουργίας των δακτύλων για να μην μπερδεύονται οι μοχλοί των πλήκτρων.

·:· τ ο βαρέλι Κάθε μέρα όλοι καταναλώνουμε 85 εκατομμύρια βαρέλια πετρελαίου . Ένα βαρέλι = 1 59 λίτρα. Το

Brent είναι ελαφρύ μείγμα (38 ΑΡΙ) με λίγο θείο, του Opec είναι βαρύ (22 ΑΡΙ) . •:• Η ουγγιά

Ουγγιά (oz) μονάδα μάζας αλλά χρησιμοποιείται ως μονάδα βάρους για πολύτιμα μέταλλα. ι (oz) = 3 1 , 1 04 γραμμάρια.

•:• Παράξενος πολλαπλασιασμός Αν έχετε να πολλαπλασιάσετε 23χ45 κάνετε το σχήμα που βλέπετε και μετράτε τα σημεία τομής των

ευθειών.

Π ο λλαπλασιάστε τώρα τους αριθμούς 3 l 2x l 34

Τα μαλλιά της δασκάλας

8 ( 1 0+ 1 2) I � 1 5

8 2 � 23 5

(8+2) 3 5 Το αποτέλεσμα είναι: I 035

3 (9+ 1 ) (2+3+ 1 2) (6+4) 8 3 1 0 1 7 1 0 8 3 1 � 1 0+ 1 I � 1 7+ 1 1 � 0 8 3 + 1 8 ο 8 Το αποτέλεσμα είναι : 4 1 808

Μια μέρα συναντήθηκαν στο δρόμο τρείς κοπέλες η Έλλη Μαύρου ιατρός, η Νίνα Κόκκινου δασκάλα και η Σοφία Λευκού ηθοποιός, όλες φορούσαν τα κόκκινα καπέλα τους. Μια από τις κοπέλες είπε: Εγώ

ΕΥΚΛΕΙΔΗΣ Α ' 74 τ.2!78

Page 81: Ευκλειδης Β 74

-------------- Τα Μ αθηματικά μας διασκεδάζουν

ιχω μαύρα μαλλιά, μια απι) σας έχει {χσπρα και η άλλη κόκκινα, 6μως σε καμία το 6νομα δεν ταιρ ιάζει μι; το χριίψu. των μαλλιών της. Λμισως όλες έβγαλαν το καπέλο τους και η Σοφία απάντησι; : Πολύ σωστά. Ερώτηση : Τι χριίψα ιχουν τα μαλλιά της δασκάλας;

Οι κόρες Ο πατιρας της Μ αρίας ιχc: ι 5 κι)ρες τις : I η Ελένη , 2 '1 Νανά, 3 '1 Νενι, 4η Ν ινί, 5 '1 ; ; μι)πως ξέρετε πι.ί)ς λένι: την 5 '1 ;

Ο Μ αραθώνιος Στο Μαραθιόνιο φιτος όλο ι ι;γιωτέλc:ιψαν εκτός από τρείς αθλητές. Κάποια στιγμι) ο τρ ίτος προσπιρασε το όεύηφο . α) ΊΊ σειρά έχε ι τώρα; β) έχει καλίηερη θέση από τον προ τι;λι;υταίο; γ) αν στην κορδΟ..α φτάσουν τελικά δυο αθλητές ποιος έχει καλύτερη θέση ο δεύτερος ή ο προ τελευταίος;

Αυτοδιαφημιζόμενος αριθμός Γ ράψτε ιναν I ΟψrΊφιο αριθμό που το πράηο από αριστερά ψηφίο του να δηλών α πόσες φορις έχc: ι το μι1δέν, το δεύτερο πόσες φορές έχε ι τη μονάδα, το τρίτο πόσες φορές έχει το δί)Ο, κ .ο .κ .

Ο Τυφλός Τ ρείς φ ίλο ι ( ο ένας είναι τυφλδς) συμφι.ί)νησαν να πα ίξουν ως εξ ι) ς : Έχουν 2 πράσινες και 3 κόκκινες κύ.ρτες. Πα ίρνει ο καΟινας στην τύχη από μια κάρτα και την στηρίζει στην πλάτη του φ ίλου του . Κερόίζει όποιος μαντιψει τ ι χριίψα {;χει 11 κάρτα που ε ίναι στην πλάτη του . Οι κάρπς που μένουν δεν αποκαλύπτονται . Ο καθένας βλέπει τις κάρτες των άλλων αλλά όχι τη δ ική του . Όταν άρχισε το παιχνίό ι ο πριίηος είπε δεν ξέρω κα ι ο δεύτερος το ίδιο . Τότε ο τρίτος φίλος που είναι τυφλός τους ε ίπε το χρώμα της κάρτας του. 'Η ταν σωστό και κέρδισε . Τι χριίψα είπε ; Πώς το βρ ιi κε ;

Μ αντέψτε Ζητήστε από το φ ίλο σας να σας πι;ι δυο 3ψι)φ ιους. Χωρίς κανένα υπολογισμό πείτε το γ ινόμενό τους συν το γ ινόμενο του ενός επί ένα δ ικό σας αριθμό π. χ. 752, 647 τότε (752χ647)+(75 2 χ τον δ ικό σας 352)

= ;

Το άθροισμα Ζητι)στε uπό το φ ίλο σας να σας πει ένα 3 ψι)φ ιο . Πε ίτε του αμέσως το άθροισμα που θα προκύψει αν προσΟισετε σε αυτόν 4 ακί>μα J ψήφιους (2 ο φίλος σας και δυο εσείς) . Ποιο είναι;

Οι ψευδαισθήσεις « Ν α πιστεύετε τυ. μισά από όσα βλ{;πετε και τίποτα από όσα ακούηω

Ακουστική απάτη ΖητιΊστι; απί> το φίλο σας να προσθέσει από μνΙΊμης 2040 και 2 040 όταν σας πει το αποτέλεσμα 40�0 εσείς το επαναλαμβάνετε δυνατά ως τέσσερα ογδόντα, και I Ο ρωτάτε ; Ο φίλος απαντά τέσσερα ενενιΊντα. Τέσσερα ενενήντα επαναλαμβάνετε δυνατά, και I Ο; Και θα ακούσι:τε 5 χιλιάδες ! ! !

Οφθαλμαπάτη α) Πόσα πόδια έχει ο Ελέφαντας;

γ) Ο Ήλιος και η Σελήνη όταν ανατέλλουν έχουν

μεγάλο μέγεθος; Φωτογραφήστε τα και θα δείτε.

�<=ΥΚΛΕ ΙΔΗΣ Α' 74 τ.2/79

Page 82: Ευκλειδης Β 74

------------- Τα Μαθηματικά μας διασκεδάζουν -------------Ερώτηση α) Πώς μπορεί να ενημερώσει ο κωφάλαλος τον φίλο του που είναι τυφλός ότι τον φωνάζει η γυναίκα του; β) Ο ταχυδακτυλουργός δείχνει λίγα φύλλα από μια τράπουλα και μας ζητά, ακόμα και από την τηλεόραση , να διαλέξουμε ένα χωρίς να το πούμε. Ύστερα με μια κίνηση μας δείχνει πάλι τα χαρτιά αλλά λείπει το δικό μας. Πώς έγινε; γ) Αν μέσα σε ένα κείμενο γράψουμε « . . . το πωτρο και το τλευτιαεο γμάρμα . . . » συνήθως διαβάζουμε αυτό που θα έπρεπε να λέει. Γιατί;

Ευχαριστούμε όλους τους συναδέλφους που στηρίζουν τη στήλη . Τον μηχανολόγο Τάσο Ιωσήφ για τις προτάσεις του καθώς και το συνάδελφο Κοπάδη Θανάση για την τόσο ωραία εργασία του στις οπτικές ψευδαισθήσεις.

Απαντήσεις στα μαθηματικά μας διασκεδάζουν. Τα μαλλιά της δασκάλας : Η κοπέλα που είπε ότι έχει μαύρα μαλλιά δεν μπορεί να είναι η Μαύρου αφού της απάντησε η Σοφία Λευκού, άρα η πρώτη που είπε ότι έχει μαύρα μαλλιά ήταν η δασκάλα Νίνα Κόκκινου.

Οι κόρες : Μαρία

Ο Μ αραθώνιος : α) δεύτερος β) ο δεύτερος είναι ο προ τελευταίος γ) στους δύο ο προ τελευταίος είναι πρώτος.

Α υτοδιαφημιζόμενος αριθμός

Ι ο Ι ο Ι ο Ι ο Ι ο Είναι μοναδικός.

Ο Τυφλός: Από την απάντηση του πρώτου που βλέπει τις δικές τους κάρτες κατάλαβε ότι δεν έχουν πράσινη κάρτα και οι δύο γιατί αλλιώς θα έλεγε έχω κόκκινη . Μπορεί δηλαδή ένας να έχει πράσινη και ο άλλος κόκκινη ή και οι δύο κόκκινη . Αλλά αν ο δεύτερος έβλεπε στον τυφλό πράσινη θα έλεγε πως η δική του είναι κόκκινη . Αφού και ο δεύτερος είπε δεν ξέρω τότε ο τυφλός ήταν σίγουρος ότι η δική του κάρτα ήταν κόκκινη και κέρδισε.

Οι ψευδαισθήσεις: Ο εγκέφαλος μας θέλει περίπου 1 / 1 0 του δευτερολέπτου για να μπορέσει να μεταφράσει μια εικόνα που δέχονται τα μάτια μας ή έναν ήχο που έρχεται στα αυτιά μας. Αυτό εκμεταλλεύονται πολλές φορές οι ταχυδακτυλουργοί και οι μάγοι ! Στην ψευδαίσθηση που έχουμε για τον ουράνιο θόλο οφείλεται και το μεγάλο μέγεθος του Ήλιου και της Σελήνης που βλέπουμε όταν είναι στην ανατολή ή τη δύση .

Το άθροισμα : Αν π.χ. σας έδωσαν 367 τότε απαντάτε το άθροισμα θα είναι 2365 δηλαδή παίρνετε δυο μονάδες και τις βάζετε στη θέση των χιλιάδων. Αφού τώρα σας δοθούν οι δύο ακόμα αριθμοί π.χ. 536,745 τότε εσείς γράψτε 463 , 254 ώστε 536+463=999,745+254=999 και το άθροισμα όλων θα είναι αυτό που προβλέψατε όταν σας δόθηκε ο πρώτος αριθμός.

Μ αντέψτε : Μας είπαν 752 και 647 απαντάμε 751 .248 δηλαδή 752- 1 και 248 ψηφία που με το75 1 δίνουν 999. Προσοχή στον 647 δώστε 352 ώστε να έχουμε 999 και έτσι (752χ647)+(752χ τον δικό σας 3 52) = 752 . (647+352) = 752 .999 = 752 . ( 1 000- 1 ) = 752000-752 = 75 1 .248 .

Ερώτηση : α) Ο τυφλός θα την ακούσει όχι ο κωφάλαλος. Τρόπος επικοινωνίας μεταξύ τους δεν υπάρχει παρά μόνο μέσω της αφής. β) Τα έχει αλλάξει όλα όχι μόνο αυτό το οποίο εμείς προσέξαμε (δηλαδή κρατήσαμε μέρος της εικόνας και όχι όλη την εικόνα) . γ) Εδώ αντίθετα, έγινε οπτική σάρωση του κειμένου και ολόκληρη η λέξη ως πακέτο πληροφορίας έφθασε στον εγκέφαλο μας από το πρώτο και το τελευταίο γράμμα και όχι για κάθε γράμμα ξεχωριστά.

ΕΥΚΛΕΙΔΗΣ Α' 74 τ.2/80

Page 83: Ευκλειδης Β 74

Επιμέλεια: Αντώνης Κυριακόπουλος Γιώργος Τασσόπουλος

Η στήλη αυτή ανήκει στους μαθητές. Εδώ θα δημοσιεύονται απορίες, ασκήσεις και λύσεις που θα μας στέλνουν οι μαθητές. Οι ασκήσεις αυτές δεν είναι απαραίτητο να είναι δύσκολες, γιατί σκοπός της στήλης αυτής είναι να βοηθήσει τους μαθητές στην κατανόηση και την σωστή αντιμετώπιση των Μαθηματικών. Παρακαλούμε τους μαθητές να γράφουν τις προτεινόμενες δικές τους ασκήσεις, (εκφώνηση και λύση) σε ξεχωριστό φύλλο χαρτί (και από τη μία μόνο όψη) με το όνομά τους, την τάξη τους, την πόλη τους και το τηλέφωνό τους και να μας τις στέλνουν σε δύο αντίτυπα.

Προτεινόμενες Ασκήσεις Α ' ΤΛ Ξ Η Λ Y K F: I O Y . ),σ κ η ση 1c.:.\ . Θεωρούμε κυρτό τετράπλευρο ΑΒΓΔ, τα σημεία Κ.Λ εσωτερικά των πλευρών του ΑΔ, ΒΓ αντιστοίχως και το μέσο Ο του ΚΑ. Να δείξετε ότι ένα τουλάχιστον από τα τετράπλευρα ΚΛΒΑ, ΚΛΓΔ βρίσκεται εντός του κύκλου (Ο,�) , όπου Π η περίμετρος του ΑΒΓ Δ.

�υ. Το τετράγωνο ΑΒΓΔ είναι εγγεγραμμένο στον μοναδιαίο κύκλο. Αν Μ,Ν τα μέσα των ΑΒ, ΑΔ αντιστοίχως και η ευθεία ΜΝ τέμνει τα κυρτογωνία τόξα ΑΒ, ΑΔ στα Ε, Ζ αντιστοίχως, τότε: Ι . Να βρεθούν τα μήκη των ΜΕ, ΜΖ, ΕΒ. 2. Να βρεθεί το εμβαδόν του μεικτόγραμμου χωρίου ΜΑΕΜ. Γ Γ\ ΞΗ Λ Y K F: I O Y. ί\ σ κη ση Γ2. Θεωρούμε τη συνάρτηση f ( χ) = max {x3 , συν2χ} (i) Να δείξετε ότι υπάρχει μοναδικό ξ ε ( 0,�) τέτοιο ώστε ξ3 = συν2ξ .

(ii) Να δείξετε ότι f (ξ) = min f ( x ) στο [ο.�] . (όί;j '-'α βρεθεί ro πλήθος των ρ1ζών της εξίσωσης f (χ ) = λ , για κάθε τιμή της -αμtτρσu λ ε [Ο, •: } . \ , c; , : ; 1 1 ρ •J ! ] _' u ί μ ε ν ο υ τεί>zuυ� 7 3 . ι-::n;J περιμi:νουμε λίJσ�:ις για τις υπίιλοιπ�:ς του τ. 73 κ α ι .,. _ ; ,.: π ρ u 7 ι· ι , . , ι ι1 � , . ε � υσκ ι) σc: ι _;; , λ Ι . αβ-ιδ = 1 ΟΟΟα + 1 ΟΟβ + 1 Ογ + δ = = 999α - 99β + 9γ + α + β + γ + δ = = 999α - 99β + 9γ + 2α + 2γ = 1 00 Ια + 99β + l lγ = 1 1α ( 9 Ια � 9β + γ ) . Άρα αβγδ = πολλ. 1 Ι

Χανιώτης Γιάννης - Βαρβάκειο Λύκειο

Δ Δ υ . Έχουμε: i ) Α Β Η = Δ Β Η διότι είναι ορθογώνια, ΒΗ κοινή και ΑΗ=ΗΔ. Άρα ΓΒΔ = ΑΒΓ και ΑΒ=ΒΔ.

Δ Δ Λ Λ Επίσης ΑΒΜ = ΓΜΕ διότι ΑΜ=ΜΕ, ΒΜ=ΓΜ, Μι = Μ2 άρα ΑΒ=ΓΕ, ΑΒΓ = ΒfΈ . Τελικά: IAB = ΒΔ = ΓΕI

άρα. IΆBr = BrE = r.BΔ I ί ί ) Ομοίως

Δ Δ ΜΔΗ = ΜΑΗ :::::> l\ιΙΔ = ΜΑ = ΜΕ :::::> :::::> ΑΔΕ = 90° :::::> ΔΕ // ΒΓ :::::> Λ Λ Λ Λ Βι = Δι , Γι = Ει :::::> Δι = Ει => ΣΔ=ΣΕ:::::>ΣΒ=ΣΓ :::::> ΣΜ .l ΒΓ , ΔΕ Χανιώτης Γιάννης - Βαρβάκειο

Λύκειο

. 2ν-ι 2ν+ι Ε zοι· με : ι ημαΙ - ( συνα) = 1 :::::>

( ημα 1 : -. - . - �- · . ; : ·. - : = ' - ;1,:.: ι 2 - ι συνα 12 =>

( ημα Γ : - = Ι _ • .=

(η μα = Ο ) ή (συνα = Ο) :::::> η μα + συνα = Ι συνα = Ι ημα = Ι Λ ντιστ ρ ο φ α : ημα + συνα = Ι :::::> ημ2α + συν2α + 2ημασυνα = Ι :::::>

ημασυνα = ο :::::> (ημα = ο ) ή (συνα = ο) :::::> συνα = Ι ημα = Ι

:::::> ημ2ν+ια + συν2ν+ια = Ι Χανιώτης Γιάννης - Βαρβάκειο Λύκειο

Β 2 . Ϊ) Έχουμε: α, β, γ ε ]R, α + β + γ = Ο, lαl + lβl + lγl > 0(2) (2) :::::> α ;:: Ο ή β ;:: Ο ή γ ;:: Ο

Έστω β ;:: Ο τότε α=-β-γ. Ι . φ(Μ) = αΜΑ + βΜΒ + γΜΓ = = (-β - γ) ΜΑ + βΜΒ + γ ΜΓ = = -β ΜΑ + βΜΒ - γ ΜΑ + γ ΜΓ = = β (ΜΒ - ΜΑ ) + γ (ΜΓ - ΜΑ) = βΑΒ + γΑΓ

Οπότε το διάνυσμα φ(Μ) είναι ανεξάρτητο του Μ. - -

i i) Έστω φ(Μ) = Ο τότε: - - - Ί -βΑΒ + ν ΑΓ = Ο => βΑΒ = -ν ΑΓ :::::> ΑΒ = - � ΑΓ α ' ' β

σού έ-ι:ουμε β ;:: Ο . :.\ρα τα Α. Β. Γ είναι σι•νευΕιειακά. Τ σάι-.:ι·)ν α � Π α να·; ι ι:)τη � - Ba.c3-:I>.:i:"..:' . .... . _ , .::-. ..='

Page 84: Ευκλειδης Β 74